You are on page 1of 299

Mathematical Association of NSW

Mathe matica l Associ ation of NSW


67 /73 St Hilliers Road, Auburn, NSW 2144, Australia
Email: authors@ masterin ghscmath ematics.c om.au
Website : www.ma steringhs cmathem atics.com .au

Nation al Librar y of Austra lia


Mastering HSC Mathema tics Extensio n 2 Year 12
ISBN: 978-0-9923745-4-9 Paperbac k

Publishe d in Australia by the Mathema tical Associati on of NSW


© Jonathan Le 2020
1st Edition published Septemb er 2020

Cover and interior design by Daisy Chen.


Printing by Round Printing.
Typeset in To\'IEX- by Jonathan Le, Elio A. Farina, Jose L. Leon

Copyri ght:
All rights reserved. Except as permitte d under the Australia n Copyright Act 1968,
no part of this publicati on
may be reproduc ed, stored in a retrieval system, or transmit ted in any form or by
any means (electroni c,
mechanic al, photocop ying, recording , or otherwise ) without the prior permissio n
of the authors . The moral
rights of the authors have been asserted. Enquirie s are to be made to the authors
of this book.

Copyri ght for educat ional purpos es:


Where copies of part or the entirety of the book are made, the law requires that the
educatio nal institutio n, or
the body that administ ers it, provide a remunera tion notice to the Copyrigh t Agency
Limited, who may be
contacted as follows:
Copyrigh t Agency Limited
Level 15, 233 Castlerea gh Street
Sydney NSW 2000
Telephone: (02) 9394 7600
Email: info@cop yright.co m.au

Errors:
Whilst the authors have taken utmost care with the editing and checking of this
book, the authors cannot
guarante e that all errors have been found and corrected . If you identify an error,
please contact the authors
directly at authors@masterin ghscmath ematics.c om.au

Disclai mer:
The authors have taken all possible and reasonab le measures to acknowle dge and
credit any sources of
material. We apologise for any accidenta l infringem ent of copyrigh t and welcome
informat ion that would
redress this situation .
Features of this book
This book is suitable for all students studying the RSC Mathematics courses. It has been designed in
a thoroughly organised manner to help students master each syllabus topic in the new Stage 6 RSC
Mathematics Extension 2 course. This book will teach, consolidate, test and challenge students. It is
an essential resource for all students and teachers.

In flavour with the new course, this book has the following features:

• Interpretation questions.
• Modelling and application problems.
• Verification questions.

Within each chapter, there are subsections divided as follows.

Fundamentals
The carefully constructed fundam entals section appears before the main body of questions. The
purpose of this section is to

• test all key formulae , definitions , concepts and t heory.


• test essential mathematical terms and language through doze-passages .
• ensure that the student has knowledge of t he essential prerequisites.
• provide a summary of basic requirements for the topic.

Questions
This is the main body of questions with the following features.

• Step-by-step questions to assist the student with more difficult problems.


• Carefully graded exercises.
• "Show"-type questions, both guides the student, and offers good exam preparation.
• Proofs and explanations to strengthen understanding and develop problem-solvi ng skills.
• Application questions to demonstrate future uses of learned theory.
• Technology-ba sed questions to teach and reinforce concepts.

Challenge
These are more difficult questions that provide

• a challenge for students wishing to test their mastery of the topic.


• rigour and higher-order thinking skills.
• extension and more in-depth treatment of the unit of work.
Chapter Review
This section appears at the end of every chapter, and offers the following.

• Revision and consolidation of the previous exercises.


• Questions that require a combination of ideas from previous exercises.

Investigations
These tasks are potential assignments and research projects. Teachers may use and adapt these to
cover the new NESA requirements on investigative assessment tasks. This section provides for the
student

• application and modelling scenarios.


• research tasks involving data collection and analysis .
• scaffolding of learning tasks.
• open-ended style problems for discussions.
• opportunity to use appropriate technology effectively in a range of contexts.
• opportunity for students to demonstrate critical thinking.

Answers

• Quick answers to questions.


• "Show" and "prove " answers can be found in the full worked solutions.

Full worked Solutions

• Can be found online for free, or a full-colour hard copy purchased for convenience.
• Provide complete worked solutions to all questions , except investigative tasks to maintain the
open-ended nature of the tasks.
• Includes several alternative solutions to problems, where possible.

Jonathan Le

B.Sc. Pure & Applied Mathematics (8yd)

. I
I
Acknowledgements

From Jonathan:
Thank you to my family and friends for your support and encouragemen t.

Thank you to Jack Tiger Lam, Matthew Cash, and my lovely students for helping to proof-read the
original manuscripts of this text.

Thank you to Anne for giving me the opportunity to create this book.

Images:
Alexandre Godreau, Alvaro Pinot , Danuel Straub , David Jorre, Gerson Repreza, Jason Leung, Joakim
Honksal, Lucas Gallone, Ricardo Gomez, Sylvie Tittel, Sam Wermut

The authors have taken all possible and reasonable measures to acknowledge and credit any sources
of material. We apologise for any accidental infringement of copyright and welcome information that
would redress this situation.
· 1
Table of Cont ents
Chapter 1: 2K Euler's Formula ........... ..... 94

The Nature of Proof Review Chapter 2 ........ . ........... 96


■ Investigation Task ........... ........ 102
Exercise
Investigation Task ........... ........ 103
1A Language of Proof ........ . .... 2
Investigation Task ........... ........ 104
1B Direct Proof ..... . ........... ... 7
Investigation Task ........... ........ 105
1C Contrapositive ........... ....... 9
11 Chapter 3:
1D Proof by Contradiction .........
1E Examples and
3D Vectors
13

Counter-examples ........... ... Exercise
1F Algebraic Inequalities .......... 17 3A Introduction to 3D Vectors ..... 108
1G Mathematical Induction ........ 23 3B Proofs using 3D Vectors ....... 111
1H Inequalities using 3C Vector Equation of a Line ...... 120
Differentiation ........... ....... 32
3D Parameterising 3D Curves ..... 125
11 Inequalities using Integration ... 35
3E Spheres and Circles ........... 134
Review Chapter 1 ........... ......... 41
Review Chapter 3 ........... ......... 137
Investigation Task ........... ........ 45
Investigation Task ........... ........ 140
Investigation Task ........... ........ 46
Investigation Task ........... ........ 141
Chapter 2: Investigation Task ........... .... . ... 142

Complex Numbers Chapter 4:



Exercise Further Integration
2A Arithmetic of Complex ■
Exercise
Numbers ........... ........... . 50
4A Integration by Substitution ..... 146
2B Solving and Factorising
53 4B Trigonometric Integrals ......... 150
Quadratics .. . ..... . .... . . . .....
4C Trigonometric Substitutions .... 152
2C Polar Form and the Argand
Diagram ........... ........... .. 55 4D Harder Standard Integrals ..... 154
2D Vector Representation ......... 59 4E Partial Fractions ........... ..... 157
2E Locus ........... ........... ..... 67 4F t-formula Substitutions ......... 162
2F De Moivre's Theorem .......... 71 4G Integration by Parts .... . ....... 164
- 2G Applications of de Moivre's 4H Reduction Formulae ........... 166
Theorem ...... .. .. . .... . ....... 75 41 Further Substitutions ......... . . 171
2H Roots of Unity ........... ....... 81 Review Chapter 4 ........... ......... 173
21 Applications of Roots of Unity .. 85 Investigation Task ........... ... . .... 176
2J Solving Polynomials ........... . 89 Investigation Task ........... ........ 177
Investigation Task . . . . . . . . . . . . . . . . . . . 178 5D Resisted Horizontal Motion . . . . 202
Investigation Task . . . . . . . . . . . . . . . . . . . 179
5E Resisted Vertical Motion . . . . . . . 205
Chapter 5: 5F Resisted Projectile Motion ..... 211
Mechanics 5G Inclined Planes and Pulleys .... 215
■ Review Chapter 5 . . . . . . . . . . . . . . . . . . . . 225
Exercise
5A Velocity-Displacement Investigation Task . . . . . . . . . . . . . . . . . . . 229
Equations . . . . . . . . . . . . . . . . . . . . . . 182 Investigation Task . . . . . . . . . . . . . . . . . . . 230
58 Simple Harmonic Motion . . . . . . . 186 Investigation Task . . . . . . . . . . . . . . . . . . . 231
5C Projectile Motion . . . . . . . . . . . . . . . 192 Answers . . . . . . . . . . . . . . . . . . . . . . . . . . . . . . 233
THE NATURE OF PROOF

■ Language of Proof

■ Direct Proof

■ Contr aposi tive

■ Proof by Contr adicti on

■ Examples and Counter-examples

■ Algeb raic Inequ alities

■ Mathematical Induc tion

■ Inequ alities using Differ entiat ion

■ Inequ alities using Integr ation


2 Chapter 1: The Nature of Proof

Exercise 1A
Language of Proof

~ Fundamentals

Fundam entals 1
(a) Expressions like 'The cat is black' or 'It will rain today' are called P - - -- - · They are
denoted by p and q.
(b) If we have

then it means that

'if p , then q'.

This is called a logical i ____ _ or a c _ __ __ statement .

(c) The c _ _ _ of the statemen t is that

q ⇒ _.

(d) If both directions are true, then we say that the statemen t is an 'iff' statemen t , which is
short for _ _ _ __ _ _ _ _ _ __

Fundam entals 2
(a) The negation of a statemen t is the 'o ____ , of the statemen t.
(b) Complete the following negation.

Statemen t: All cats are cute.


Negation : _ _ all cats are cute.

(c) The negation of a propositio n p is denoted by _ _ .

Fundam entals 3
You can also have a negation of a condition al statement . The negation of a condition al statemen t
is the directly contradic tory statemen t t hat 'proves it wrong'.

Statemen t: If I study, I will do well in my exam.


Negation : I studied , but _ _ _ _ __ _ _ _ _ __

The negation of a condition al statemen t is importan t because that's how we disprove statemen ts.

MASTERING MATHEMAT ICS


1A Language of Proof 3

Questio n 1 Let p be the proposit ion

'Bob studies for his exams'

and q be the proposit ion

'Bob will get 99.95 ATAR'.

different matter!
Write down the following in English. Whethe r or not the stateme nts are true is a

(a) p-+q (b) q-+ P (c) rvp-+q

(d) q-+rvp (e) rv (p-+ q) (f) rv p -+ rv q

Questio n 2 Let p be the proposit ion

'It is cloudy'

and q be the proposit ion

'It will rain today'.

Write down the following in terms of p and q.

(a ) If it is cloudy, then it will rain today. (b) If it rains today, then it was cloudy.

(c) It was cloudy and it didn 't rain. (d) It didn 't rain and it was cloudy.

(e) It wasn 't cloudy and it didn 't rain. (f) If it doesn't rain today, then it was not cloudy.

Questio n 3 Write down the negation of the following stateme nts.

(a ) The cat is black. (b) If it is a cat, it has a tail.

(c) If it doesn't have four legs, it is not cat. (d) If I study, I will do well in my exam.

Questio n 4 [Negation of 'and ' and 'or']

Write down the negation of the following stateme nts.

(a ) Bob likes swimmi ng and running. (b) Bob likes swimmi ng or running.

Questio n 5 Write down the negation of the following stateme nts.

(a) a< b (b) x E [a,b]

(c) Vn E Z, 2n + 1 is odd. (d) 5 divides 2n + 1 for some values of n.

(e)
2
Vm E Z, 3n ~ m such that n = m . (f) =lx E IR such that sin x = 0.
4 Chapter 1: The Nature of Proof

Questi on 6 [Contrapositive]
Let p be the stateme nt

'I am in Sydney '

and let q be the stateme nt

'I am in NSW '.

(a) Write down p----+ q.


(b) Write down rv q ----+rv p.
(c) Comme nt on the truthfu lness of the following.

Questi on 7 [Distinguishing 'negatio n' and 'converse']


Explain the difference betwee n the negation of a stateme nt p----+ q and the
convers e of a stateme nt.

Questi on 8 Write down the following 'iff' stateme nts as two separat e 'if' stateme nts.
(a) A number is divisible by 6 iff it is divisible by both 2 and 3.
(b) A quadra tic has two real roots iff 6. > 0.
(c) A polynom ial P(x) has (x - a) being a factor iff x = a is a zero.

Questi on 9 [Involving numbers]


For each of the following , write down the converse and state whethe r the
converse is necessa rily true.
(a) If a and bare odd, then a+ bis even. (b) If a is odd , then a 2 is odd.
(c) If a> b then a 2 > b2 . (d) If a, b E Q , then a+ b E Q.
(e) If a and b are even, then ab is even. (f) If a numbe r ends with the digit 5, it is odd.

Questi on 10 [Geome try]


For each of the following , write down the converse and state whethe r the
converse is necessa rily true.
.• I
(a) If it is a rhombu s, then it has perpen dicular diagona ls .
(b) If exactly two angles of a triangle are equal, then exactly two sides of the
triangle will be equal.
(c) If the sides of a triangle are a , b, c and a + b = c , then the triangle
2 2 2
is right-an gled.

MASTER ING MATHEM ATICS


1A Language of Proof 5

Question 11 [Graphs]
true.
For each of the following, write down the converse and state whether the converse is necessarily

(a) If f(x) has a vertical asymptot e at x = a, then f(a) does not exist.

(b) If f(x) has a horizonta l asymptot e at y = b, then f(x) = b has no solutions.

(c) If there is a point of inflexion at x = a, then f" (a) = 0.

(d) If f'(a) = 0, then there is a stationar y point at x = a.

Question 12 Write down the following sentences using formal logic notation.

(a) For every complex value of z, there exists some complex conjugate z such that z + z is real.
(b) For every pair of distinct integers a , b, there exists some rational number r between a and b.

(c) For every real r, there exists a rational q that is at most d units away, where d is rational.

Question 13 Write down the following in English.


(a) \Im > n, ::l a E lR such that m - a = n.
(b) If P(x) = CnXn + · · · + c1x + co, where Ck E Z, and ::la E Z such that P(a) = 0, then a I co.

(c) If n -/- k 2 , k E Z, then fo, t}_ Q.

(d) If z E CC, z -/- 0, then ::l w E (C such that zw = 1.


1
(e) If a E JR+, then a+ - ;2: 2.
a

Note from the author: Although it 's nice to be able to read a string of logical symbols with
fluency
forgo
as though it 's another language, keep in mind 'clarity before formality '. Sometime s, it 's okay to
it.
some mathema tical precision for the sake of clarity. Try not to overuse notation for the sake of
6 Chapter 1: The Nature of Proof

0 Challenge Problems

Problem 1 Determi ne if the following are 'iff' statemen ts.

(a) If a and (3 are the zeroes of a quadrati c P(x) and a 2 + (3 2 > 0, then the zeroes
are real.
(b) If a and b are both irrationa l, then a + b is irrationa l.
(c) If a and b are both irrationa l, then ab is irrationa l.

Problem 2 Suppose that f (x) is a function that is continuo us at x = a.


Then lim f (x) = L if v'E > 0, :3 5 > 0 such that
x----+a

IJ(x)-L l<E
for any 0 < Ix - al < 6. What does this mean in English?

Problem 3 Let ck E JR , where k = 0, l , 2, · · · , n. Define the polynom ial


n
P(x) = L ckxk.
k=O
If P(a) = 0, where a E C\JR, then :3 a s.t P(a) = 0. What does this mean in English?

MASTERIN G MATHEMA TICS


1B Direct Proof 7

Exercise 1B
Direct Proof

~ Fundamentals

Fundam entals 1
Describe the general steps to prove that an integer n is
(a) even. (b) odd. (c) divisible by a.

Questio n 1 Prove that


(a ) if m and n are even, then mn is even. (b) if m and n are odd, then mn is odd.

(c) if m and n are even, then m + n is even. (d) if m and n are odd, then m + n is even.

Questio n 2 Prove that


(a ) if n is even, then n 2 is even. (b) if n is even, then n 2 + 1 is odd.
2
(c) if n is odd, then n 2 is odd. (d) if n is odd, then n + 1 is even.

Questio n 3 Prove that if m is even and n is odd, then

(a) m+nis odd. (b) m - n is odd.

Questio n 4

(a ) Prove that n 2 + n is always even.


(b) Prove that n 3 - n is divisible by 6.

(c) Prove that n 2 is even if and only if n is even.

(d) Prove that n is divisible by 6 if and only if n is divisible by 2 and 3.

Let n be odd. Prove that n has a remaind er of 1 when divided by 8.


2
Questio n 5

Questio n 6 Prove that the sum of three consecu tive integers is always divisible by 3.

20.
Questio n 7 Prove that the sum of two consecut ive positive powers of 4 is always a multiple of

n n2 n3
Questio n 8 Prove that + + is an integer \/n E Z.
3 2 6
Questio n 9 Prove that every odd integer is the sum of two consecu tive integers.
8 Chapter 1: The Nature of Proof

Questio n 10 [Application of binomia l expansio ns]


Prove that gn - 1 is divisible by 8.

Questio n 11 Prove that the difference between the squares of any two consecu
tive odd integers is
always divisible by 8.

Questio n 12 Prove that if the sum of the digits of a 3-digit number is divisible
by 3, then the
number itself is divisible by 3.

Questio n 13 Prove that every odd integer is the difference between two consecu tive perfect squares.

0 Challenge Problems

Problem 1 [Modular arithme tic propertie s]

(a) Prove that if a has a remaind er of b when it is divided by n , then a 2 and b2 will have
the
same remaind er when they are divided by n.
(b) Prove that if a has a remaind er of b when it is divided by n , then ac and be will have
the
same remaind er when they are divided by n.

Problem 2 Prove that 'i/n E z+, n ~ 3, ::l p prime such that n < p < n!

Problem 3 Prove that a number is divisible by 8 if and only if the last three digits themselv es
form a number that is divisible by 8.

Problem 4 Let p be a prime number and let q be some positive integer. Find the smallest
value of q such that p + q is never prime.

Problem 5 [Application of the Sophie Germain Identity]

(a) Show that a 4 + 4b4 = (a 2 + 2ab + 2b 2 )(a 2 - 2ab + 2b 2 ).


(b) Hence, show that if n > 1, then n 4 + 4n is composi te.

Problem 6 [Trivial proof]


Prove that no three positive integers a, b and c satisfy the equation

for any integer value of n ~ 3.

MASTERIN G MATHEMA TICS


1C Contrapositive 9

Exercise 1C
Contrapositive

~ Fundamentals

Fundam entals 1
Consider the statemen t p ➔ q.
(a) The contrapos itive is of 'if p then q is

If not q, then not p

or using symbols , expressed as rv q ➔ __ .

(b) The contrapos itive is e ____ _ to the original statemen t , so

Fundam entals 2
(a) Sometime s, it is easier to prove statemen ts indirectly by instead proving their c _ _ _ __

(b) This is called proof by c _ _ _ __

Question 1 Write down the contrapos itive of each of the following statemen ts.
(a) If I can see you, then you can see me. (b) If it is raining , then Bob will drive to work.

(c) If Bob attends, then Mary will also attend. (d) If I miss my train, I will be late to school.

(e) If it is cloudy, it will rain. (f) If I have a cat, it has four legs.

(g) If Bob studies for his exam, he will do well. (h) If you are in Sydney, you are in NSW.

Question 2 Write down the contrapos itive of each of the following statemen ts.
(a) If a number ends with the digit 5, it is odd.
(b) If a and b are odd, then a + b is even.
1
(c) If a is positive, then a+ - is at least 2.
a
(d) If a and b are integers, then ab is an integer.
(e) If a quadrilat eral is a square, it is also a rhombus.
(f) If all angles are equal in a triangle , then it has three equal sides.
10 Chapter 1: The Nature of Proof

Question 3 [Introduc tion to proof by contrapos ition]


Sometime s, we can prove statemen ts indirectly by instead proving the contrapos ition of the statemen
t.

(a) Prove that if n is even, then n 2 is even.


(b) Hence, it follows that if n 2 is o _ _ , then n i s - - ·

Question 4 Prove the following statemen ts indirectly by proving instead the contrapos itive.

(a) If 3n + 7 is even, then n is odd. (b) If n 2 - 6n + 5 is even, then n is odd.


(c) If mn is even, then morn are even. (d) If mn is odd, then m and n are odd.
2 2
(e) If a (b - 2b) is odd, then a and bare odd. (f) If n 3 - 1 is even, then n is odd.
(g) If n is odd, then 8 divides n 2 - 1. (h) If n 2 + 1 is even, then n is odd.
(i) If n 2
- 2n + 7 is even, then n is odd. (j) If n 2 - 6n + 5 is even, then n is odd.

Question 5 Prove the following statemen ts indirectly by instead proving the contrapos itive.

(a) If n 2 + 5n < 0, then n < 0.

Question 6 Prove that if n 2 is divisible by 3, then n is divisible by 3.

Question 7 Prove that if ab is not divisible by 5, then neither a nor b are divisible by 5.

Question 8 Prove that n 2 is divisible by 5 if and only if n is divisible by 5.

Question 9 Prove that if a 2 + b2 is even, then a+ bis even.

0 Challenge Problems ----- ----- ----- ----- ----- ----- ----- ---- 1
I
Problem 1 Let a, b E Z. Prove that if a+ b > 20, then either a > 10 or b > 10. I
1
I
I
Problem 2 Prove that if 2n - 1 is prime, then n is prime. 1
I
L _____ _____ _____ _____ _____ _____ _____ _____ I
_____ _____ ___I

MASTERING MATHEMAT ICS


1D Proof by Contradiction 11

Exercise 1D
Proof by Contradiction

~ Fundamentals

Fundamen tals 1
(a) Proof by contradictio n is another technique to determine the truth value of a proposition
p. We do this by first considering then ____ of p, denoted by rv p.

(b) We prove that rv p is f _ _ by obtaining a contradiction.

(c) If rv p is false , then that means p is t _ _ .

Question 1 Complete the following proof that v12 is irrational.


(a) Suppose that v12 = E, where p , q E _.
q

(b) Also, suppose that p and q have no common f _ _ so that r_q cannot be further simplified.

(c) Squaring both sides, p 2 = _ so p 2 is e _ _ , which implies p is also e _ _ .

(d) Let p = 2n for some n E _ .

(e) Hence, q2 = _ so q2 is e _ _ , which implies q is also e _ _ .

(f) Therefore, both p and q are e _ _ integers, which contradicts the statement in part _ .

(g) Hence, v12 -=f. r_ for any p, q E _ , and therefore v12 is irrational.
q

Question 2 Use a similar proof to the previous question to prove that the following numbers are
irrational.
(a) v'3 (b) ~ (c) v'6 (d) Js

Question 3 Let p be any prime number. Prove that vP is irrational.


Question 4 Let x be any irrational number. Prove that fi is also irrational.

Question 5 Complete the following proof that log 2 3 is irrational.


p
(a) Suppose that log 2 3 =- , where p , q E _ .
q
p
(b) So 2q = _ and hence

(c) The left-hand side is _ whereas the right-hand side is _ , which is a contradiction. Hence,
log 2 3 is irrational.
12 Chapter 1: The Nature of Proof

Question 6 [Irrational proofs and logarithms]


Use a similar proof to the previous question to prove that the following are irrational.

(b) log 3 5 (d) log 4 12

Question 7 Prove that J2 + v'3 is irrational.


Question 8 Prove that there are no integers a and b such that 3a + 12b = 145.

Question 9 Prove that ij2 is irrational.

Question 10 Prove that n 2 - 3 will never be divisible by 4.

Question 11 Let a, b E JR. Prove that if a+ b is irrational, then at least one of a, b is irrational.

0 Challenge Problems

Problem 1 [Alternative way of stating an irrationalit y problem]

Prove that 4n + 3 is never a square number for any n E Z.

Problem 2 Prove that if a and b are odd, then a 2 + b2 is never a square number.

Problem 3 Define the polynomial

P(x) = E:, .
n k

Prove that P(x) = 0 cannot have any double-root s.

Problem 4 Define the polynomial

P (x) = x 3 + px + q

where p, q -=f. 0. Prove that P(x) = 0 cannot have any triple-roots .

Problem 5 Let a be rational and let vb be irrational. Prove that avb is irrational.

Problem 6 Prove that there are infinitely many prime numbers.

MASTERING MATHEMATICS
. I
I
1E Examples and Counter-examples 13

Exercise 1E
Examples and Counter-examples

&fu Fundamentals
Fundam entals 1
A conditional stateme nt has structur e
p ➔ q

It is in
A counter- example to the stateme nt is an element that satisfies p , but q does not follow.
other words the n ____ of the conditio nal statemen t.

Fundam entals 2
you disprove
A stateme nt claims there exists some n such that a stateme nt is true. How would
the stateme nt?

Fundam entals 3
of numbers.
A stateme nt claims that a stateme nt is true for all n E S , where S represen ts some set
How would you disprove the stateme nt?

Questio n 1 Find a counter- example to show that the following stateme nts are
not always true, and
give a possible modifica tion to correct them.

(a) vx E IR, 3x > 2x (b) vx E IR, 2x -1- 2x

(c) vx E IR, 3x < 2x (d) v x E IR, ax = bx ⇒ a =b

(e) vx E IR, R, = X (f) vx E IR, ✓ 1 - sin 2 x = cosx


(h) VX E IR eln x = X
(g) vx, y E IR, lx l + IYI :::; Ix + YI '
(i) vx E IR, sin ( sin- 1 x ) =x (j) Vx E JR, tan- 1 (x) + tan-
1
(~) =~

Questio n 2 Prove that the following stateme nts are false by providin g a counter- example .

(a) vn E z+, 7n + 4 is prime. (b) vn 2


E Z, n + n + l is prime.

(c) \:Jn E z+, n 2 + n + 41 is prime. (d) vn E z+, n 2 + 1 is never divisible by 25.

Questio n 3 Prove that the following stateme nts are false by providin g a counter-example .

(a) If x 2 > 0, then x > 0.


(b) If a and b are factors of n, then ab is also a factor of n.

(c) If n 2 is divisible by 4, then n is divisible by 4.


14 Chapter 1: The Nature of Proof

Questio n 4 Prove that the following stateme nts are false by providin g a counter- example .

(a) If x and y are non-zero integers , then -X is an integer.


y
(b) If x and y are irrationa l, then xy is irrationa l.

Questio n 5 Let f(x) be any continuo us smooth function with domain x E


R Prove that the
following stateme nts are false by providin g a counter-example .
(a) \la E IR, J'(a) = 0 implies a turning point at x = a.
(b) \la E IR, J"(a) = 0 implies a horizont al point of inflection at x = a.

Questio n 6 Determi ne whether the following stateme nts are true or false.

(a) If a cubic polynom ial has roots a, /3, ry, and a 2 + /3 2 + ry 2 < 0, then it has non-real roots.
(b) If m is a perfect square, then y0nn, is irrationa l.

(c) Every odd number is either one more or one less than some multiple of 4.
(d) If a and b divide n, then ab divides n.

(e) If J'(x) > 0 and f(0) = 0, then f(x) 2:: 0 for all x > 0.

Questio n 7 Determi ne if the following are 'iff' statemen ts. If not , provide a counter-example .

(a) If a and bare odd, then a+ b is even. (b) If a is odd, then a 2 is odd.
(c) If a and b are even, then ab is even. (d) If a number ends with the digit 5, it is odd.

Questio n 8 [Polynomials]

Let P(x), Q(x) be polynom ials. Disprove the following stateme nts by providin g a
counter- example .
(a) The sum of two quadrati cs is always a quadrati c.
(b) If P(x) is divisible by (x - a) and (x - b), then it is also divisible by (x - a)(x
- b).
(c) If P(x) has an integer constant term and an integer root a, then a divides the
constan t term.

Questio n 9 [Geomet ry]


Determi ne if the following are 'iff' stateme nts. If not, provide a counter- example .

(a) If a quadrila teral has perpend icular diagonal s, then it is a rhombus .

(b) If two angles of a triangle are equal, then two sides of the triangle will be equal.

(c) If the sides of a triangle are a , b, c and a 2 + b2 = c2 , then the triangle is right-an gled.

MASTERIN G MATHEMA TICS


1E Examples and Counter-examples 15

Question 10 [Involving graphs]

Determine if the following are 'iff' statements. If not, provide a counter-exa mple.

(a) If J(x) has a vertical asymptote at x = a, then J'(a) does not exist.
(b) If f(x) has a horizontal asymptote at y = b, then f(x) = b has no solutions.

(c) If y" = 0 at x = a, then there is a point of inflexion at x = a.

(d) If y' = 0 at x = a, then there is a stationary point at x = a.

Question 11 Consider the following claim.

If n divides (a+ b) and n divides (b + c), then n divides (a+ c).

Prove or disprove the claim.

Question 12 Consider the following claim.

Let n be a positive integer that is at least 2 digits long and divisible by 4. Let r be the remainder
when n is divided by 100. Prove that r is also divisible by 4.

Prove or disprove the claim.


16 Chapter 1: The Nature of Proof

0 Challenge Problems

Problem 1 [Complex numbers]


State whether the following statements are true. If they are not true , provide a counter-exa mple.

(a) If the roots of a quadratic P(x) are a and /3, and a 2 + 13 2 is negative, then P(x) must
have non-real roots .
(b) If a quadratic P(x) has non-real roots , then a 2 + 13 2 will be negative.

Problem 2 [Further functions from Year 11 Extension 1]

Let f(x) be any continuous smooth function with domain x ER State whether the following
statements are true. If they are not true , provide a counter-exa mple.

(a) If y = f (x) has a root at x = a, then y = v7[xj has a vertical tangent at x = a.


1
(b) If y = f (x ) has a turning point at x = a, then y = f (x) also has a turning point at x = a.

Problem 3 [Integration]
State whether the following statements are true. If they are not true, provide a counter-exa mple.

(a) If f( x ) 2: g( x ) for x E [a, b], then l f( x ) dx > l g(x) dx.

(b) If l f(x) dx > l g(x) dx , then f(x) > g(x) for x E [a, b].

(c) If n is any integer, then j xn dx = -n+l


1
-xn+I + C.

MASTERING MATHEMATICS
1F Algebraic Inequalities 17

Exercise 1F
Algebraic Inequalities

if& Fundamentals
Fundam entals 1
(a) If a, b > 0, then a+ b_ 0. (b) If a, b > 0, then ab _ 0.

Fundam entals 2
(a) If a 2 b, then a+ c _b + c. (b) If a ~ b, then a + c _ b + c.

(c) If a 2 b and c > 0, then ac _bc. (d) If a 2 b and c < 0, then ac _ bc.

Fundam entals 3
(a) If a 2 b and b 2 c, then a _ c.

(b) If a 2 b and c 2 d, then a + c _ b + d.

(c) If a 2 b > 0 and c 2 d > 0, then ac _ bd.

Fundam entals 4
2
If a and b are real, then (a - b) _ 0.

Fundam entals 5
a+b ~
If a and b are P - - , then - - 2 vab.
2

Fundam entals 6
If x and y are real, then lxl + IYI 2 1--1•

Question 1 [Drill]
Prove the following statemen ts for a , b > 0 by considerin g

(x -y) 2 2 0

for appropria te choices of x and y.

(a) a 2 + b2 2 2ab (b) a4 + b4 2 2a2b2 (c) a +b22Fo i>

1 1 1 2 a b
(d) a +- 2 2 (e) - 2 + -2 > - (f) - + - > 2
a a b - ab b a -
18 Chapter 1: The Nature of Proof

Question 2 [Demonst rating different proof techniques]


Consider the inequality
(a+ b) 2 2:: 4ab
which is true for all a , b E IR. Prove the inequality by

(a) starting from the left-hand side and working towards the right-han d side.
(b) investigat ing (a+ b) 2 - 4ab and proving that it is positive.
(c) assuming that (a+ b) 2 < 4ab and using a proof by contradic tion.

Question 3 Suppose that x, y > 1. Prove that x 2 + y 2 2:: x + y.

Question 4 Let a, b > 0. Prove the following inequaliti es.

Question 5 Let a > b > 0. Prove the following inequaliti es.

(a) (b)

1 1 8
(c) -2+ -2 > - - -2 (d)
a b - (a + b)

Question 6 Prove that if a 2 + b2 = x2 + y2 = 1, then ax +by:=:; 1.

Question 7 Prove that if a+ b = l , then

Question 8 Prove that (ab + cd) (ac + bd) 2:: 4abcd.

Question 9 Let a , b > 0.


3
(a) Show that a 3 + b3 > _(a_+_b)_
- 4 (b) Hence, show that y/a,3+b3
3 a+b
~ 2:: - -.
2

Question 10 [Three importan t means]


Prove the following inequaliti es for 0 < x < y.
x+y 2
(a) x< - 2- <y (b) X < 0£Y < y (c) X < -1- -1 < y
-+-
x y

MASTERING MATHEMAT ICS


1F Algebraic Inequalities 19

Question 11 [HM/ AM/GM- inequality ]

Define the three expressio ns

A= x +y H = 2x y
2 ' x +y

Prove that H :S G :S A.

Question 12 Prove that

X +y ✓X2 + y2
;-;;:;;-,<
y w :; - 2~- -< 2

for x, y > 0.

Question 13

(a) Prove that (a 2 - b2 )(e2 - d 2 ) :S (ae - bd) 2 .

(b) Hence, prove that (a 2 - b2 )(a4 - b4 ) :S (a 3 - b3 )2.

Question 14 [Classic inequality and applicatio n]

Let a , b, e > 0.
(a) Prove that a 2 + b2 2 2ab.

(b) Write down two more similar inequaliti es involving a, b and e.

(c) Hence, show that a


2
+ b2 + e2 2 ab + be + ae.
(d) Deduce that for x, y ,z > 0

Question 15 Let a , b, e > 0.


(a ) Prove that a 2 + b2 + e2 2 ab +be + ae.
1
(b) Prove that if a+ b + e = l , then ab +be + ae :S .
3

Question 16 Let a , b, e > 0.

(a ) Prove that a+ b 2 2\!'aFJ.


(b) Write down two more similar inequaliti es involving a, b and e.
(c) Hence, show that
(a+ b)(b + e)(a + e) 2 8abe.
20 Chapter 1: The Nature of Proof

Question 17 [Proving AM/GM inequality for n = 3]

Let a , b, e > 0.

(a) Prove that a 2 + b2 + e2 2:: ab +be+ ae.


(b) Show that a 3 + b3 + e3 - 3abe = (a+ b + e)(a 2 + b2 + e2 - ab - be - ae).

a3 + b3 + e3
(c) Hence, show that 2:: abe.
3
(d) By making an appropria te substituti on, deduce that

X +y+Z > 3r;;:;;;::;


_ yxyz.
3

Question 18 Let a, b, e > 0.

+ -ab -> 2.
a
(a) Prove that -
b
(b) Write down two more similar inequalities involving a, b and e.

(c) Hence, show that (a+ b + e)


(-a1+ -1b + -1)e 2 9.
Question 19 Let a, b, e > 0. Prove the following inequalities.

(a) (a + b + e) 2 ::; 3(a 2 + b2 + e2) a+b b+e a+e


(b) --+--+ -- > 6
e a b -
(c) (a ( 1 1 1) 2:: 27
+ b + e) 2 a 2 + b2 + e2 (d) ab(a + b) + be(b + e) + ae(a + e) 2:: 6abe

Question 20 [Proving AM/GM inequality for n = 4]

Let p, q 2 0.
p+q
(a) Prove that - - 2:: y'pq.
2
(b) Let a, b, e, d > 0. Deduce that
a +--
- b + e-+-
d > v4~b d
aoea.
4 -

Question 21 [Proving AM/GM inequality for n = 3]

By making an appropria te choice for d in the inequality in the previous question, prove that

a+b+e
2:: v3aoe.
~

MASTERING MATHEMAT ICS


1F Algebraic Inequalities 21

Question 22 [Techniques from the Advanced course are useful!]

Prove that
CX) 1
L
k=l ( loga b + logb a)
k :S 1.

Question 23 Prove that if a, b, e > 0 and (1 + a)(l + b)(l + e) = 8, then abe :S 1.

Question 24 Let a, b and e be positive.


(a) Prove that a 2 + b + e ~ ab +be+ ae.
2 2

(b) Hence, prove that if x, y , z > 0, then

x y z l 1 1
-y z + -xz + -xy -> -x + -y + -.
z

Question 25 [Bounding the factorial]

~ a+b
(a ) Prove that v au :S - -, where a~ 0 and b ~ 0.
2
(b) If 1 :S x :Sy, show that x(y- x + l) ~ y.

(c) Let n and k be positive integers with 1 :S k :S n. Prove that


vn :Sk(n - k + l) :S -n+l-.
2

(d) For integers n ~ l, prove that


(vnt :Sn! :S n +1
( -2-
)n

Question 26 [Arithmetic-m ean/Harmonic -mean inequality]

Let a, b, e > 0. Prove that

Question 27 [Proving AM/GM inequality for n = 3]

Let a, b, e > 0.
(a) Show that a3 + b3 ?_ abc (~ + D·
a3 + b3 + e3
(b) Hence, show that ~ abe.
3
(c) Deduce that
a3 b3 e3
-b + -e + -a -> ab + be + ae.
22 Chapter 1: The Nature of Proof

0 Challenge Problems -------- -------- -------- -------- ------~


Problem 1 Let a, b, c 2:: 0 and abc = l. Prove that

Problem 2 Let a, b, c > 0. Prove that

a(a - b)(a - c) + b(b - a)(b - c) + c(c - a)(c - b) 2:: 0.

Problem 3 [Combinatoric s useful here]

Let x1, x2, x3, ... , Xn > 0. Prove that

Problem 4 [Nesbitt 's inequality]


Let a, b, c > 0. Prove that
a c 3 b
--+--+ -- > -.
b+c a+c a+b - 2

Problem 5 [Cauchy-Schwarz inequality]


Consider the quadratic polynomial with real coefficients
n
P(x) = I)ak x - bk) 2 .
k=l

(a) Explain why P(x) cannot have two distinct real roots.
(b) Deduce that

(c) Let S = x1 + x2 + X3 + · · · + Xn where Xk > 0 for all l ~ k ~ n.

Show that
-S- + -S- + -S- + ... + -S- > -n- .
2

S - X1 S - X2 S - X3 S- Xn - n- 1

MASTERING MATHEMATICS
1G Mathematical Induction 23

Exercise 1G
Mathematical Induction

&fu Fundamentals
Fundamen tals 1
(a) A first-order recurrence formula is a recurrence where the value of each term depends on
the value of the t erm directly b ____ it.

(b) In order to generate a sequence of numbers, an i ____ value is needed, and then the
other successive t erms of the sequence can be generated by iterating the values back into
the same recurrence formula. Since all other values come from this i ____ value, it is
sometimes called the seed value.

A formula that instantly gives the n term is called the c _ _ -form solution of the
th
(c)
recurrence. For a given recurrence formula , different seed values will generate different
sequences of numbers, and hence will result in different c _ _ -form solutions.

(d) When we use induction, we are proving that the c _ _ -form solution satisfies the recur-
rence. We are NOT attempting to prove the r _ _ __ formula, since it is given.

Fundamen tals 2
For inequality induction problems , there are generally two types of problems.

(a) The first type is proving that

where A and B are single terms. In general, instead of proving that LHS 2: RHS , it is a lot
easier to prove that _ _ _ _ _ __ 2: 0 instead.

(b) The second type is proving that

series-type problems. These types are a lot more difficult because you will often need to
prove a 'smaller' inequality in order to prove the main inequality. These types can similarly
be proven by showing that _ __ _ _ _ _ instead of _ _ _ __ _ _

Fundamen tals 3
For geometry induction problems , there is no single approach that 'always works '. However , you
should always draw a d _ __ _ where possible.
24 Chapter 1: The Nature of Proof

Question 1 [Guided inequality induction problem)

This is a guided proof to show that

for all n 2:: 4, n E Z , using mathematic al induction.

(a) Step 1: P rove t he statement is true for n = 4.

(b) Step 2: Let n = k be any integer value where the statement is true, so

for k 2:: 4, k E Z.

Write down the statement required to prove true for n = k + l.


RTP 2k+l > ----
Note that equality was lost. This is because we already established equality at k = 4, so we need
only to prove ' > ' part of the inequality.
(c) Step 3: Prove that n = k implies n = k + l is true.

LHS - RHS = 2k+l - (k + 1) 2

2:: 2 ( - ) - k 2 - 2k - 1 from assumption

= k 2 - 2k - 1

2
2:: _ - 2 since k 2:: _

>0

.·. LHS > RHS

Question 2 Use induction to prove that

(a) 5n < n! for all n 2:: 12. (b) n 2 2:: 2n + 3 for all n 2:: 3.
(c) n 3 2:: 2n 2 - n for all n 2:: 1. (d) 2n>4nfor alln2::5.
(e) 4n > 10 x 2n for all n 2:: 4. (f) 3n > 2n 2 for all n 2:: 2.

MASTERING MATHEMATICS
1G Mathematical Induction 25

Question 3 [Guided first-orde r recurrenc e induction problem]

This is a guided proof to show that if


Tn = 3Tn-l +2
th
with starting value To = l , then the closed form of the n term is

Tn = 2 X 3n - 1

for all n 2 0, n E Z , using mathema tical induction .

(a) Step 1: Prove the statemen t is true for n = 0.


(b) Step 2: Let n = k be any integer value where the statemen t is true , so

Tk =2 X 3k - 1,

for k 2 0, k E Z.

Write down the statemen t required to prove true for n = k + l.

(c) Step 3: Prove that n = k implies n = k + l is true.

LHS= _ _ _ _ __ using the recurrenc e

= 3(2 X 3k - 1) +2

=2 X 3 X 3k - l

=2 x _ _ - l

=RHS
26 Chapter 1: The Nature of Proof

Question 4 Consider the following first-orde r recurrenc e relations and initial values.

(a) Tn = 2Tn-1 - 1 (b) Tn = l + 2Tn-1


T1 = 3 T1 = 5
Prove that Tn = 2n + 1. Prove that Tn = 6 x 2n- l - 1.
(c) Tn = 8 + 3Tn-1 (d) Tn = 2Tn-1 + 2 - n
T1 = 2 T1 = 3
Prove that Tn = 6x 3n-l - 4. Prove that Tn = 2n + n.

(e) T _ 3Tn-1 - l
n - 4Tn-1 - l
(f) Tn = Tn-1 +n
T1 =1 T1 = 1
n n(n + 1)
Prove that Tn = - - - Prove that Tn = .
2n -1 2

Question 5 [Proving the sum of an AP formula]

Let Tn = Tn-1 + d where T1 = a and a, d E R Prove that

Tn =a+ (n - l)d

for all n E z+.

Question 6 [Proving the sum of a GP formula]

Let Sn - Sn-1 = arn-l where S1 = a and r -f. 1. Prove that

for all n E z+.

Question 7 [Proving a general formula]

Let Tn = pTn-1 + q where T1 = a and p -f. 1. Prove that

Tn = apn-1 + q Pn-1 -
( p-1
1)
for all n E z+.

MASTERING MATHEMAT ICS


1G Mathematical Induction 27

Question 8 [Guided series-inequ ality induction proof]

This is a guided proof to show that


1 1 1 1 1
- + 2-2 + -+···+ -<2- -n
n2 -
12 32

for all n E z+, using mathematic al induction.

(a) Step 1: Prove the statement is true for n = 1.


(b) Step 2: Let n = k be any integer value where the statement is true, so
1 1 1 1 1
-+-+- +··•+- <2--k'
k2 -
12 22 32

fork E z+.

Write down the statement required to prove true for n = k + 1.


1 1 1 1 1
RTP l2 + 22 + 32 + ... + k2 + (k + 1) 2 <
Note that equality was lost. This is because we already established equality at k = 1, so we need
only to prove '<' part of the inequality.
(c) Step 3: Prove that n = k implies n = k + 1 is true.
1
RHS - LHS = (2 - - -) - (~2 + ~2 + ~2 + ... + ~
1 + ___)
k+1 1 2 3 k 2 ( k + 1) 2

2: (2- k!l) - (2-¼+ (k;l) 2) fromassum ption

1 1 1
--- ---
k (k+l)2 k+l

?
k(k + 1) 2

>0

.·. RHS > LHS


28 Chapter 1: The Nature of Proof

Question 9 Use induction to prove the following for n E z+.


(a) t!
k=l
< n
k -
+2
2
(b) t!
k=l k -
> ~
n +l
(c) ~
n
L..t(k-1)
k=l
3 n
4
::;-
4

Question 10
(a) Prove that k(4k + 3) 2 < (k + 1)(4k + 1) 2 for all k E z+.
(b) Prove that

t Fk ::;
k=l
4n + 3 vn
G
for all n E z+.
Question 11

(a) Prove that 2k + 3 > 2 ✓ (k + l)(k + 2) for all integers k ?_ 0.


(b) Use induction to prove that
n l
I: n > 2(vn+1- 1)
k=l V k
for all integers n ?_ l.

Question 12 Use induction to prove that


n l
I: n?_vn
k
k=l V

for all integers n ?_ l.

Question 13 Prove that


n l
I:- <2fo,-1
k=l ~
for all integers n ?_ 2.

Question 14 [The alternating partial harmonic series is never negative]


n l
Prove that I:(-1l- 1
- > 0 for all n E z+.
k=l k

Question 15 [Induction involving geometry]


Prove the following.
n(n - 3)
(a) A regular n-gon has - - - diagonals.
2
(b) 1 2
n non-paralle l lines, no three of which are concurrent , divide the plane into -(n +n + 2) regions.
2
(c) n non-paralle l lines, no three of which are concurrent, have n(n - 1) points of intersection .
2

MASTERING MATHEMATICS
1G Mathematical Induction 29

Question 16 [Induction involving calculus]

(a ) Use induction to prove that

for all integers n 2': 1.

(b) Use induction to prove that


d (
dx xn
1) - n
- xn+l

for all integers n 2': 1.

(c) Prove that

for all integers n 2': 0.

Question 17 [Induction involving combinatorics]

(a ) Prove that the number of non-empty subsets of a set of n elements is 2n - 1.

(b) A room contains n people and each person shakes hands with everyone else exactly once.
n(n - 1)
Prove that there are handshakes in total.
2

Question 18 Use induction to prove that

for all integers n 2': 1.

Question 19

(a) Use induction to prove that

for all integers n 2': 2.

(b) Deduce that

for all integers n 2': 2.


30 Chapter 1: The Nature of Proof

Question 20 [Bernoulli's Inequality]


Use induction to prove that
(1 + xr ~ 1 + nx
for all n E z+ and X ~ - l.

Question 21

(a) Prove that for all n E z+ , there exist unique positive integers Pn and qn such that

(b) Write down a similar result for ( 1 - -/2r.


(c) Hence, find the limit of Pn as n gets large.
qn

Question 22 Use mathematic al induction to prove that

for all n E z+ and X > l.

Question 23 Use mathematic al induction to prove that

x2 x3 xn
ex > l
-
+ x + -2! + -3! + · · · + -
n!
for all n E z+.

Question 24 Use mathematic al induction to prove that

for all n E z+.

Question 25 [Not as simple as it looks!]


Use mathematic al induction to prove that

1 1 1 1 n
-+-+-
2 3 4 +··• +-
2n -> -2

for all n E z+.

MASTERING MATHEMATICS
1G Mathematical Induction 31

0 Challenge Problems

Problem 1 Define a sequence of angles 0k E [ 0, ~ ] where k = 1, 2, 3, ... , n such that


7r
0 < 01 + 02 + 03 + · · · + 0n < 2
Use mathema tical induction to prove that

Problem 2 Prove that a 2n x 2n square grid with exactly one tile removed can be completel y
tiled by L-shaped trominoes .

Problem 3 Prove that

using induction for all n E N.

Problem 4 Use induction to prove that

for all integers n 2". 1.

Problem 5 [Generali sed product rule]


dk
Let j(k)( x ) denote the kth derivative of f( x ), or in other words j(k)(x) dxk (f (x )). Use
induction to prove that
dn uv)
-----;;:;,-( = nL (n) u(n-k)v(k)
dx k=O k
where u and v are functions in terms of x .

Problem 6 [Colouring problem]

The plane is divided by n lines into regions. Prove by induction that it is possible to colour all
regions with 2 different colours such that no two neighbou ring regions have the same colour.

Problem 7 Matthew flips a coin (2n + 1) times. Let P(k) be the probabilit y that Matthew
flips exactly k heads.

(a) Show that P(n) = P(n + 1).


1
(b) Hence , prove by mathema tical induction that Matthew has a probabili ty of -2 of flipping
at least (n + 1) heads.
__ _
L _____ _____ _____ _____ _____ _____ _____ _____ _____ _____
32 Chapter 1: The Nature of Proof

Exercise 1H
Inequalities using Differentiation

&Eb Fundamentals
Fundam entals 1
To prove that f (x ) 2:: 0 for x 2:: a, we can prove two things.
• J'(x) 2:: 0 for x 2:: a

It is importan t to note that both are required to safely conclude that f (x) 2:: 0 for x 2:: a.
(a) Explain what happens if we have only the first condition , but not the second condition .
(b) Explain what happens if we have only the second condition , but not the first condition .

Fundam entals 2
Suppose f(x) has a global minimum stationary point (a , b). This means that f (x) 2:: _ for all
x in the domain of f( x ).

Question 1 Prove the following inequalities.

(a) vx > 0, ex > 1 + x (b) vx>0,s inx<x

Question 2 Prove the following inequaliti es.

x2
(a) \/ x > 0, x 2:: ln ( 1 + x) (b) 'ix > 0, ex > 1 + x +2
x3
(c) 'ix > 0, sin x > x - (d)
6
x2
(e) VX E JR, COS X 2:: 1 -
2 (f) 'ix E [o,~] , x < ln(secx + tanx)
(g) \/ X E [0' 1], sin - l X 2:: X (h) 'ix> 0, tan- 1 x 2:: x - x 3

Question 3 [Not like the other ones!]

Prove that lnx ~ x - 1 for all x > 0.

MASTERING MATHEMAT ICS


1H Inequalities using Differentiation 33

4
Question 4 Consider the curve y = ex ( 1 - ~) .

(a) Find the coordinates of the stationary points.

(b) Sketch the curve, labelling stationary points.


256
(c) Deduce that ex s; ( _ x) 4 for all x < 4.
4

(d) Hence, show that


625 256
256 < e < 81.

Question 5 Let f(x) = ex - x - 1.


(a) Prove that f(x) > 0 for all x =/- 0.
(b) By finding a suitable value of x, prove that e1r > 1re.

lnx
Question 6 Let f(x) = -.
X

(a) Find the stationary point of y = f(x) and show that it is a maximum.
1 1
(b) Deduce that ee > 1r"n.

Question 7 Let f(x) = xn + (2 - x)n.


(a) Show that there is a minimum stationary point at x = l.

2a
(b) By letting x = --b , prove that
a+
a + b) n < an + bn
( 2 - 2

(a) Show that there is a maximum turning point at (n, nne-n).

(b) Sketch the graph for x 2: 0.

(c) Deduce that ( 1 + ¾) n < e.


34 Chapter 1: The Nature of Proof

0 Challenge Problems

Problem 1 Let f(x) = lnx - x + l.


(a) Show that \:Ix> 0, f(x) ~ 0.
(b) Let Pi> 0, Vi = 1, 2, ... , n such that

Pl + P2 + · · · + Pn = 1.

Prove that
n
L ln(npk) ~ np1 + np2 + · · · + npn - n.
k=l

(c) Deduce that

Problem 2 [Simple proof of the AM/GM inequality]


Let ak > 0, where k = 1, 2, ... , n be a set of n positive real numbers.

(a) Prove that x ~ ex-l for all x ER


(b) By finding an appropriate value of x to substitute, prove that

MASTERING MATHEMATICS
. I
11 Inequalities using Integration 35

Exercise 11
Inequalities using Integration

&fu Fundamentals
Fundamenta ls 1
The diagram below shows a function f (x) and a number of upper and lower-bound rectangles.
y

X
a b

Let the total area of the upper and lower-bound rectangles be U and L respectively.

- < t f( x) dx < -

Fundamenta ls 2
The diagram below shows some function f(x) ~ g(x) for x E [a, b].

g(x)

X
a b

It follows that

t J(x)dx > l-- dx

Equality is lost because although t he functions were equal to each other originally at the point
of intersections, their areas are often not equal and so their integrals are not necessarily equal.
36 Chapter 1: The Nature of Proof

Question 1 The diagram below shows a section of the graph of y = -1 . Consider the region x E [l, 2].
X

X
1 2

1
2 < ln 2 < 1.
Use the diagram to prove that

Question 2 The diagram below shows a section of the graph of y = -1 . Consider the region x E [l, t].
X

X
1 t

Use the diagram to prove that

t -
1 - 2
1( 1)
1 - - < ln t < - t - -
t
.

Question 3 1
The diagram below shows a section of the graph of y = -. Consider the upper-boun d
X
rectangle in the domain x E [e, 1r].

e 1f X

Use the diagram to show that e1r > 1re.

MASTERING MATHEMATICS
:1
I
11 Inequalities using Integration 37

1
Question 4 The diagram below shows a section of the graph of y = -t .

n n+x t

Consider the region t E [n, n + x], where x > n.

(a) Prove that ~


1 +:;:;:
< n ln ( 1 + ~) < x.
n
(b) Hence, show that lim
n-+oo
(1 + ~)
n
n = ex.

Question 5 [Harmonic Series]


1
The diagram below shows the graph of y = - . Upper and lower-boun d rectangles of unit width are
X
constructed over the domain x E [l, n].
y

X
1 2 3 n -l n

1 1 1
Define the series Hn = l + -2 + -3 + · · · + -n .

(a) Show that


1
-n + ln n < Hn < l + ln n.

(b) Hence, find two integers which are lower and upper bounds of the following sum.
1 1 1
l+-+-+ ···+--
3 2 2020

(c) Does the series


1 1 1 1
l+-+ -+-+-+ ···
3
2 4 5

have a finite limit?


38 Chapter 1: The Nature of Proof

Question 6 The diagram below shows the graph of y = fi. Upper and lower-boun d rectangles of
unit width are constructed over the domain x E [O, n).

1 2 X
3 3 n- l n

Let Sn = /I + vf2 + vf3 + · · · + fa.


(a) Prove that
2nfo S (2n + 3)fo
3 < n< 3 ·

(b) Hence, find two integers which are lower and upper bounds of the following sum.

Question 7 Let O :S t :S 1.
(a) Prove that

(b) Deduce that for O :S x :S 1


X
-2 -< ln ( 1 + x) -< x.

Question 8 Let O :S t :S 1.
(a) Prove that
1 1
-2 <
- -
1 +-
t 2 <1
- .

(b) Deduce that for O :S x :S 1


X 1
-2 <
- tan- (x) <
- x.

MASTERING MATHEMATICS
11 Inequalities using Integration 39

Question 9 [Proof of a well-known limit]

Let x > 0 and n E z+.


1
(a) Prove that 1 - x < - - < l.
- 1+ x -
(b) Show that
1- ~2n -< n ln (1 + ~)n -< 1.

(c) Deduce that lim


n-+oo
(1 + ~)n n = e.

Question 10 Define the integral for n 2: 2

½ 1
In = { y' dx.
Jo 1 - xn

(a) Show that if m > n, then Im < In.

(b) Hence, show that


1 7f
-2 < I n < -
- 6

for all n 2: 2.

Question 11 [Alternatin g harmonic series]

Consider the series

(a) Show that


S = _ l_ + (-l)n+1___f:__
n l+t l+t'

(b) Hence, show that

( )n+l lnl -tn- dt.


1 - -1 + -1 - · · · + ( -1 )n-1 -1 = ln 2 + - 1
2 3 n o l + t

(c) Hence, prove that


1 1 1
1- - + - - - + · · · = ln 2.
2 3 4
40 Chapter 1: The Nature of Proof

Question 12 [Gregory-Leibniz series]

(a ) Show that

-x2n < _l_ _ (l _ x2 + x4 _ x6 + ... + (-l)n-lx2n -2) < x2n_


- 1 + x2 -

(b) Hence, show that

- - 1- < -7r
- ( l - -1+ -1 - •··+(-1) n- l -1
-) < -
1 -.
2n + 1 - 4 3 5 2n - 1 - 2n + 1

(c) Hence, write down the exact value of the series

1 1 1
1--+- - -+···
3 5 7

Question 13 [Integration by first principles]

The diagram below shows the graph of f(x) = x 2 in the domain x E [O , l] that has been partitioned
into n equal-width sub-interva ls.

X
1 2 n-l
n n n 1

(a) Show that


l n-1 {l l n
n3 ~ k2 < Jo f (x) dx < n3 ?; k2
n
(b) Use the fact that L k2 = !!_(n
6
+ 1)(2n + 1) to show that
k=l

(n-1)(2n -l)
2 <
1
X
fn
J( )d
X<
(n+l)(2n +l)
52
6n o n

(c) Find the limits of the expressions on either side of the inequality as n ---+ oo, and state the
significance of this result.

MASTERING MATHEMATICS
11 Inequalities using ~.ntegration 41

0 Challenge Problems

Problem 1 Find the value of


1 n n2
lim - ~ - -
n-+oo n ~ n 2 + k2 .
k=l

Problem 2 [A way to approximat e ~ ]

The diagram below shows the graph of y = ln x. Upper and lower-boun d rectangles of unit
width are constructed over the domain x E [l, n + 1].

X
2 3 4 5 n n+l

(a) Prove that


n+l
ln n ! < J,
1
ln x dx < ln (n + 1) !

(b) Hence, show that

(c) Enter {1/foo! in your calculator. What is the output, and what might be causing it?
10

10
(d) Hence, find two integers which are lower and upper bounds of {1/foo!
42 Chapter 1: · The Nature of Proof ,-: "fl,
,,

I, I I'

Ch · r.: 1 Re\iiew -·_ .


The Nature of Proof

0 Review

Question 1 Consider the statement below.

If n is an odd integer, then n 3 - n is a multiple of 12.

Either prove it , or disprove it.

Question 2 Consider the statement below.

For all a, b E Z, if a divides b and b divides a, then a = b.

Either prove it, or disprove it.

Question 3 Prove that

(a) the product of two odd numbers is odd.


(b) the sum of two even numbers is even.
(c) n2 - n is always even.
(d) n2 + n + 1 is always odd.
(e) the product of three consecutive integers is always divisible by 6.

Question 4 Prove the following statements indirectly by instead proving the contraposit ive.

(a) If n does not divide ab, then n does not divide a nor b.

(b) If ab is irrational, then either a or b must be irrational.

(c) If 5 is not a factor of mn , then 5 is neither a factor of m nor n.

(d) If a does not divide be, then a does not divide b.

(e) If both ab and a + b are even, then both a and b are even.

(f) If ab is divisible by 5, then a or b must be divisible by 5.

Question 5 Prove that ~ is irrational.

Question 6 Prove that log 4 8 is irrational.

Question 7 Let n be a positive integer. Prove that ✓ 4n - 2 is irrational.

MASTERING MATHEMATICS
Chapter 1 Review 43

Question 8 Determine whether the following statements are true or false. Remember that a
single counter-exa mple is enough to render a statement false , even if it is mostly true.

(a)
2
If a > b, then a 2 > b . (b) If a 2 is not divisible by 4, then a is odd.
2
(c) If n 2 + n is even, then n is even. (d) If n is any integer, then n + n is even.

(e)
1
If a < b, then - > - .
1
(f) If a and b are real , then ba.1s real.
a b
(g) If 3x - 3y is even, then x and y are either (h) If m 2 - n 2 is divisible by 3, then m - n is
both odd or both even. divisible by 3.

(i) If x and y are both irrational, then x y is (j) If xy and x + y are both even, then x and
irrational. y must both be even.

(k) If a has a remainder of 1 when divided (1) If a has a remainder of b when divided
3
by 5, then a 2 has a remainder of 1 when by n, then a 3 will have a remainder of b
divided by 5. when divided by n.

1 1 2
Question 9 Prove that - 2
a
+ -b2 >- -.
ab

Question 10
.
Prove that 1f xk, Yk > 0 for all k = l , 2, 3, ... , n then L -XkYk + -YkXk
n ( ) 2:: 2n.
k=l

Prove that if a 2:: b 2:: 1, then ab + b 2:: a+ b .


2
Question 11

Question 12 Let a, b, e > 0.


(a) Prove that a 2 - ab + b2 2:: ab.

(b) Show that a 3 + b3 = (a+ b)(a


2
- ab + b2 ).

(c) Hence, show that a3 + b3 + c ::> ab ( a ;


3 b) + be C; c) + ac ( a ; c) .
Question 13 Let a+ b + e = 1. Prove the following inequalities .
1 1 1 1
(a) ab + be + ae s; (b) -+ - + - >9
3 a b e -
1 1 1 1 1 1 9
(c) - + - + - >27 (d) - - + -- + - - >-
ab be ae - a+ b b+e a+e - 2
8 a b c
(e) (a+b)(b+e )(a+e)::::; (f) - + - + ->9
27 be ae ab -

Question 14 Use induction to prove that

(a) 3n > n 3 for all n 2:: 4. (b) 2n<n!for all n2::4.


44 Chapter _1: - The Nature of Proof

Question 15 Consider the following first-order recurrence relations and initial values.

(a) Tn+l = 2Tn-l + 1 (b) = 5Tn + 4


Tn+l
T1 = 1 T1 = 4
Prove that Tn = 2n - l. Prove that Tn = 5n - 1.

Question 16 Use induction to prove that

for all integers n 2 7.

Question 17 Use induction to prove that the interior angle sum of an n-gon is (n - 2)1r for all
integers n 2 3.

Question 18 Prove that

for n E z+ using induction.

Question 19 Let x 2 0. Prove the following inequalities .


2
(a) tanx 2 x (b) x 2'. In ( 1+ x +~ )

Question 20 Define the integral for n 2 2

1 l
In=
1 - -dx.
o 1 + xn

(a) Show that if m > n, then Im < I~.


(b) Show that
7f

4 :s; In <1
for all n 2 2.

Question 21 Prove that for x > 0

x2 x2 x3
x - - < ln(l + x) < x - - +- .
2 2 3

I MASTERING MATHEMATICS
·j

I
Chapter 1: Investigation Task 45

. 1
Question 22 The diagram below shows the graph of y = fa. Upper and lower-boun d
rectangles of unit width are constructed over the domain x E [l , n].

X
1 2 3 4 n-1 n

1 1 1 1
Let S = -+-+ -+·· •+-.
fa
n~\/'2 \1'3
(a) Prove that
1
2fo, + fo - 2 < Sn < 2fo, - 1.

(b) Hence, find two integers which are lower and upper bounds of the following sum.

1 1 1 1
-+-+ -+··· +--
vl2020
~ v'2 v'3
46 Chapter 1: The Nature of Proof

~ Investigation Task

Modular Arithmetic

In this chapter, we prove some basic number theory results using direct proof, proof by contra-
position and proof by contradictio n. However, modular arithmetic is a powerful tool that often
does a quick job of many problems that would normally be time consuming.

Question 1 Explain briefly what modular arithmetic is.

Question 2 Complete the following modular arithmetic properties.

(a) If a + b = c, then a (mod n) + b (mod n) = ...


(b) If a =b (mod n), then a + k =...
(c) If a= b (mod n) and c = d (mod n), then ...

(d) If a= b (mod n), then -a= ...

Question 3

(a) Write down the multiplicat ion properties of modular arithmetic.


(b) Write down the exponentia tion properties of modular arithmetic.

Question 4 Find 32020 (mod 4).

Question 5

(a) Explain how you could use modular arithmetic to calculate the last digit of a number.
(b) Find the last digit of 2020 2020 .

Question 6

(a) Define the term modular inverse.


(b) Explain the relevance of the Euclidean Algorithm and Bezout 's Identity in the context of
the modular inverse.

(c) Find 17- l (mod 163).


(d) Explain why 69- 1 (mod 420) does not exist.

Question 7 Modular arithmetic is incredibly important in the field of cryptography. Write a


one-page article with examples and explains how modular arithmetic is relevant in cryptograph y.

MASTERING MATHEMATICS
Chapter 1: Investigation Task 47

q Investigation Task

Truth Tables

In the topic Nature of Proof, we lightly covered some areas of logic like implication s, equivalent
statements , counter-exa mples, contraposit ive, and proof by contradictio n. However, no study of
logic is really complete without including truth tables.

Question 1 The truth table below contains three fundamenta l connectives between p and q.

p q p/\q pVq p ➔ q

T T
T F
F T
F F

(a) Complete the truth table.

(b) Explain how the truth values for p ➔ q are obtained. Your answer should include an
explanation of why two F 's lead to a T.

Question 2 An implication containing three proposition s P, Q and Risa theorem if it returns


true for all truth combinatio ns of P, Q and R. One of the following is a theorem and the other
isn't. Determine which one is a theorem, and find all counter-exa mples for the one that isn't.

(a) (p V r) /\ (q V r) ➔ (p I\ q) V r (b) (p V r) /\ (q V r) ➔ (p V q) /\ r

Question 3

(a) Research and define De Morgan's Laws for logical proposition s.


(b) Give two 'word examples' for each law to demonstrat e how they work.
(c) Prove De Morgan's Laws using truth tables.

Question 4 Use truth tables to verify that the contraposit ive of p ➔ q is logically equivalent.

Question 5 Use truth tables to verify that p ➔ q is logically equivalent to rv p V q.


OMRIJEX NUMB

tic of Com ex umbers

Solving and Factorising Quadratics

Polar Form and the Argand Diagram

Vector. Representation

Locus

De MoiVire's iTheorem

Applications of de Moiv rem

Roots of Unity

pplications of oots

Solving Pol¥nomials
50 Chapter 2: Complex Numbers

Exercise 2A
Arithmetic of Complex Numbers

&fu Fundamentals
Fundamentals 1
If z =a+ ib,
(a) a is called the r _ _ component of z, and is denoted by _ _ .

(b) bis called the i _ _ _ _ component of z, and is denoted by _ _ .


(c) If a = 0, then the number is purely i _ __ _
(d) If b = 0, then the number is purely r _ _ .
(e) The complex conjugate of z is z = _ _ .
(f) The modulus of z is lzl = - - .

Fundamentals 2
Complete the following conjugate properties.

(a) z+w = (b) zw= _ _ (c)


(:J= -
Fundamentals 3
Simplify the following.

(a) z+z (b) z -z (c) zz

Question 1 Simplify the following.


(a) i2 (b) i3 (c) i4 (d) i5

(e) i-5 (f) i2020 (g) i4n+l (h) i2n

Question 2 Let z = 3 - 4i and w = 2 + i. Find the following.


(a) z+w (b) z- w (c) zw
(d) w2 (e) lzl (f) lwl

Question 3 Let z = 2 + 3i. Find the following.


(a) z (b) z+z (c) z- z (d) zz

MASTERING MATHEMATICS
2A Arithmetic of Complex Numbers 51

Question 4 Simplify the following.


l+i 7 - 4i -7 + 24i
(a) 1 - i (b) (c)
2+i 3 + 4i

Question 5 Find x and y in each of the following.


(a) 3x + 4iy = 12 + 24i (b) 5x + iy = (2 - i) (3 + 4i )
10 - 5i . X y .
(c) - -.- = 2 +i (d) --+--=2-i
X + iy 1 + 2i 2- i

z
Question 6 Find z if - - = 1 + i .
l+z

Question 7 Prove the following conjugate properties.

(a) z +w=z+w (b) zw = (z) (w) (c)

Question 8 Prove the following conjugate properties.


(a) z + z = 2Re (z ) (b) z - z = 2i Im (z ) (c) zz = lzl2

1 1 2Re (z)
Question 9 Prove that -
z
+=
z
= I
z 12

Question 10 Prove the following.


(a) z2 + (z) 2
is purely real. (b) z2 - (z) 2 is purely imaginary.

Question 11 Let z = a + ib. Express the real/imaginary parts of the following in terms of a and b.
Hence, write them in the form x + iy , where x and y are in t erms of a and b.
(a) z2 (b) 1 (c) z +l
z z -l

Question 12 Let
a+ bi
z =--
a - bi '
where a, b E R Show that lzl = 1.

k .
Question 13 Find the possible value(s) of k if +ki_ is purely real.
1+ 7,

Question 14 Prove that if z 2 is purely imaginary, then IRe (z) I = IIm (z) I-

Question 15 Prove that if lz - 11 = 1, then lzl2 = 2Re (z).

Question 16 [Parallelogram Law)

Prove that if z and w are any complex numbers , then


52 Chapter 2: Complex Numbers

0 Challenge Problems

Problem 1

z -l
(a) Prove that if - - is purely real , then z is purely real too.
z +l
z- l
(b) Prove that if lzl = 1, then - - is a purely imaginary number.
z +l

Problem 2 Prove that if l2z - ll = lz - 21, then lzl = 1.

Problem 3 Let z and w be complex numbers.


(a) Show that lzl ~ IRe (z) I•
(b) Hence, prove the triangle inequality lz + wl :S lzl + lwl.

Problem 4 Show that if either lzl = 1 or lwl = 1, then I z - ~ = 1.


11- zw
I

Problem 5 Let z1, z2, and z3 be three complex numbers with modulus 1. Prove that

Problem 6 [A way to generate the vertices of the tangential quadrilateral]

Let a and /3 be any complex numbers. Let z be a complex number that satisfies both

z = a(l + ki)
z = ,6(1- ki)
for some k ER

(a) Show that lal = 1/31.


(b) Show that lz-al = lz-/31.
(c) Interpret this result geometrically.

Problem 7 Consider the identity


1 1
-+-=l.
z z

(a) By letting z =a+ ib, show that (a - 1) 2 + b2 = 1.


(b) Find a possible non-real value of z.
(c) Show that O < Re (z) :S 2 and -1 :S Im (z) :S 1.

MASTERING MATHEMATICS
2B Solving and Factorising Quadratics 53

Exercise 2B
Solving and Factorising Quadratics

~ Fundamentals

Fundamentals 1
Expand and/ or simplify the following .
(a) a+ a (b) cw (c) (z-a)(z - p)

Fundamentals 2
A monic quadratic polynomial P( z ) has complex roots a and a.
(a) The quadratic polynomial can be expressed as the product of complex linear factors.

P( z ) = (z - _ )(z - _ )

(b) Expand P(z).


P(z) = z 2 - _ _ z+ _

Fundamentals 3
(a) To solve a quadratic with complex coefficients, we can use the q _ _ _ formula.
(b) The d _ __ _ will most likely be a non-real number.
(c) Simplify the s _ _ root term , and remember that plus/minus of plus/minus is overall just
plus/minus.

(d) Once you obtain the two roots, you may like to use the s __ /P - - - of roots to verify
that your roots are correct .

Question 1 Express each of the following in the form x + iy, where x and y are real.
(a) ✓3 +4i (b) Js- 6i (c) ✓5 - 12i
(d) ~ (e) J35 + 12i (f) Jg - 40i

(g) ✓7 + 24i (h) ✓ -21+20i (i) J-24 + lOi

Question 2 Solve the following quadratic equations and hence express them as the product of linear
complex factors.
(a) z 2 +16=0 (b) z 2 + 2z+5 = 0 (c) z 2 +z+l=O

Question 3 Write down the roots of the following quadratics.


(a) (z - 1 + 2i)(z - 1 - 2i) = 0 (b) (z+3+4i)(z +3-4i)=0
54 Chapter 2: Complex Numbers

Question 4 Solve the following quadratic equations.


(b) iz 2 - 2(1 + i)z + 10 = 0

Question 5 The quadratic equation z 2 - (3 + i)z + k = 0 has a root 1 - 2i. Find the value of k.

0 Challenge Problems

Problem 1 Express each of the following in the form x + iy, where x and y are real.
(a) vfl (b) ✓4 + 3i (c) ✓ - 12 + 5i

Problem 2 Let z = x + iy represent the square roots of a+ bi. Prove the following.

X
2 lzl + a
=---
2

2
y =--
lzl - a
2

Problem 3 Solve the polynomial equation z 4 = - 7 + 24i.

Problem 4 [Useful result for roots of unity]

A monic quadratic equation has roots a and a. Show that the quadratic is of the form

z2 - 2Re (a) z + lal 2 .

Problem 5 [Conjugate root theorem for the quadratic]

The quadratic polynomial equation az 2 + bz + c = 0 has real coefficients a, b, c and a non-real


root a. Prove that the other root is a.

MASTERING MATHEMATICS
2C Polar Form and the Argand Diagram 55

Exercise 2C
Polar Form and the Argand Diagram

~ Fundamentals

Fundamentals 1
(a) Complex numbers have two components called the r _ and i _ _ _ components.

(b) Hence, they can be represented on a plane called the complex plane, otherwise known as
the A__ diagram.

(c) On the complex plane, the r _ component of z is represented on the x-axis , and the
_ _ _ component of z is represented on the y-axis.

Fundamentals 2
Let z = a+ bi be a complex number.

(a) The distance of a complex number z from the origin is called them _ __

· lz l =

(b) When connected to the origin, the angle that z makes with the positive real axis is called
the a _ _ _ , and it is given by solving

tan0 = - -

where 0 E ( - , _ ]. Use quadrants to determine the correct argument.

(c) Combined together, we get the p _ _ form of z .

z = a + bi = r ( cos 0 + i sin 0) = r __

Fundamentals 3
If z = A cis a and w = B cis (3, then
z
(a) ZW= _ _ __ (b)
w

Fundamentals 4
Complete the following properties.

(a) lzwl = - (b) 1:1=- (c) lkz l = _ if k > 0

(d) arg(zw) = _ __ (e) arg (:) = _ __ (f) arg( k) = _ if k > 0


56 Chapter 2: Complex Numbers

Question 1 Convert the following to the form r cis 0, where r > 0 and 0 E ( -1r , 1r].
(a) 1 +iJ3 (b) -1 + iJ3 (c) 1-iJ3 (d) -1 - iJ3
(e) J3+i (f) -J3+i (g) J3-i (h) -J3-i
(i) 1+ i (j) 1- i (k) -l + i (1) -1-i
(m) 3i (n) - 4i (o) 5 (p) -2

Question 2 Convert the following to the form r cis 0, where r > 0 and 0 E ( -1r , 1r].
(a) -2J3+2i (b) -\/'2 - iv/6 (c) i(l-i)

Question 3 Convert the following to Cartesian form.

(a) 2 cis G) (b) 6 cis (2;)


Question 4 The Argand diagrams below show a plot of z . Find the polar form of z.
(a) (b)
iy iy

J3 ··• ... 1

X X
1 -J3

(c) (d)
iy iy

-1

X X

• -1

Question 5 Let z = - 1 + i. Convert the following to polar form.


(a) z (c) iz (d) z2

Question 6

(a) 2cis m Express the following in the form r cis 0, where r > 0 and 0 E (-1r, 1r].

G)
X 4cis (b) 4cis (~) X 3cis (-n
(c) 6cis (7;) 2cis G)
7

MASTERING MATHEMATICS
2C Polar Form and the Argand Diagram 57

Question 7 Simplify the following by first converting to polar form.

1 + vl3i
(a) (b)
1 + vl3i vl3 + i
1- i i(-vl3 + i)
(c) (d)
vl3 - i (- 1 + i)(l - ivf3)

Question 8

(a) Expand and simplify (2 + i)(3 + i).

(b) Deduce that tan- 1 G) +tan- G) = ~- 1

Question 9 [To encourage use of the properties]

Let z = -vl3 + 3i and w = )2 - iy12. Find the following.

(a) lz l (b) lzwl (c)


1:1 (d)
I~
(e) argz (f) arg(z 2 w) (g) arg ( ;2 ) (h) arg(zz)

Question 10 Let z = 1 + i and w = 1 + ivf3.


(a) Calculate zw and express in polar form.

(b) Deduce that sin -


12 (7w) =
l+vf3
)2 and cos -
2 2
(7w)
12
=
l-yl3
)2 .
2 2

Question 11 Prove the following by letting z = r cis 0.


(a) zz = lzl 2 (b) If lz l = 1, then z = z- 1

Question 12 Let z = cos 0 + i sin 0. Find z 2 using two different methods to prove the following.
(a) cos 20 = cos 2 0 - sin 2 0 (b) sin 20 = 2 sin 0 cos 0

Question 13 [Double-angle formulae can be used to simplify]

Show that
1 1 1
1 + cos 0 + i sin 0 = 2 - 2i tan
(0)2 ·
58 Chapter 2: Complex Numbers

0 Challenge Problems

Problem 1 Let z be a complex number with argument 0 and Re (z) -=f. 0. Show that

lzl - iz
Iz I + iz. = -i(sec0 + tan0).

7r
Problem 2 Let z = cos0 + isin0 and w = sin0 + icos0 where 0 < 0 < .
2
(a) Show that lz + wl = 2sin (0 + ~)-
(b) What is the maximum value of lz + w l? For what value of 0 does it occur?

1 1
Problem 3 Let z = a+ bi and w = -;;, + bi, where a, b > 0.

(a) Calculate zw.


(b) Deduce that

tan -1 (a)
b + tan (b)
-;;, = .
2
-1 1r

z2 - 1
Problem 4 Let z = cos 0 + i sin 0. Show that - 2- - = i tan 0.
z +l
Problem 5 Prove that if lzl = lw l = 1, then

1
arg(z + w) = 2 (argz + argw).

7r
Problem 6 Prove that if lzl = lwl = 1 and 0 < arg z < argw <
2 , then
1
arg(z - w) = (arg z + argw - 1r).
2

Problem 7 Define the polynomial

P(x) = x 2 + 2ax + b,
where a 2 < b. Let the zeroes of P( x) be a and f3, represented by A and B on the complex
7r
plane. Show that if LAOB = , then b = 2a 2 .
2

MASTERING MATHEMATICS
2D Vector Representation 59

Exercise 2D
Vector Representation

~ Fundamentals

Fundamentals 1
(a) If the points A and B are represented by complex numbers a and b respectively, then

(b) Similarly, the quantity b - a represents the vector _ _ .


(c) When we say that a complex number represents a vector , what we mean is that the complex
number represents the p _ _ _ _ _ vector.

Fundamentals 2
Let the point OP
be the position vector of the complex number z , and let a > 0. Describe the
effect on OP
when we multiply z by

(a) 2 (b) cis a (c) cis( - a) (d) i

Note from the author: In this chapter, we will multiply vectors by R cis 0 to rotate the vector. This
is not technic~ sound as we cannot multiply a vector by a complex number. What we really mean
when we say AB x R cis 0 is that the complex number represented by AB
is multiplied by R cis 0.

Question 1 [Practising using AB = b - a]


Let A = 3 + 2i, B = -5 + i and C = 4 - 3i. Write down the complex number representing the position
vectors of the following.

(a) AB (b) BA (c) AC (d) CB

Question 2 [Application of AB = b - a]
Let A= 3 + 2i, B = -5 +i and C = 4 - 3i. Find the complex numbers representing P, Q and R if

(a) AP = s + 4i (b) QB = 6 - 7i (C) Jic = -3 - 5i

Question 3 Let z 1 = 3 + 2i and z 2 = -1 + 4i. Draw the following on separate Argand diagrams.

(a ) z1 + z2 (b) z1 - z2

(d) z1 + ii (e) Z1 - Z1
60 Chapter 2: Complex Numbers

--=----t
Question 4 The diagram below displays a complex number z represented by the vector OA.
Roughly plot the following on the Argand diagram .

... . ;. 4

... 3

-4 -3 -2 -1 1 2 3 4 X
.. .. .:............ :_1
----: -2 . ..... ·...... ,: .. ... ..:.

-3
..... .... . .. : -4

(a) z x 2 cis G) (b) z x ~ cis ( -i)


(c) zxi (d) z x(-i)

Question 5 Let P be the point on the Argand diagram represented by a complex number z .
What complex number should we multiply into z in order to
(a) double the length of OP
and rotate it anti-clockwise by
5
;?
(b) halve the length of OP and rotate it clockwise by i?

(c) preserve the length of OP and rotate it clockwise by i?

Question 6 State the effect of multiplying a vector by


1 1 . 1 1 .
(a) l+i~ (b) 2y0 - 2y0i (c) --+-i
v0 v0
Question 7 Suppose a= 4+6i and bis some complex number such that DAOB forms a right-angled
isosceles triangle and arg b > arg a.

0 X

Find the complex number represented by B.

MASTERING MATHEMATICS
2D Vector Representation 61

Question 8 The diagram below shows the vector represented by a complex number z .

·4

•. . . . . . ... .. . . : . 1 . . . . . .. ..

-4 -3 -2 -1 2 3 4 X
.:.. :-1
...... • - 2

... ·.. .. ...·.......·..... ·: -3

.: _4

Plot the position vectors represented by the following complex numbers.

(a) iz (b) z2 (c) zz (d) (1 - i)z

Question 9 Let A and C be points representing 1 - i and 3 + 4i respectively. Suppose that AC is


a diagonal of a square ABCD.

Find the complex numbers represented by B and D.


62 Chapter 2: Complex Numbers

Question 10 Let P = 1 + iJ3 and Q = - 2 + Oi be two points of an equilateral triangle L::,PQ R.

iy

Q X

(a ) Draw all possible positions of R.

(b) Find the complex number represented by R for all cases.

Question 11 Let P = -6 - 4i, Q = 6 + 4i and R form an equilateral triangle. Let PQ RS be a


rhombus where the labelling of the vertices is not necessarily in order.

(a) Find the complex number represented by R in the diagram above.

(b) Draw all possible positions of S.

(c) Find the complex number represented by S for all cases.

MASTERING MATHEMATICS
2D Vector Representation 63

Question 12 Let P = 6 + 4i be a point on a right-angled isosceles triangle DO PQ.

iy

p

X
0

(a) Draw all possible triangles DOQP if the right-angle occurs at Q, and find the coordinates of Q
for each scenario.

(b) Draw all possible triangles DO PQ if the right-angle occurs at P, and find the coordinates of Q
for each scenario.

Question 13 Let A and B be points representing 2i and 4 + 6i respectively.

iy
B

Suppose AB is one side of a square ABCD.

(a) Draw the two possible scenarios.

(b) Find the complex numbers representing C and D for both scenarios.

(c) Find the complex numbers representing C and D if AB was instead the diagonal of the square.
64 Chapter 2: Complex Numbers

Question 14 Let OABC represent a rhombus, where A represents the complex number 1 + i.
7f
Let B be in the second quadrant and L_BOC = .
3
iy

0 X

Find the complex numbers that represent points B and C.

Question 15 The origin, w and z form a right-angled isosceles triangle such that lzl = lwl.

iy
z

0 X

(a) Explain why either z = iw or w = iz. Which one is shown in the diagram above?
(b) Deduce that in either case z 2 + w2 = 0.

7f
Question 16 The diagram below shows the origin, w and z forming a triangle with angle
4 at the
origin such that lwl = 2lzl.

iy
w

0 X

Prove that w 4 + 16z4 = 0.

MASTERING MATHEMATICS
2D Vector Representation 65

Question 1 7 The origin, w and z form an equilateral triangle.

iy
w

0 X

(a) Prove that z 3 + w 3 = 0.


(b) Expand (z + w)(z 2 - z w + w 2).
(c) Deduce that z 2 + w 2 = zw .

Question 18 Suppose that z1, z2 and z3 are represented by A , Band C respectively on the complex
plane, and z2 - z1 = i(z3 - z1). What type of triangle is DABC?

Question 19 Show that if z1 + z2 + z3 = 0 and lz1I = lz2I = lz3 I = 1, then z1, z2 and z3 form an
equilateral triangle on the unit circle.

Question 20 Suppose that z1, z2 and z3 are represented by the points A, B and C respectively on
Z3 - Z1
the complex plane. Show that if A, B and Care collinear, then - - - is a real number.
Z2 - Z1

Question 21 Let a, band c be complex numbers representing the vertices of DABC , as shown in
2
the diagram below. Let w = cos ( ~) + i sin ( ; )-

iy

Show that DABC is equilateral if and only if a+ bw + cw 2 = 0.


66 Chapter 2: Complex Numbers

0 Challenge Problems

Problem 1 Let z1, z2 and z3 form an equilateral triangle as shown below.

iy

2
(a) Show that z - zi = cis (~).
Z3 - Z1 3
(b) Deduce that zr + Z§ + z] = z1z3 + z1z2 + z2z3.

Problem 2 If lzl = lwl , show that z + w is purely imaginary.


z -w

Problem 3 Let z and w be non-real numbers such that lz + wl = lz - wl.


(a) What can we say about the origin, z, w and z + w?
(b) Hence, what is arg (:)?
Z3 - Z1
Problem 4 What can we say about z1, z2 and z3 if - - - is a purely imaginary number?
Z2 - Z1

Problem 5 [Cross-ratio]

Show that if
(z3 - z1)(z4 - z2)
(z3 - z2)(z4 - z1)
is real, then z1, z2, z3 and Z4 are either concyclic or collinear.

Problem 6 [Application of circle geometry]

1 1 1
Prove that if the origin, z1, z2 and z3 are concyclic, then - , - and - are collinear.
z1 z2 z3

MASTERING MATHEMATICS
2E Locus 67

Exercise 2E
Locus

~ Fundamentals

Fundamentals 1
(a) Let z be some general complex number on the A _ __ _ diagram. If z has no restric-
tions , then it is free to be anywhere.

(b) However, if a condition is imposed upon z, then z must lie on a particulars _ of points .
When we connect those points, it often forms some kind of l _ _ , c _ _ , or r _ _ .

(c) The set of all allowable points of z is called the l _ _ of z .

Fundamentals 2
Describe the general shape of the locus of z if z is

(a) exactly r units away from a fixed point C.


(b) at most r units away from a fixed point C.
(c) at least r units away from a fixed point C.
(d) equidistant from two fixed points A and B .
(e) closer to A than it is to B .
(f) inclined at an angle of 0 from the horizontal with respect to a point C.

Question 1 Sketch the locus of z defined by each condition.


(a) lzl = 1 (b) lz - (2 + 3i) I = 2
(c) Iz - 4 - 3i I = 3 (d) lz - 1 - ii = v'2

Question 2 Sketch the locus of z defined by each condition.


7r 7r
(a) arg(z) = (b) arg(z) = - -
3 4
21r 7r
(c) arg(z - 1) = - (d) arg(z + 2) = -
3 6
. 7r 7r
(e) arg(z - (3 - 4i)) =- (f) arg(z +4 - 3i) =
4
3

Question 3 Sketch the locus of z defined by each condition.


(a) lz l > 1 (b) lzl ~ 1
(c) l~ lz-2 1~2 (d) 1 < lz + 1 - 2il ~ 2
68 Chapter 2: Complex Numbers

Question 4 Sketch the locus of z defined by each condition.


(a) Re (z) = 2 (b) -1 < Im (z) :S 1
(c) Re (z) ~ - 1 (d) Re (z) = Im (z)

Question 5 Find the Cartesian equation and sketch the locus of z defined by each condition.
(a) lz - 21 = lz + 41 (b) Iz +iI = Iz - 3i I

(C) Iz - 2i I = Iz + 21 (d) lz - 2 + ii = lz - 4 - ii

Question 6 Sketch the locus of z defined by each condition.


(a) lz - 4il ~ lz - 41 (b) lz - 2 - ii < lz + 1 + 4il

Question 7 Sketch the locus of z defined by each condition.


w 2w 2w
(a) ::;argz< (b) 0:Sarg(z-2)<
3 3 3
31r 7f 7f
(c) 0 :S arg(z - 1 + i) :S (d) - :S arg(z + 1 - iv13) :S
4 3 3

Question 8 Sketch the intersection of each pair of loci.


7f 7f 7f 7f
(a) lz + 1- 2il :S 3 and - :S arg z :S (b) lz - 11 :S 2 and - :S arg(z - 1) :S
3 4 4 4
(c) lz + 21 ~ 2 and lz - ii :S 1 (d) 1 :S Iz I :S 2 and O :S z + z =S 3
(e) lz+zl :S 1 and lz -il :S 1 (f) lz - 3 + ii :S 5 and lz + 11 :S lz - 11

Question 9 Sketch the locus of z defined by each condition.

(a) arg(z - 1) - arg(z - i) =0 (b) arg(z - 1) - arg(z - i) = 1r

Question 10 Find the Cartesian equation and sketch the locus of z defined by each condition.

(a) lz l2 = z + z+ 1 (b) z2 - z2 = 4i (c) z + z = Im (z)


z z 1 1
(d) Iz - i I = Im ( z ) + 1 (e) -=+-=2 (f) -+-=l
z z z z

7f
Question 11 Consider the locus of z defined by arg(z - 2) = .
3
(a) Find the range of values of lzl. (b) Find the range of values of arg (z).

Question 12 Consider the locus of z defined by lz - 1 - ii = 1.


(a) Find the maximum value of lzl. (b) Find the minimum value of lzl.
(c) Find the maximum value of arg(z ). (d) Find the minimum value of arg(z).

MASTERING MATHEMATICS
2E Locus 69

Question 13 Consider the locus of z defined by lz - 4i l = 3.


(a) Find the maximum value of lzl. (b) Find the minimum value of lzl.
(c) Find the maximum value of arg(z). (d) Find the minimum value of arg(z).

Question 14 Find the Cartesian equation and sketch the locus of z defined by each condition.

(a) 2lz - 11= lz + 11 (b) I 2z - 1I = Iz - 2I

Question 15 Sketch the locus of z defined by each condition.


z-l l-z
(a) - - . is purely real. (b) - - is purely imaginary.
z+i z
z- i 1
(c) - - is purely real. (d) z - - is purely imaginary.
z+ l z

Question 16 [Circle Geometry]

Sketch the locus of z defined by each condition.

(a) arg G~ ~) = ~ (b) arg G~ ~) = i (c)


z -
arg ( - -
z+l
1) = -271"
3

Question 1 7 Find and sketch the locus of z if

Question 18 Suppose a, b E IR and a =f. b. Let z be a complex number such that

Find and describe geometrically the locus of z.


Hint: There are two cases here!

Question 19 [A familiar result! ]

Let a and b be complex numbers. Describe geometrically the locus of


2 2 2
lz - al + lz - bl = la - bl .

without explicitly letting z = x + iy.


70 Chapter 2: Complex Numbers

0 Challenge Problems

Problem 1 [Mobius Transformations can map circles to circles]

Consider the expression


+b
W=--,
az
z
where a, bare real non-zero constants. Show that if z lies on the unit circle, then w will always
lie on a circle centred at (a, 0) with radius b.

Problem 2 [Cool way to prove a classic problem!]

Let the origin, z1, z2 and z3 be four points on a circle centred at c in the complex plane.

(a) Describe the locus of w satisfying lw - kl = lwl, where k is a fixed complex number.
, 1 1 1
(b) Deduce that - , - and - are collinear.
ZI Z2 Z3

Problem 3 Show that the roots of

(z + 1r + (z - 1r = 0

are purely imaginary, by considering an appropriate locus.

Problem 4 [More on Mobius Transformations]

Find the locus of z if


kz - 11 = 1
I z- k
where k E IR and k i= ±1.

Problem 5 [Circle of Apollonius]

Let z1 and z2 be fixed complex numbers, and suppose m > n > 0. The locus of z given by

is a circle called the Circle of Apollonius.

m 2z1 - n 2z2
(a) Show that the centre of the circle is - - - - -
m2 -n2
mn
(b) Show that the radius is 2 lz2 - z1 I•
m -n 2

MASTERING MATHEMATICS
2F De Moivre's Theorem 71

Exercise 2F
De Moivre's Theorem

~ Fundamentals

Fundamentals 1
Let z = cos 0 + i sin 0. De Moivre's theorem states that for all n E Z

Fundamentals 2
The following results follow from de Moivre 's theorem for all integer values of n.
(a) lznl = _ _ (b) arg(zn) = _ _ (c) zn = _ _

Question 1 Simplify the following.

(a) (cos 20 + i sin 20) 3 (b) (cos 0 - i sin 0) 5

(c) (cos30 + isin30)- 2 (d) (cos 30 - i sin 30)- 1

Question 2 Simplify the following.

Question 3 Let z = l + i/3.


(a) Find the modulus and argument of z. (b) Express z in polar form.
4
(c) Hence, express z 4 in polar form. (d) Express ( 1 + i/3) in Cartesian form.

Question 4 Express the following in Cartesian form.

(a) (1 + i) 5 (b) (1 + i/3) 6 (c) (- /3 + i)3

(d) (/3-i)7 (e) (1 - i) 4 (f) (1 - i/3) 9

(g) (-/3 - i)6 (h) (- /3 + 3i) 3 (i) (-2+2i) 5

Question 5 Show that


1
. . = cos 0 - i sin 0.
cos 0 + ism 0
72 Chapter 2: Complex Numbers

Question 6 Show that for all n E Z

( cos 0 - i sin 0 f = cos n0 - i sin n0.

Question 7 Let z = cos 0 + i sin 0. Simplify the following.

(a) (b)
2 2i

Question 8 [Proof of de Moivre's theorem]

(a) Use mathematical induction to prove de Moivre's theorem for n E z+.


(b) Deduce that de Moivre 's theorem holds true for n E Z in general.

Question 9 Simplify the following expression.

( cos 30 + i sin 30) 2 ( cos 0 - i sin 0)


4

3
( cos 20 - i sin 20) -

Question 10 Find the values of n for which ( vf3 - i)n is

(a) purely real. (b) purely imaginary.

Question 11 Find the values of n for which (1 + i)n is

(a) purely real. (b) purely imaginary.

Question 12 Find the values of n for which (1 + i-J3r is


(a) purely real. (b) purely imaginary.

Question 13 Consider the complex number

(a) Show that z is always real for n E z+.


(b) Can the same be said for (1 + ir - (1 - ir?
(c) Show that if n is a multiple of 4, then z is an integer.

Question 14 Suppose that z1, z2 and z3 are complex numbers that all lie on the unit circle.
Show that if z1 + z2 + z3 = 0 then
1 1 1
- +- +- =0.
z1 z2 z3

MASTERING MATHEMATICS

I
.I
2F De Moivre's Theorem 73

Question 15

(a ) Show that

(b) Hence, show that (1 + ivf3r + (1 - ivf3r is always an integer for n E Z.

Question 16 Simplify
1 + cos e + i sin e) n
( 1 + cos e - i sin e

Question 17 Prove the following identity.

(1 + COS 20 + i sin 20r + (1 + COS 20 - i sin 20r = 2n+l COSn eCOS ne

Question 18 Prove the following identity.

( 1 + i tan er + (1 - i tan er = 2 seen ecos ne

Question 19 Let z = cos (;) + i sin (;). Show that

1+z + z 2 + ••• + zn- l = l + i cot ( 2: ) .

Question 20 Let z = cos 20 + i sin 20 and w = cos e + i sine.

(a) Show that 1 + z = 2wRe (w).


(b) Hence , show that

t (:)
k=O
cos (2ke) = 2n cosn cos e ne.
74 Chapter 2: Complex Numbers

0 Challenge Problems

Problem 1 [Dirichlet kernel]

Let z = cos 0 + i sin 0. Define the series

(a) Sh ow t h at S n
_ cis (n + l )0 - l
- . .
ClS 0- 1
(b) Show that
0 sin ( n ! l 0)
Sn = cis ( ~ ) x ( )
sin !

(c) Deduce that

sin Ul)
2
sin ( n+2 l 0)
sin 0 + sin 20 + sin 30 + •••+ sin n0 = . ( )
sm fi2

Problem 2 [Establishing one of t he key steps for the Basel Problem]

Define the polynomial

where n is a positive integer.


(a) Show that
2i sin n0
(cot 0 + it - (cot 0 - it = . ·
Slnn 0

(b) Show that the zeroes of P(x) are x = cot ( k:) where k= 1, 2, · · · , n - 1.

(c) Hence, show that

~ l)1r) =
2 3 2
cot 2 (~) + cot 2 ( :) + cot 2 ( :) + ... + cot 2 ( (n (n - l~(n - ).

(d) Deduce that

MASTERING MATHEMATICS
2G Applications of de Moivre's Theorem 75

Exercise 2G
Applications of de Moivre's Theorem

&&, Fundamentals
Fundamentals 1
Let a , (3 and 1 be the roots of ax 3 + bx 2 + ex+ d = 0. Write the following in terms of a, b, c, d.
(a) a+ /3 + 1 (b) a (3 + /31 +a1 (c) a /31

Fundamentals 2
To express cos( n0) as a polynomial in terms of cos 0, follow the following steps .

(a ) Define z = _ _ _ +i _ __
(b) Simplify zn using _ _ _ _ _ _ _ theorem.

(c) Expand zn manually using P _ _ triangle, or using the b _ _ _ expansion.


(d) Equate the real/imaginary (circle one) components of the two expressions for zn.
(e) Turn all even powers of sin 0 into powers of ____ using the identity _ _ _ _ __

Fundamentals 3
To express tan( n0) as a rational expression in terms of tan 0, follow the following steps .

(a) Obtain expressions for cosn0 and _ _ using the same steps outlined above . Keep the
expressions as they are after equating real/imaginary components and do not modify them.

(b) D ____ the two expressions to obtain tan n0 in terms of powers of cos 0 and sin 0.
(c) Divide the top and bottom by the highest power cosine/ sine (circle one) term.
(d) Simplify and express everything in terms of _ _ __

Question 1 Let z = cos 0 + i sin 0. Prove the following trigonometric identities.


(a) cos 30 = cos 3 0 - 3 sin 2 0 cos 0 (b) sin 30 = 3 sin 0 cos 2 0 - sin 3 0

3tan0 - tan 3 0 3 cot 2 0 - l


(c) tan 30 =------ (d) cot 30 = ------
1 - 3 tan 2 0 cot3 0 - 3 cot 0

Question 2 Let z = cos 0 + i sin 0. Prove the following trigonometric identities.


(a) cos 30 = 4 cos 3 0 - 3 cos 0 (b) sin 30 = 3 sin 0 - 4 sin 3 0

Question 3 Let z = cos 0 + i sin 0. Prove the following trigonometric identities.


(a) cos40 = cos 4 0 - 6cos 2 0sin 2 0 + sin 4 0 (b) sin40 = 4sin0cos 3 0-4sin 3 0cos0

4 tan 0 - 4 tan 3 0 4 cot 3 0 - 4 cot 0


(c) tan 40 = - - - - - -- - (d) cot 4 0 = - -- - - -- -
1 - 6 tan 2 0 + tan 4 0 cot4 0 - 6 cot 2 0 + l
76 Chapter 2: Complex Numbers

Question 4 Let z = cos 0 + i sin 0. Prove the following trigonometric identities.


(a) cos 50 = cos 5 0 - 10 sin 2 0 cos 3 0 + 5 sin 4 0 cos 0
(b) sin 50 = sin 5 0 + 5 sin 0 cos 4 0 - 10 sin 3 0 cos 2 0

Question 5 Let z = cos 0 + i sin 0. Prove the following trigonometric identities.

(a) cos 60 = cos 6 0 - 15 cos 4 0 sin 2 0 + 15 cos 2 0 sin 4 0 - sin 6 0


(b) sin 60 = 6 sin 0 cos 5 0 - 20 sin 3 x cos 3 0 + 6 sin 5 0 cos 0

Question 6 Let z = cos 0 + i sin 0.


(a) Show that zn + z-n = 2 cos n0 for n E z+.
1
(b) Hence, show that cos 5 0 =
16
(cos 50 + 5 cos 30 + 10 cos 0).

(c) Calculate fo'f, cos 5 0 d0.

Question 7

(a) Use a similar technique to the previous question to prove that

1
cos 4 0 =
8 (cos 40 + 4 cos 20 + 3).

(b) By finding a suitable substitution for 0, deduce that

1
sin 4 0 = ( + 3) .
8 cos 40 - 4 cos 20

1
Hence, show that cos 4 0 + sin 4 0 =
4 (cos 40 + 3).
(c)

Question 8

(a) Show that


1
cos 6 0 = (10 + 15 cos 20 + 6 cos 40 + cos 60).
32

(b) Find a similar result for sin 6 0.

Question 9 Let z = cos 0 + i sin 0.


(a) Show that zn + z-n = 2 cos n0. (b) Hence, solve z 4 + 4z 3 + 2z 2 + 4z + 1 = 0.

MASTERING MATHEMATICS
2G Applications of de Moivre's Theorem 77

Question 10 [Guided question for a classic problem]

Define the following cubic polynomial.

P (x) = 8x 3 - 6x - 1
You may assume that cos 30 = 4 cos 3 0 - 3 cos 0.
1
(a) Let x = cos 0. Show that solving P(x) = 0 is equivalent to solving cos 30 = .
2
(b) Solve the trigonometric equation to find 3 distinct values of 0.
(c) Hence, write down the three zeroes of P(x).
(d) Find the exact value of

cos m + cos c;) + c;).cos

(e) Find the exact value of

cos m c;) m c;)


cos + cos cos + cos c;) c;).
cos

(f) Find the exact value of

cos m c;) cos cos c;).

Question 11 You may assume that cos 30 = 4 cos 3 0 - 3 cos 0.


(a) Find the zeroes of P(x) = 8x 3 - 6x + l.
(b) Show that
2 4
cos ( ; ) + cos ( ;) = cos ( %) .

(c) Find the exact value of

cos m c;) cos cos (4;) .

Question 12

(a) Show that cos 40 = 8 cos 4 0 - 8 cos 2 0 + l.


(b) Hence, solve 8x 4 - 8x 2 +1= 0.

(c) Hence, find the exact values of cos ( i) and cos ( 5; ) .


78 Chapter 2: Complex Numbers

3 tan 0 - tan 3 0
Question 13 You may assume that tan 30 =
1 - 3 tan 2 0
(a) Solve the polynomial equation t 3 3t 2 3t + 1 = 0.

c; )-
- -

(b) Find the exact value of tan (;;) and tan

Question 14

(a) Prove that cos 50 = 16 coss 0 - 20 cos 3 0 + 5 cos 0.


(b) Hence, solve the polynomial equation 16x 4 - 20x 2 + 5 = 0.
(c) Hence, show that

(d) Write down the exact value of cos(~~).

Question 15

(a) Prove that sin 50 = 16 sins 0 - 20 sin 3 0 + 5 sin 0.

(b) Show that x = sin ( :0) is a solution to the polynomial equation 16x 5
- 20x 3 + 5x - 1 = 0.

(c) Find the polynomial P(x) such that (x - l)P(x) = 16xs - 20x 3 + 5x - 1.
(d) Find the value of a such that P(x) = (4x 2 + ax - 1) 2 .

(e) Hence, find an exact value for sin (:a).


Question 16

(a) Show that cos 60 = 32 cos 6 0 - 48 cos 4 0 + 18 cos 2 0 - 1.


(b) Hence, find all the roots of the polynomial 32x 6 - 48x 4 + 18x 2 - 1 = 0.
(c) Show that

cos(;;) cos G;) = i


(d) Find the exact value of

cos
2
( ;;) + cos 2 c;) .
(e) Hence, show that

cos ( 127f) + cos (57f)


12 vl6
= 2.

MASTERING MATHEMATICS
2G Applications of de Moivre's Theorem 79

Question 17

(a) Show that


- 15 tan 2 0 + 15 tan 4 0 - tan 6 0
cot 60 = -1 - - - - - -3 - - - - -
6 tan 0 - 20tan 0 + 6tan 0
5

(b) Find the exact value of tan ( ; ) tan ( ~;) .


2

(c) Find the exact value of tan2 ( ;


2
) + tan2 ( ~;) .

(d) Hence, find tan ( ~) +tan c;) .


(e) Write down the equation of the quadratic polynomial that has roots tan ( ; ) and tan ( ~;).
2

(f) Hence , find the exact value of tan ( ; ) and tan(~;)-


2

Question 18

(a) Prove that


tan 5 0 - 10 tan 3 0 + 5 tan 0
tan 50 = - -~ ---~~--.
5 tan 4 0 - 10 tan 2 0 + 1

(b) Hence, find the roots of the polynomial t 4 - 10t2 + 5 = 0.

(c) Show that tan m (2;)


tan = vf5.

(d) Show that tan G) + tan (2;) = J10 + 2v'5


(e) Hence, prove that tan(~) = Js - 2v'5.
2
(f) Find the exact value of tan ( ; ) and justify your answer.
80 Chapter 2: Complex Numbers

0 Challenge Problems

Problem 1 Show that if -1 < r < l , then

1 - r cos 0
1 + r cos 0 + r 2 cos 20
3
+ r cos 30 + · · · = ·
1 - 2rcos 0 + r 2

Problem 2 Let z = cos 0 + i sin 0.


(a) Show that zk + z-k = 2 cos k0, where k E z+.
(b) Let n E z+. Prove the following identity.

2
(2 cos 0) n = 2 'to (2:) cos(2n - 2k)0.

(c) Hence, prove that


{~ 2n
0 d0 = 22n+ 1 (2n)
1r
}O cos n .

Problem 3 Let z = cos 0 + i sin 0.

(a) Simplify ( z +~ r z n.

(b) Show that

(c) Hence, show that

!
7r 1r
cosn 0 cos n0 d0 = -n--1 .
-1r 2

MASTERING MATHEMATICS
2H Roots of Unity 81

Exercise 2H
Roots of Unity

~ Fundamentals

Fundamentals 1
(a) The complex number w is called a root of unity if wn = _ for some n E .Z, n -I 0.
(b) The P - - - - root of unity is the root of unity with the smallest positive a _ _ __

Fundamentals 2
Let w be the principal cube root of unity.

(a) w3 = _ (b) 1+ w + w2 = _

Fundamentals 3
Let w be the principal n th root of unity.

(b) 1+ w + w 2 + •••+ wn-l = _

Fundamentals 4
Follow the following steps to find the n th root of any number a.

(a) Construct the polynomial equation zn = _ .


(b) Convert a to p _ _ form.
(c) Add _ _ to the argument , where k E .Z. The polynomial equation is now

zn = r cis(0 + 2hr).

(d) Raise both sides to the power of _ to make z the subject. The solutions are

for k = 0, ±1, ±2 , ... until n values have been listed. If n is even then you will eventually
need to decide whether to pick the positive or negative value of k. Choose the one that
satisfies the p ____ argument.

(e) List out all n solutions. If a E ~ then the roots will occur in complex c ____ pairs.
82 Chapter 2: Complex Numbers

Question 1 Let w be the principal cube root of unity.

(a) Write down the value of w 3 . (b) Hence show that 1 + w + w 2 = 0.

Question 2 Let P( z ) = z 3 - 1 and let w be the principal root of P( z ) = 0.

(a) Find all the roots of P( z ). (b) Plot them on the Argand diagram.
(c) Show that the other non-real root is w2. (d) Prove that w 2 = w.

Question 3 Let w be the principal cube root of unity. Simplify the following.

(a) (1 + w- 1 )(1 + w- 2 ) (b)


1 1 a + bw + cw 2
(c) (d)
1+w + 1 + w2 b +cw+ aw 2

Question 4 Let w be the principal cube root of unity. Find the equation of the quadratic polynomial
with zeroes (1 + w) and (1 + w2 ).

Question 5 Sketch the roots of the following.

(a) z3 =1 (b) z4 = 1 (c) z5 =1 (d) z6 = 1

Question 6 Find all the roots of the following.

(a) z
3
+1 = 0 (b) z3 + 27 i = 0 (C) z3 - 4 - 4/3i =0

Question 7 Find all the roots of the following.

(a) z4 + 1 = 0 (b) z4 - l 6i =0 (c) z


4
+ -/3 - 1.
-i = 0
2 2

Question 8 Find all the roots of the following.

(a) z5 - 1 =0 (b) z5 + 32i = 0 (c) z 5 - 16y'2 + 16v'2i =0

Question 9 Find all the roots of the following.

(a) z6 - 1 =0 (b) z
6
+ 64i = 0 (c) z6 - 32/3 + 32i =0

Question 10 Let w be the principal n th root of unity.

(a) Prove that wk for k E Z are also roots of unity.


(b) Deduce that 1 + w + w 2 + •••+ wn-l = 0.

MASTERING MATHEMATICS
I
2H Roots of Unity 83

Question 11 [Important exercise to match the roots of unity with t heir conjugate pairs)

Let w be the principal n th root of unity.

(a) Prove that w = wn-l.

(b) Prove that in general wk = wn-k.

(c) Consider the 9 roots of z 9 = 1 and let w be the principal root of unity. Write down all complex
conjugate pairs. For example, the conjugate of w is w 8 .

Question 12 Let w be a non-real root of z 5 = 1.

(a) Show that 1 +w +w 2 + ... +w 4 = 0.

(b) Hence , show that cos (2;) + cos ( 4;) = -~


Question 13 Let w be a non-real root of z 9 = 1.
(a) Show that 1 +w +w 2 + ... +w 8 = 0.

(b) Hence, show that cos (2;) + cos ( 4


;) = cos G)-
Question 14 Let P(z) = z 7 + 1, and let w be the principal root of unity.
(a) Find all the roots of P( z ) = 0.

(b) Hence , show that cos G) + cos (3;) + cos ( 5


; ) = ~.

Question 15 Let w be a seventh root of unity. Find the equation of the quadratic polynomial with
roots w + w2 + w4 and w3 + w5 + w6 .

Question 16 Let w be an n th root of unity. Find the value of

~
~ ( wk 1 ) .
+k
k=O W

Question 17 Show that the roots of (z + 1) 8 - z 8 = 0 are

z = _!
2'
- !2 (1 ± i cot ( br))
8 '

where k = 1, 2, 3.
84 Chapter 2: Complex Numbers

0 Challenge Problems ----------------------------- ---------,


Problem 1 Let w be a cube root of unity. Find all possible values of

and state when they occur.

Problem 2 Let w be an n th root of unity. Show that

(1 + 2w + 3w 2 + 4w 3 + · · · + nwn- 1 )(w - 1) = n.

Problem 3 Find all the roots of

and show that they are purely imaginary.

Problem 4 Let w be the (n + l) th root of unity. Show that

Problem 5 Find the value of L cos


n (
-2br
--) .
k=O 2n + 1

Problem 6 (2k1r) + i sin (2k1r)


Let ak = cos ----:;;:- ----:;;:- for k = l , 2, ... , n.

(a) Show that

(b) Hence, show that

L _____________________________ _______________________ _

MASTERING MATHEMATICS
21 Applications of Roots of Unity 85

Exercise 21
Applications of Roots of Unity

~ Fundamentals

Fundamentals 1
Let a be any complex number.

(z - a)(z - a) = _ __ _ __ _
Fundamentals 2
(a) If n is any positive integer, then

zn - 1 = (z - 1)( - - - - - - - - )-

(b) If n is any positive odd integer, then

Fundamentals 3
(a) If a polynomial P(z) is factorised over Z , it means that the factorisation contains only
coefficients within the set of i _ __

(b) If a polynomial P(z) is factorised over IR, it means that the factorisation contains only
coefficients within the set of r _ _ numbers.

Question 1 Expand the following using the identity


(z - a)(z - a)= z 2 - 2Re (a) z + lal 2 .

(a) (z - 1 + i)(z - 1 - i) (b) (z-3+2i)(z - 3 - 2i)

Question 2 Express the fo llowing as the product of real quadratic factors


(z - a)(z - a)(z - f3)(z - /3).

Question 3 [Useful identities for this topic]

Prove the fo llowing by expanding the right-hand side.


(a) z 3 + 1 = ( z + 1) (z 2 - z + 1) (b) z3 - 1 = ( z - 1) (z 2 + z + 1)
(c) z 5 + 1 = (z + 1) (z 4 - z 3 + z2 - z + l) (d) z5 - 1 = (z - 1) (z 4 + z 3 + z2 + z + 1)
(e) z 6 + 1 = (z 2 + 1) (z 4 - z 2 + 1) (f) z6 - 1 = (z2 - 1) (z 4 + z2 + 1)
86 Chapter 2: Complex Numbers

Question 4 Write down a factorisation of the following over Z.

(a) z
7
+1 (b) z7 - 1 (c) z8 - 1

Question 5 Let P(z) = z 5 - 1.


(a) F ind the roots of z 4 + z 3 + z 2 + z + l = 0.
(b) Hence show that

z
4
+ z 3 + z 2 + z + 1 = ( z2 - 2z cos (2;) + 1) (z 2
-
4
2z cos ( ; ) + 1) .
(c) Show that cos (2;) + cos ( 4;) = - ~-
(d) Show that cos (2;) cos ( 4;) = - ~

(e)
2
Construct a quadratic polynomial with roots cos ( ; ) and cos ( 4;) .
(f)
2
Hence, find the exact values of cos ( ; ) and cos ( 4; )-
(g) Substitute z = 1 into part (b) to prove t hat sin ( ~) sin (2;) = v;.
Question 6 Define the polynomial

Pn(x) = (x + 1)2n+l + xn+2


and let w be a non-real cube root of unity. Prove that x 2 +x+l is a factor of Pn (x ).

Question 7 Let z = cos 0 + i sin 0.


(a) Show that zn + z-n = 2 cos n0 for n E z+.
(b) Show that z 4 +1= (z 2 - J2z + l)(z 2 + J2z + 1).
(c) Deduce that cos20 = 2cos 2 0- l.

Question 8 Let z = cos 0 + i sin 0.


(a) Show that zn + z-n = 2 cos n0 for n E z+.
(b) Show that z 6 +1= (z 2 + l)(z 2 - y/3z + l)(z 2 + y/3z + 1).
(c) Deduce that
cos 30 = 4 cos 0 ( cos 0 - cos i) (cos 0 - cos 5; ) .

MASTERING MATHEMATICS
21 Applications of Roots of Unity 87

Question 9 Let P(z) = z 9 + 1.


(a) Explain why the roots of z 6 - z3 + 1 = 0 are also roots of P(z).
(b) Find the roots of z 6 - z3 + 1 = 0.
(c) Show that

z
6
- z
3
+l = (z
2
- 2zcos G) + 1) ( z 2
- 2zcos (5;) + 1) ( z 2
- 2zcos (7;) + 1).
(d) Deduce that

5 7
2cos30 - 1 = 8cos0 ( cos0 - cos~) (cos0- cos ; ) (cos0 - cos ; ) .

Question 10 Let P(z) = z 9 - 1.

(a) Explain why the roots of z 6 + z3 + 1 = 0 are also roots of P(z).


(b) Find the roots of z 6 + z3 + 1 = 0.
(c) Show that

z
6
+ z 3 + 1= ( z 2 - 2z cos ( ; )
2
+ 1) (z
2
- 2z cos ( 4;) + 1) (z 2
-
8
2z cos ( ; ) + 1) .

(d) Deduce that


. (7r)
sm . (27r) . (47r) y'3
9 sm 9 sm 9 = 8 .

(e) Use a similar technique to prove that

cos (927r) cos (47r) (81r) 1


9 cos 9 = -8.

(f)
2
Hence, find the exact value of tan ( ~) tan ( ; ) tan ( 4;) .
88 Chapter 2: Complex Numbers

0 Challenge Problems

Problem 1 Let w be the principal n th root of unity.


iy

I
I
I

l X

(a) Prove that for any two roots of unity wk and wk-l we have

(b) The n roots of unity form a regular n-gon with some perimeter Pn. Explain why

(c) Hence, prove that as n gets large, Pn --+ 21r.


(d) State the geometric significance of your result.

Problem 2 The diagram below shows an n-sided polygon centred at the origin with vertices
Pi for i = 1, 2, ... , n , where Pi is represented by 1 + Oi.

iy

21r
Let 0 = - , and let w be the principal n th root of unity. Define dk = IPi Pk+ 1 J.
n
(a) Show that d~ = (1 - cos k0) 2 + sin 2 k0.
(b) Show that d~ = 2 - wk - wn-k_

(c) Deduce that dy + d§ + d§ + · · · + d';_ 1 = 2n.

MASTERING MATHEMATICS
2J Solving Polynomials 89

Exercise 2J
Solving Polynomials

&fu Fundamentals
Fundamentals 1
The Conjugate Root Theorem states that if the coefficients of a polynomial are r _ _ , then the
non-real roots will occur in complex c _____ pairs.

Fundamentals 2
Write down the expansion of

(z - a)(z - a) = _ _ _ _ _ __

Fundamentals 3
Let the roots of ax 3 + bx 2 +ex+ d = 0 be a , f3 and ,.
(a) a+ /3 +r = - (b) a /3 + /3, +a,= -

Fundamentals 4
Let the roots of ax 4 + bx 3 + cx 2 + dx + e = 0 b e a , /3 , r and 6.

(a) a+ /3 + , +6=- (b) a ,B + a, + a6 + /3, + /36 + , 6 = -


(c) a /3, + a /36 + a, 6 + /3,6 = - (d) a /3,6 = -

Question 1 [Guided question for solving cubic polynomials given a root]

Consider the polynomial equation


x3 - x2 - 7x + 15 = 0.
It is given that x = 2 + i is a root.

(a) Explain how we know that x = 2 - i is also a root.

(b) Use the sum of roots to find the third root.

(c) Hence fully factorise P(x) = x 3 - x2 - 7x + 15.

Question 2 Use a similar technique to solve the following polynomial equations.


(a) x3 - x2 + 2 = 0, given that x = 1 + i is a root.
(b) x3 - 3x 2 + 4x - 12 = 0, given that x = 2i is a root.
90 Chapter 2: Complex Numbers

Question 3 [Guided question for solving quartic polynomials given a root]

Consider the polynomial


P(x) = x 4 - x 3 - 12x 2 + 26x - 24.

It is given that x = 1 + i is a root of P(x) = 0.

(a) Write down the other root and justify your answer.

(b) Construct a quadratic factor containing the above two roots.

(c) Divide out the quadratic factor from P(x) to obtain another quadratic factor.

(d) Solve the other quadratic factor, and hence write down all the zeroes of P(x).

Question 4 Use a similar technique to solve the following polynomial equations.

(a) x4 - 6x 3 + 18x 2 - 30x + 25 = 0, given that x = 1 - 2i is a root.

(b) x4 - 2x 3 + 6x 2 - 8x +8 = 0, given that x = 2i is a root.

Question 5 Consider the quintic polynomial

P(x) = x 5 - 5x 4 + 12x 3 - 16x 2 + 12x - 4.

(a) Show that x = 1 + i is a double-root.


(b) Hence, find all five zeroes of P( x).

(c) Express P(x) as the product of complex linear factors.

(d) Express P(x) as the product of real linear and quadratic factors.

Question 6 Use polynomial long division to find the quotient and remainder when

P( x ) = x3 - 2x 2 +x+1
is divided by (x - i).

Question 7 [Remainder theorem and complex numbers]

(a) Prove the remainder theorem. That is , when P( x ) is divided by (x - a) the remainder is P(a).

(b) Analyse your proof carefully and observe if at any point your proof relies on a being real.
Determine whether the remainder theorem also works for complex numbers in general.

(c) Find the remainder when P( x) = x3 + x2 - 2x + 4 is divided by (x + i).

MASTERING MATHEMATICS
2J Solving Polynomials 91

Question 8 [Guided question for finding the remainder when dividing by a quadratic]

Consider the polynomial P( x) = x 4 - x 3 + 5x 2 + 2x - 1 and a divisor x 2 + 1.

(a) When P(x) is divided by x 2 + 1, the remainder is ax + b for some a, b E JR. Explain why this is
the case.

(b) Re-write P(x) using the division transformation.


(c) Find P( i) and hence find a and b.
(d) Write down the remainder when P (x) is divided by x 2 + 1.
(e) Hence, find p and q such that P(x) = x 4 - x 3 + 5x 2 + px + q is divisible by x 2 + 1.

Question 9 Find the remainder when the polynomial P(x) = x 5 -4x 3 + 2x + 1 is divided by x 2 + 1.

Question 10 Find the remainder when the polynomial P( x) = x 8 + x 4 - 1 is divided by x 2 + 2x + 2.

Question 11 Find all the zeroes of the polynomial P(x) = x 4 + 4x 3 + llx 2 + 14x + 10 given that
the roots are in the form a ± bi and a ± 2bi.

Question 12 Let the zeroes of P(x) = x 4 + ax 3 + bx 2 +ex+ d be a, /3,, and 5.


(a) Show that a 2 + /3 2 + , 2 + 52 = a 2 - 2b.

(b) Show that zeroes of P( x) = x 4 - x 3 + 6x 2 + 3x + 1 cannot all be real.


(c) Bob claims that the polynomial P(x) = x 4 - 4x 3 + x 2 - 7x +3 has all real roots because
a 2 + /3 2 + , 2 + 52 > 0. Do you agree? Explain your answer.

Question 13 Consider the polynomial

P(x) = x 4 + ax 2 + b
where a, b > 0 and a 2 - 4b 2: 0.

(a) Prove that the zeroes are purely imaginary.


(b) Let two of the imaginary zeroes be ai and /3i. Write down the other two zeroes.

(c) Hence, show that that a 2 + /3 2 = a.

Question 14 Define the general polynomial

where ck ER When P(x) is divided by (x - a), where a is non-real, the remainder is /3.
(a) Prove that when P(x) is divided by (x - a), the remainder is "/3.
(b) What familiar result follows on from this?
92 Chapter 2: Complex Numbers

Question 15 [Palindromic coefficients lead to nice properties]

Let P(x) = x 3 + kx 2 + kx + 1, where k E JR.

(a) Show that x = - 1 is a zero of P(x).


1
(b) Show that if a is a zero of P(x), then - is also a zero of P(x).
a
(c) Show that if a is a non-real zero of P(x), then all the zeroes of P(x) must lie on the unit circle
in the complex plane.

Question 16 Consider the polynomial P(x) = x 3 + px + q where p, q > 0.

(a) Prove that the polynomial cannot have all real zeroes.
(b) Prove that the real root , must be negative.
(c) Let a be any non-real root. Show that Re (a) > 0.
(d) Hence, show that lal > fa.

Question 1 7 Define the polynomial

P(z) = z4 - 2kz 3 + 2k 2 z 2 - 2kz + 1,


where k ER Let a = x + iy and it is given that a and ia are zeroes of P(z), where a-/=- ia.
(a) Explain why a and - ia are zeroes of P(z).
(b) Show that

(c) Hence, show that if P( z) has a real zero, then either

P(z) = (z 2 + l)(z + 1) 2

or
P(z) = (z 2 + l)(z - 1) 2

(d) Show that all the zeroes of P(z) lie on the unit circle.
(e) Show that k = x - y.

(f) Deduce that -\/'2 ::; k ::; \/'2.

MASTERING MATHEMATICS
2J Solving Polynomials 93

0 Challenge Problems

Problem 1 [Cauchy's Bound]

Let a be a root of the complex monic polynomial

Let M be the value of the coefficient with largest magnitude, or in other words

(a) Show that

(b) Deduce that all the zeroes of P(z) satisfy

lal < 1 +M.

Problem 2 [Introduction to Root Bounding Polynomial Techniques]

Define the polynomial

where - 1 ::;; k :=;; 1 and n E z+.


1
(a) Show that if a is a zero of P(z), then - is also a zero of P(z).
a
(b) Suppose that there exists n + 1 distinct zeroes of P(z), none of which lie on the unit circle
in the complex plane. Explain why there must exist at least one zero a outside of the unit
circle.
(c) Show that lal 2nla - kl2 = 11 - kal 2 .
(d) Deduce that

(e) Hence, prove that all the zeroes of P(z) must lie on the unit circle.
94 Chapter 2: Complex Numbers

Exercise 2K
Euler's Formula

&fu Fundamentals
Fundamentals 1
Euler 's identity states that

This is called thee _____ form of a complex number.

Fundamentals 2
Let z = cos 0 + i sin 0.

(a) (b)
2i

Question 1 Convert the following to exponential form.


(a) 1 + iyf3 (b) 2 - 2i (c) -yf3-i

Question 2 Convert the following to Cartesian form.


(a) (b) (c)

Question 3 Show that


ein0 + e-in0 ein0 _ e-in0
(a) ----=cosn0 (b) ----=sinn0
2 2i

Question 4 Express the following in terms of cos n0 and sin n0, for some n E z+.
(a) (ei0_ci0)2 (b) ( ei0 + e - i0)2
(c) ( e2i0 _ e - 2i0 )3 (d) ( e2i0 + e - 2i0 )3

Question 5 Show that


(a) e 2i 0 - 1 = 2i sin 0ei 0 (b) e 2i 0 +1= 2 cos 0ei 0

Question 6 Let z = ei 0 . Simplify the following.


e2i0 _ 1 ei0 - 1
(a) (b)
e2i0 +1 e2i0 +1

MASTERING MATHEMATICS
2K Euler's Formula 95

Question 7

(a) Find an expression for tan 0 in complex exponential form.


(b) Hence, prove that
2tan0
tan20 = 2 0·
1-tan

Question 8 Use the complex exponential form to prove that

1
cos 4 0 + sin 4 0 =
4 (3 + cos40).

Question 9 [De Moivre's Theorem trivially follows from the exponential form]

Let z = r( cos0 + isin0). Show that zn = rn( cosn0 + isinn0).


Question 10

(a) Express cos 0 in complex exponential form.


1
(b) Hence, show that cos 3 0 =
4 (cos 30 + 3 cos 0).

Question 11 [The exponential form can be used to prove the double angle formulae]

Use the complex exponential form to prove that

(a) cos 20 = cos 2 0 - sin 2 0 (b) sin 20 = 2 sin 0 cos 0

Question 12 [The exponential form can be used to prove the compound angle formulae]

Use the complex exponential form to prove that

(a) cos(a + /3 ) = cosacos /3 - sinasin /3 (b) sin( a + /3 ) = sin a cos /3 + cos a sin (3
96 Chapter 2: Complex Numbers

0 Challenge Problems

Problem 1

(a) Show that

cos 0+ cos cp = 2 cos ( -0+cp)


2- cos (0- -2- cp)

. + smcp
sm0 . = 2sm. (0+</J
- - ) cos (0-
- - cp )
2 2

(b) Hence, show that

Problem 2 [Proof of the Dirichlet kernel using Euler's Identity]

Let z = eie. Consider the expression

(a) Show that


ei(n+l)0 _ 1
Sn = - -_e_ __
e 2
- 1

(b) Show that


· n0
2
sin (n+l2 0)
Sn= e 2 X . ( )
sm ~2

(c) Deduce that

sin (!3:.0)
2
sin ( n+2 1 0)
sin 0 + sin 20 + sin 30 + •• • + sin n0 = . ( )
sm ~2

MASTERING MATHEMATICS
Chapter 2 Review 97

Chapter 2 Review
Complex Numbers

0 Review

Question 1 Let z = 10 - 5i and w = 3 - 4i. Find


z
(a) zw (b)
w

z
Question 2 Prove that if - - . is purely real, then z is purely imaginary.
z-i

Question 3 Let
z +l = ki
z- l '
where k E JR. Show that lzl = 1.

Question 4 Find J - 24 + 70i.

Question 5 Solve the following quadratic equations.

(a) z 2 - 6z+13 = 0 (b) z2 - (7 - i)z + (14 - 5i) = 0

Question 6 Convert the following to Cartesian form.


2
(a) 2 cis ( - ; )

Question 7 Convert the following to polar and exponential form.

(a) J3 - 3i (b) -2 - 2i

Question 8 Simplify the following.

cos 0 - i sin 0 1 + e2i0


(a) (b)
cos 0 + i sin 0 1 _ e2i0
98 Chapter 2: Complex Numbers

Question 9 Let A and B be points representing 3i and 2 - i respectively.

Suppose AB is one side of a square ABC D.


(a) Draw the two possible scenarios. (b) Find C and D for each scenario.

Question 10 The diagram below shows vertices of a triangle ABC represented by z 1 , z2 and
z3 respectively.

iy
B

The triangle is isosceles and right-angled at the point B.

(a) Show that (z1 - z2 ) 2 + (z2 - z3 ) 2 = 0.


(b) The point D is chosen so that ABC D is a square. Find the complex number z4 that
represents D in terms of z1, z2 and z3.

(c) Hence, show that if z2 #- 0, then

MASTERING MATHEMATICS
Chapter 2 Review 99

Question 11 Let A and C be points representing 2 + 2i and 6 + 4i respectively.

iy

C

A

Given AC is a diagonal of a square ABCD, find the complex numbers representing Band D.

2
Question 12 Let z = i-/2 and w = --.
1-i

iy

z
w

(a) Express z and w in polar form.

(b) On the same Argand diagram, plot z, w and z + w.


31r
(c) Show that arg(z + w) = .
8
3
(d) Hence, find the exact value of tan ( ;).

Question 13 Find the Cartesian equation of the following loci.

(a) Im ( z - l + 3i) = 4 (b) Im(z) =l z l (c) zz :s; 2(z + z)


(d) z + z = Im ( z) (e) z2 - z2 = 8i (f) z2 + z2 = 4
100 Chapter 2: Complex Numbers

Question 14 On an Argand diagram , shade the region where the fo llowing conditions are both
satisfied.
7f 7f
0 ::; Re (z ) ::; 2 and Iz - l + i I ::; 2
4 ::; arg z ::; 4
(a) (b) lz l ::; lz - 21 and -

(c) Iz - z I < 2 and Iz - l I 2: 1 (d) Iz - 2 + i I ::; 2 and Im (z ) 2: 0


7f 7f 7f
(e) lz - 11 ::; 2 and - ::; a rg(z - 1) ::; (f) Iz - l - i I < 2 and 0 < arg (z - l - i) <
4 4 4
Question 15 Find the locus of z if l2z + 11 = lz + 21.

Question 16 Consider the locus of z defined by Iz - 2 - 2i I = 1.

(a) Find the maximum value of lzl. (b) Find the minimum value of lzl.
(c) Find the maximum value of arg(z). (d) Find the minimum value of arg(z).

Question 17 Find the integer values of n for which ( -vl3 + i r is purely real.

1
Question 18 Show that sin 5 0 = - (sin 50 - 5 sin 30 + 10 sin 0).
16

Question 19 Let z = cos 0 + i sin 0.


(a) Show that zn + z-n = 2 cos n0. (b) Hence, solve z 4 - 4z 3 + 2z 2 - 4z + l = 0.
Question 20

(a) Show that cos 40 = 8 cos 4 0 - 8 cos 2 0 + l.


(b) Hence, solve 16x 4 - 16x 2 + 1 = 0.
(c) Hence, find the .exact values of cos ( ; ) and cos(~;).
2

Question 21

(a) Prove that cos 30 = 4 cos 3 0 - 3 cos 0.

(b) Hence find the roots of 8x 3 - 6x +l = 0.

(c) Deduce that sin(~)+ sin G;) =sin(:;)•


Question 22 Find the roots of
(a) z5 + l =0

MASTERING MATHEMATICS
Chapter 2 Review 101

2 7ri
Question 23 Let w = eT.
6
(a) Show that 1 + w + w 2 + w 3 + w 4 + w 5 + w = 0.
(b) Hence , show that

(727r) +cos (47r) (67r) = - 21.


cos
7 +cos 7

Question 24 Let w be a non-trivial cube root of unity. Simplify the following.

Question 25

(a) Show that


z6 - l = (z 2 - 1) (z 2 - z + l) (z 2 + z + l).

(b) Deduce that


sin 30 = sin 0 (2 cos 0 + l )( 2 cos 0 - l) .

Question 26 Let P(z) = z 5 + l.

(a) Express P(z) as the product of real linear and quadratic factors.

(b) Factorise P (z) over Z.

(c) Show that cos G) + cos (3;) = t


(d) Show that cos G) cos (3;) = -~-

(e) Write down the equation of the quadratic polynomial with roots cos GD 3
and cos ( ;)-

3
(f) Hence, find the exact value cos(~) and cos ( ;).

Question 27 Find the roots of the following polynomial equations.

(a) x3 - 4x 2 + l4x - 20 = 0, given that x = l - 3i.

(b) x4 - 6x 3 + 15x 2 - l8x + 10 = 0, given that x = 2 - i.

Question 28 Find the remainder when P(x) = x 5 - 6x 4 + x 3 + 2x - 1 is divided by (x 2 + 1).


102 Chapter 2: Complex Numbers

Question 29 Find all the zeroes of P(x) = x 3 - 8x 2 + 29x - 52.


(a) Show that P(x) cannot have all real zeroes.
(b) Hence, find the zeroes of P ( x) given that two of the roots are in the form a + bi and 2a.

Question 30 Define the polynomial P(x) = x4 - 4x 3 + llx 2 - 14x + 10.


(a) Find all the zeroes of P(x) given that two of the roots are in the form a + bi and a+ 2bi.
(b) Hence, express P(x) as the product of real quadratic factors.

Question 31 Express the following in terms of cos n0.

(a) (eie + e-i0) 3 (b) (e2i0 _ c2i0) 4

Question 32

(a) Express sin 0 in complex exponential form.


1
(b) Hence, show that sin 3 0 =
4 (3 sin x - sin 3x).

MASTERING MATHEMATICS
Chapter 2: Investigation Task 103

q Investigation Task

Failure of de Moivre's Theorem

In this chapter, we learn de Moivre's theorem and use it for integer powers. For example, we
learn how to calculate

or even something like

7f Jf)-3
(
cos
3 + i sin 3
Without much further thought, it is tempting to do the following.
1
21r 21r) 2
( cos 3 + i sin 3 = cos
1r
3
+ i sin 31r

However , this is incorrect. This investigation task allows the student to explore this further.

Question 1 Explain why de Moivre's theorem does not work for non-integer powers and what
happens if you 'try ' to do it anyway. What is the significance of the answer you initially obtain?

Question 2 Give a few examples that demonstrate failure of de Moivre 's theorem for non-
integer powers.

Question 3 The fact that de Moivre's theorem does not work for non-integer powers is not
necessarily a bad thing, and in actual fact it is used for a particular topic in Complex Numbers.
What is this topic, and why is it that we actually want de Moivre's theorem to not work?
104 Chapter 2: Complex Numbers

q Investigation Task

Complex Exponentiation

We know what it means to raise a real number to an integer power. We know what it means
to raise a real number to a rational power. But what does it mean to raise a real number to
anything beyond this? We can type things like 2vt2 in our calculators and get answers, but upon
deeper thought it 's not entirely clear what it means to raise 2 to the power of something like v/2,
and even less so for the case of something like ei1r. Raising e to the power of i1r at first thought
makes just about as much mathematical sense as ebanana.

Write an article that de-mystifies everything and explains intuitively what it means to raise a
number to a complex number. Your answer should include a discussion of

• What does it mean to raise a real number to an irrational power?

• What does it mean to raise a real number to a complex power?

• Why is a number like ii not well-defined?

MASTERING MATHEMATICS
Chapter 2: Investigation Task 105

q Investigation Task

Fundamental Theorem of Algebra

When we study polynomials, we use the fact that if P(a) > 0 and P(b) < 0, then there is a real
root a < a < b. In other words, we have some way of 'guaranteeing' that there is a real root
somewhere.

We also claim that if P(x) is of degree n, then P(x) = 0 has n roots. This allows us to make
conclusions like P(x) = x 2 + 1 has two zeroes, even though we physically cannot see two x-
intercepts. But upon further thought, it 's not obvious at all that all polynomials of degree n
MUST have a root, let alone n of them. For example, consider the following polynomial equation.

P( x) = (37 - 29i)x 2020 + (-17 + 13i)x 2019 + ...


Is it really 'obvious' that P(x) has 2020 values that we can plug in to get zero? This is the power
of the Fundamental Theorem of Algebra, and this investigation task allows the student to explore
this and appreciate that this theorem truly has earned the title of being a fundamental theorem.

Question 1 The idea that if P(a) > 0 and P(b) < 0 then there is a real root a < a < b is
actually a theorem. What is the name of this theorem and what broader theorem is it actually a
consequence of?

Question 2 Give the actual statement of the Fundamental Theorem of Algebra, and explain
how this leads to the statement that a polynomial of degree n has n complex roots, including
repeated roots.

Question 3 The full proof of the Fundamental Theorem of Algebra is not accessible to the
majority of Extension 2 students. However, a rough outline of the proof and the intuition behind
it is accessible. Produce a five to ten minute presentation that gives an outline of the proof of
the theorem. Your presentation should address the following key questions.

• What is the brief history behind the theorem?

• What assumptions are made at certain steps?

• What was the intuition behind certain steps?


106 Chapter 2: Complex Numbers

q Investigation Task

Polynomial Root Bounds

Towards the end of Exercise 2J , there was a small collection of problems that explore the idea of
polynomial root bounds. The theory behind this is rich and combines many aspects of polynomial
theory and complex numbers. It also has many real-life applications in an assortment of fields of
mathematics. This investigation task allows the student to get some hands-on experience with
this theory and to allow them to explore how useful such theorems can be!

Question 1 Research and find two well-known root bounds that confine the roots of P(x) = 0
to a certain region. Your response should include

• any restrictions on the coefficients of the polynomial.

• an example of a polynomial and a visual indicator of the region of where the roots should
reside within.

Question 2 Polynomials such as

P(z) = 9z 4 + 6z 3 + 5z 2 + 2z + 1
have strictly decreasing positive coefficients. Such polynomials have their zeroes satisfying a
particularly nice condition that makes them easier to find.

(a) What is that 'nice' condition?


(b) Prove it.

Question 3 Produce a five minute presentation on the Enestrom-Kakeya Theorem. Your pre-
sentation should include

• a statement of the theorem.

• any conditions on the coefficients.

• an example of a polynomial, and what the Enestrom-Kakeya Theorem states about where
the zeroes lie.

• any real-life applications or benefits of root-bounding theory in general.

MASTERING MATHEMATICS
108 Chapter 3: 30 Vectors

Exercise 3A
Introduction to 3D Vectors

&fu Fundamentals
Fundamentals 1
Let '!J =ad+ a2j__ + a3ls and '!J = bii + b2j__ + b3ls. Write an expression for the following.

Fundamentals 2
Write down t he formula for the following.

(a) scalu v (b) pro ju v

Question 1 Let P be the point (3 , -7, 4). Find the distance of P from the following.

(a) xy- plane (b) yz-plane (c) xz-plane


(d) x-axis (e) y-axis (f) z-axis

Question 2 Let g = [~] and Q = [ ; i]. Calculate the following.

Question 3 Calculate the angle between the following vectors, correct to the nearest degree.

(b) = - 2i + 5 j__ + 3 ls
'!J (c) 'fJ=3i - 5j-2k
Q = 4,i + ls r=-3j+3k

Question 4 Let '!J = i + j__ and Q = i + ls- Find a unit vector that is perpendicular to '!J and Q.

Question 5 Let '!J = i + 3j__ + 2ls, Q = Oi- 2j__ + ls and 'I}} = 2,i + j__ + 2ls. Find constants a, b, c
such that

MASTERING MATHEMATICS
3A Introduction to 3D Vectors 109

Question 6 Calculate, correct to the nearest degree, the three angles in the triangle defined by
P(l, -3, 4), Q(-2, 0, 6) and R(2 , 3, -1).

Question 7 Define A(2, 0, -3) , B(-1, 4, 2) and C(0, 6, -3). Find all possible points D such that
ABCD forms a parallelogram.

Question 8 Let u = 3i + 1 + 2]5 and v = i - 2j + 3]5. Find the following.

(a) projv u (b) proju v (c) scalv u (d) scalu v

Question 9 What can we say about two vectors u and v if


(a) proju v = projv u? (b) scalu v = scalv u?

Question 10 Define A(2, 0, -3), B(-1, 4, 2) and C(0, 6, -3). Let u = .BA and v = BC.
(a) Find scalv u.
(b) Find iul.
(c) Hence , find the perpendicular distance of A from BC.
(d) Find the area of D,ABC.
(e) Find the area of the same triangle but using an alternative method.

Question 11 Let 'Y: = 3i - 4j + ls and Q = -i + 2j - ]5. Find a vector with length 12 that is
perpendicular to both 'Y: and Q.

Question 12 Let Q, Q and £ be vectors in 3D space. Prove the following identity

Question 13 Consider the triangle formed by A(l, -3 , -2) , B(5 , -1 , 2) and C(-1 , 1, 2).
(a) Prove that the triangle is isosceles.
(b) Show that it is not a right-angled isosceles triangle.

Question 14 Simplify the following by using the fact that Q ·Q = lgl2 .


(a) IY: + Ql 2 (b) IY: - Ql2
(c) IY: + Ql 2 + IY: - Ql2 (d) IY: + Ql2 - IY: - Ql2

Question 15 [Deducing a well-known inequality from the Cauchy-Schwarz inequality]

(a) Prove that 'Y: · Q ~ IY:I IQI •


(b) Hence, prove that IY: + QI ~ IY:I + IQI•
(c) State the significance of this result.
110 Chapter 3: 3D Vectors

Question 16 [Using algebraic symmetry to deduce similar results]

Let P(a , b, c) be a point that forms a rectangular prism with the origin.

C ------- -/J
/ / I
/ / I
/
/
/ /
/

( -- --- --•(P I
I

IB y
Q ,__.....__ _ _~ - ----

/
/
I I /

A ____ ____ _, /
I I /

Let A , B and C be the points on the x, y and z axes representing their respective components of P.

(a) Find the size of L_OAP.


(b) W hat can we deduce about L_OBP and L_OCP as well?
(c) Find an expression for L_AP B and deduce similar expressions for L_BPC and L_APC.

0 Challenge Problems

Problem 1 [The vector proj ection is a linear operator]

Let Pw (u) be t he vector projection of u onto w. Let v be any vector and k E IR. Show that

Problem 2 Find the area of L:,OAB for points A(x1 , Y1 , z1) and B( x 2, Y2 , z2).

Problem 3 [Cross product]


Find an expression for a vector that is perpendicular to both Yo and 12, if

Yo = a ii + a2 j + a3 ls
12, = bi,i + b2 j + b3 ls

Problem 4 [Normal vector]

Show t hat the vector !2 = a_i + b j + c ls is perpendicular to t he plane ax + by + cz + d = 0.

MASTERING MATHEMATICS
3B Proofs using 3D Vectors 111

Exercise 38
Proofs using 3D Vectors

&fb Fundamentals
Fundamentals 1
Describe the main steps required to do the following.
(a) Prove that two vectors are perpendicular.
(b) Prove that two vectors are parallel.
(c) Prove that three points are collinear.
(d) F ind the angle between two vectors.
(e) Show that two intervals bisect each other.

Question 1 [Proving the distance formula]

Define the point P(a , b, c) as shown below.

I
I

\ P(a, b, c)
I

b y

I
I I
I I
I I / /
\I /

Show that the distance of P from the origin is


112 Chapter 3: 3D Vectors

Question 2 The diagram below shows vectors']!;, 'Q and w in a rectangular prism produced by the
origin and P(a, b, c).

7r
Show that if the angles between ']!;, 'Q and w are all equal to , then the rectangular prism is a cube.
3

Question 3 [Finding the bond angle of methane]

A molecule of methane CH4 is in the shape of a regular tetrahedron with a carbon atom at the
centroid and four hydrogen atoms on each vertex of the tetrahedron.

The bond angle is the angle formed by any H - C - H chain.

(a) Suppose that three of the hydrogen atoms are located at (1, 0, 0) , (0 , 1, 0) and (0 , 0, 1). What
are the coordinates of the fourth hydrogen atom, if it is to be in the first octant?

(b) What are the coordinates of the carbon atom?

(c) Hence, show that the bond angle of methane is approximately 109.5°.

MASTERING MATHEMATICS
3B Proofs using 3D Vectors 113

Question 4 The diagram below shows a square pyramid where the side faces are equilateral triangles.

Let the coordinates of the square base be (a , a, 0) , (a, -a , 0) , (-a, a, 0) and (-a, -a , 0).

(a) Find the coordinates of the tip of the vertex in terms of a.


(b) Find the angle of inclination between any of the side faces and the square base.

Question 5 Consider the tetrahedron defined by A(t, 0, 0) , B(0, t , 0) and C(0, 0, t).

1
Show that the angle between the plane ABC and the base is 0 = cos- ( ~).
114 Chapter 3: 3D Vectors

Question 6 [Proving the dot product for 3D vectors]

The diagram below shows two arbitrary vectors Y, = uii + u2 j + u3 ls and '.Q = vii + v2 j + v3 ls and
an angle 0 between them. ~ ~

(a) Write down two different expressions for the distance between the t ips of Y, and '.Q -

(b) Deduce that 1!, · '.Q = u1 v1 + u2v2 + u3v3.

Question 7 Let Y, be a unit vector in the xy-plane, and let '.Q be a unit vector in the yz-plane. Let
a be the angle between Y, and i, and similarly let /3 be the angle between '.Q and j.

Let 0 be the angle between 1!o and '.Q. Prove that

cos 0 = ± sin a sin (3.

MASTERING MATHEMATICS
3B Proofs using 3D Vectors 115

Question 8 [Three-dimensional analogy of Pythagoras ' Theorem]

Consider the tetrahedron defined by A(a, 0, 0), B(0 , b, 0) and C(O, 0, c).

z
C

Show that if IDABCI represents the area of DABC, then

Question 9 [Three-dimensional analogy of the median property of triangles]

The diagram below shows a tetrahedron OPQR where p , q and r., are the position vectors of P , Q
and R respectively. Let A , B , C , D , E and F be the midpoints of OR, PQ , OP , QR, OQ , and PR
respectively.

Prove that AB , CD and EF are concurrent and bisect each other.


116 Chapter 3: 3D Vectors

Question 10 The diagram below shows a tetrahedron PQRS with vertices P(0, 0,p), Q(q , 0, 0),
R(0, r, 0) and S(0, 0, 0). Let p_ be the vector from P perpendicular to the face opposite point P with
length being the area of that same face.

z
p

Define vectors Q, r and §, similarly.

(b) Find similar expressions for Q, r and §,.

(c) Hence, show that p_ + Q + r + §, = 0.

Question 11 The diagram below shows two main diagonals 'J.J; and r in a rectangular prism produced
by the origin and the point P(a , b, c).

(a) Find expressions for 'J.J; and r in terms of a, b and c.

(b) Show that 'J.J; and r are perpendicular if and only if a 2 + b2 = c2 .


(c) Find a possible point P that will have two perpendicular main diagonals.

MASTERING MATHEMATICS
3B Proofs using 3D Vectors 117

Question 12 [Equation of a plane]

A plane in 3D space can be defined by a point on the plane and a vector perpendicular to the plane
called the normal vector. This works similarly to how a line in 2D space can be defined by a point on
the line and a gradient.

The diagram above shows two vectors rand ro representing the position vectors of points P( x, y , z )

and Po(xo, Yo, zo) on a plane. Let rr = [~] be the vector perpendicular to the plane.

(a) Explain why


(r - ro) · n = o.

(b) Hence, show that the equation of the plane is

ax + by + cz = axo + byo + czo.

(c) Find the equation of the plane that passes through (2 , -3, 1) and is perpendicular to the vector
-3i + 2j - Js.
(d) What is the shortest possible distance between any point on the plane ax + by + cz + d = 0 and
the origin?
118 Chapter 3: 3D Vectors

Question 13 [Direction cosine]

The direction cosines of a vector ~ = ai + b1+ c J5 are the cosines of the angles from the vector to each
of the coordinate axes. Let a, /3 and ry be the angles of vector~ from the x, y and z axes respectively.

I /

' I
X - -- - ', 1,'
- - - ,_I,,

a
(a) Show that cos a = l~I·
(b) Deduce that
cos 2 a+ cos 2 /3 + cos 2 ry = 1.

Question 14 [Connection between direction cosines and spherical coordinates]

Let 0 and </> be the angles shown in the diagram below. Let the direction cosines be defined as in the
previous question.

Show that the direction cosines of the vector ~ can be expressed as the following.

cos a = cos </> cos 0


cos /3 = cos </> sin 0
cosry =sin</>

MASTERING MATHEMATICS
3B Proofs using 3D Vectors 119

0 Challenge Problems

Problem 1 [Special case of a regular polytopic distance]

Let A, P2, P3 and P4 be the vertices of a regular tetrahedron circumscribed by the unit sphere.
Let the position vectors of the vertices be P,1, P,2, 'f!,3 and 'f!,4 respectively.

(a) Show that P,1 + P,2 + P,3 + P,4 = 0.


1
(b) Prove that 'f!,i · 'f!,j =- for all i #- j.
3
(c) Let P be any arbitrary point on the sphere. Show that the expression

is independent of the position of P.

Problem 2 Show that two non-zero vectors 1!1 and 1::2 are perpendicular if and only if their
direction cosines satisfy

Problem 3 Let ?J; and 1! be any two non-zero vectors in the 3D plane. Define the following
vector.
1Y = ?J; - tr ,
where t ER
· U·V
(a) Show that the value oft that minimises l1YI is t = lrl;--.
(b) Interpret your result geometrically.

Problem 4 Let 1:: 1, 1::2 and 1!3 be three mutually perpendicular vectors in the 3D plane. Define
the following vector.

Show that the scalars c1, c2 and c3 are given by

for i = 1, 2, 3.
120 Chapter 3: 3D Vectors

Exercise 3C
Vector Equation of a Line

&b Fundamentals
Fundamentals 1
The vector equation of the line £ that passes through ro in the direction of 'Q is

y
X

(a) The letter A is called a p _ __


(b) It is called a P- - - because it controls the position of a variable point P along the
line. Each point P along the line has its own unique value of A.

(c) It is important to note that r,(,\) is the P- - - vector of the set of points along a line
£, but it is not the line £ itself.

Fundamentals 2
The line segment from g pointing towards f2 is represented by the vector equation

y
X

(a) The domain of A is A E [ - , _].


(b) When A= 0, then r(,\) points towards g / Q (circle one).
(c) When A = 1, then r (A) points towards g / f2 (circle one).

MASTERING MATHEMATICS
3C Vector Equation of a Line 121

Question 1 Find the vector equation of the line that passes through the point P in the direction
of '.Q for the following scenarios.

(a) P(l, 2, 3) and '.Q = 4i + 5j + 6,b; (b) P( - 3, 1, 5) and 1! = - 2i + 4j + k


(c) P(2 , 1, - 3) and '.Q = 4i - 3,b; (d) P(3, - 5, 2) and '.Q = 2j + 7,b;

Question 2

(a) Draw on the 2D plane the line £ represented by the following vector equation.

(b) P lot on £ the two points corresponding to ,\ = 1 and ,\ = 2.

Question 3

(a) What is the gradient of the line with direction vector [~] ?

(b) Find a possible vector equation of 2x + 3y - 6 = 0.

Question 4 Find the vector equation for the line AB if A and B are defined as such.

(a) A( - 7, 3, 4) and B(2, 6, - 1) (b) A(-4, 1, 5) and B(6, -2, 3)

Question 5 Define points A(3 , - 2, 4) and B(l , 0, 2). Find the vector equation for the
(a) interval AB. (b) interval BA.

Question 6 Determine if the following points are collinear.

(a) P(0 , - 5, 5), Q(l , - 2, 4), R(3, 4, 2) (b) P(2, 4, 2), Q(3, 7, -2), R(l, 3, 3)

Question 7 Determine whether the following points lie on the line represented by

(a) (-3 , -2, 12) (b) (-8, 8, -9) (c) (-4, 0, 8)


122 Chapter 3: 3D Vectors

Question 8 Let f be the line with vector equation

(a) Find the value of,\ that corresponds to the point P where f intersects the xy-axis.

(b) Hence, find the coordinates of P.

(c) Similarly, find the coordinates of the point Q where f intersects the plane y = 2.

Question 9 Consider the line P through P(l, - 2, 3) and parallel to 2i, + 3 j__ - "Js:.

(a) Find two points that lie on f. (b) Determine if the point R(0, 1, -2) lies on f.

(c) Determine if the point R(7, 7, 0) lies on f. (d) Find where f intersects the yz-plane.

Question 10 Find the vector equation of the line defined by the parametric equation

x = 3 + 2,\ , y = l - ,\ , z = - 2 + ,\

where ,\ E JR.

Question 11 Determine whether the lines represented by the vector equations

~(,\) = (i - 2j__ + 3k) + ,\(2i + k)


7:1(µ) = (3i+ j__) +µ(-2i+ j__+3k)

are parallel, intersect, or skew.

Question 12

(a) Find the vector equation of the line P that passes through P(-3 , 6, 1) and Q(l , 0, 3).

(b) Hence, find where f intersects the plane x +y - z + 2 = 0.

Question 13 Find the point of intersection of the lines represented by the following vector equations.

MASTERING MATHEMATICS
3C Vector Equation of a Line 123

Question 14 Consider a unit cube with all of the vertices in the first octant.

(a ) Find the parametric vector equation of any two main diagonals.


(b) Hence , find the point of intersection of the main diagonals and verify that it is indeed halfway
through the main diagonal.

Question 15 Let f, be the line represented by the vector equation

F ind the perpendicular distance between the point P (2, 1, 3) and e.

Question 16 Consider the two lines £1 and £2 represented by the vector equations

~ (A) = [ 13]+ ~A[ ~]

~ (µ) = [ i2] l 2]
+µ [

(a ) Explain why £1 and £2 are parallel.


(b) Find the distance between e1 and f 2.

Question 17 Let £1 be the line through P(l , - 2, 3) that is perpendicular to £2 represented by

Find a vector representation of £2 if and intersects f 1.


124 Chapter 3: 3D Vectors

0 Challenge Problems

Problem 1 [Symmetric equations]

Consider the vector equation

(a) Show that


x - x0 y - Yo z - z0
a b c

(b) Interpret this result geometrically.

Problem 2 Consider two skew lines £1 and £2 defined by the vector equations

~(>.) = (i+2ls) +>.(-2j + ls )


'C](µ) = ( - 2 + ls) + µ(i + j)

(a) Find an expression for the vector that connects a point Pon £1 to a point Q on £2. Express
your answer in terms ofµ and >..
(b) Find the shortest distance between £1 and £2.

Problem 3 Let p be the position vector of a point Pon a sphere S with radius r and centred
at C represented by £· You may use the fact that the position vector y_ for the set of all points
on Sis

(a) Consider a line ,e that passes through P. If the direction vector of ,e is Q, write down a
vector representation of £.
2Q. (£ - p)
(b) Show that if ,e intersects S, then >. = 0 or >. = IQl 2 - ·

(c) Deduce that ,e is tangential to S if and only if

MASTERING MATHEMATICS
3D Parameterising 3D Curves 125

Exercise 3D
Parameterising 3D Curves

&fu Fundamentals
Fundamentals 1
Similarly to straight lines , curves in 3D space are defined P - - -- using

X = f (t)
y = g(t)
z = h(t)
It is often difficult to draw the full curve and demonstrate all features properly by hand. So
instead what we can do is draw the projections of t he curve onto the __ , _ _ and __ -planes
to gain some insight as to what t he curve may look like from different P - - - - --

Fundamentals 2
(a) The curve may have some restriction on the P - - - - · When this happens, it may
restrict the curve so only a s _ _ _ of it is drawn.

(b) Be wary t hat the curve may also be restricted by other means. For example

X = t2
1
y = t2

is not simply the curve y = 1/ x because x and y are always P - - - and so it only
represents the first q _ _ _ of the curve.

Question 1 Find the Cartesian equation of the following , and state any restrictions where necessary.

(a) r(t) = (t - l)i + (2t + 3)j (b) r (t) = (2t - 1) i + (t 2 )j


(c) r(t) = t 2 i + (t 2 + l)j (d) r( t) = 4 cos (t) i + 3 sin (t )j
(e) r (t) = 3 sec (t) i + 2 tan (t) j (f) r( t) = (et)i + (e-tu

Question 2 Draw the curve defined by

r (t) = (t + l)i + (2t - l)j + (t 2 ) k

in the xy, xz and yz-planes fort E [O , 3].


126 Chapter 3: 30 Vectors

Question 3

(a) Find the Cartesian equation of the curve defined parametrically by

r (t) = (cos t + sin t) i + (cos t - sin t) j.

(b) Find the Cartesian equation of the curve defined parametrically by

r(t) = v'2 cos(t)i + v'2 sin(t) j.

(c) Describe the difference in their position vectors as t varies.

Question 4 [Parametrisation of the ellipse]

Consider the ellipse defined parametrically by

r (t) = a cos (t) i + b sin (t) j


where t E [O, 21r].
(a) Find the Cartesian equation of the ellipse.
(b) Sketch the ellipse and indicate the orientation of the ellipse as t increases.
(c) How would you adjust the parametrisation such that the ellipse is the same, but the orientation
is reversed?

(d) How would you adjust the parametrisation so that the starting point is instead on the positive
y-axis?

Question 5 [Witch of Agnesi]

Consider the curve defined parametrically by

r(t) = 2a tan(t)i + 2a cos 2 (t) j.

(a) Find the Cartesian equation. (b) Sketch the curve.

Question 6 [Parametrisations are not unique! ]

Consider the Cartesian equation y = x 3 and three possible parametrisations below.

7) ( t)= (2t)i + (8t 3 )j


r:](t) = (t 2 )i + (t 6 )j

[3(t) = mi+ c~) 1


What is the difference between all three paths despite them having the same Cartesian equation?

MASTERING MATHEMATICS
3D Parameterising 3D Curves 127

Question 7 [Spiral behaviour]

All of the following parametrisations below represent spirals, but they differ in some way. Describe
the behaviour of each spiral as t increases, given that t ~ 0.

(a) r(t) = cos(t)i + sin(t) j + (t) !5 (b) r(t) = cos(t)i + sin(t)j + (-t)Js
(c) r (t) = cos (t) i + sin (t) j + (t 2 ) }5 (d) r (t) = cos (t) i + sin (t) j + ln (t) }5
(e) r(t) = cos(t)i + sin(t) j + e-t }5 (f) r (t) = cos (t) i + sin (t) j + (1 + sin (t)) }5

Question 8 Consider the parametrisation

r (t) = cos (t) i + sin (t) j + (1 - sin (t)) }5.

(a) State what the curve looks like in the xy-plane.


(b) State what the curve looks like in the yz-plane.
(c) State what the curve looks like in the xz-plane.
(d) Describe geometrically what the parametrisation represents.

Question 9 Describe the shape of the curve defined by the following vector equations.
(a) r (t) = (1 - t) i + (t + 2 )j + (2t + 3) J5 (b) r(t) = (t)i + (t 2 + 2)1, + (t)Js
(c) r (t) = cos (t) i + sin (t) j + (t) !5 (d) r (t) = cos (t) i + cos (t) 1 + sin (t) }5

Question 10 [Twisted Cubic]

The diagram below shows a twisted cubic for t E [- 2, 2]

5
-10 -

and the parametrisation of the twisted cubic is

(a) Find the equation of the curve along the xy, xz and y z-axes.
(b) Sketch the curve along the xy, xz and yz-axes .
128 Chapter 3: 3D Vectors

Question 11 [Parametrisations and motion]


Three different particles move in the xy-plane according to the trajectories defined parametrically by

?} (t) = cos(t)i + sin(t) j


T_J ( t) = cos (2t) i + sin (2t )j
D (t) = sin (t) i + cos (t) j
What is the difference in their motions?

Question 12 Two particles travel according to the trajectories defined by

?} ( t)= (2t - 1) i + (t 2 )j_ + (t + l) _h;


r,J(t) = (t 2 )i + (3t - 2)j + (3 - t) k

Determine if the particles will ever collide.

Question 13 A particle moves according to the path defined parametrically by

r(t) = (2t 3 + t)i + (4 - t 2 )j + (3t + l)k


where t is time in seconds.

(a) Write down an expression for the velocity vector r(t) and acceleration vector g(t).
(b) Find the speed and acceleration of the particle after one second.

Question 14 Find a possible vector representation, with conditions if necessary, of the curve that
satisfies each of the following descriptions. Note that there are multiple possible answers.

(a) A spiral section of radius 2 that wraps around the y-axis and begins at (0 , 0, 2).
(b) A straight line that passes through (3, -6, 1) and (-2, 0, 4).
(c) A curve that is the parabola y = x 2 in the xy-plane but the linear function z 2y in the
yz-plane.

(d) An ellipse that in the xy-plane is the unit circle, but in the yz-plane forms the linear function
z = 2 -y.

MASTERING MATHEMATICS
3D Parameterising 3D Curves 129

Question 15 The diagrams below show sketches of eight parametrically defined curves.

(i) (ii)
z

(iii) (iv)
z

(v) (vi)

(vii) (viii)
z z

Match the curves to the appropriate set of parametrisations below.

(a) r.,(t) = (cost)i + (sint)j + (e- 0 -5t)ls (b) r(t) = (tcos5t)i + (tsin5t)j + (t)ls
(c) r(t) = (t)i + (t 2 ) 2 + (t 3 ) ls (d) r (t) = (cos t) i + (sin t) 2 + (cos 2t) ls
(e) r(t) = cos(t)i + sin(t) 2 + (ln t) ls (f) r(t) = (et)i + (t)j + (t 2 )ls
(g) r(t) = (t)i + (cos6t) 2 + (sin6t)ls (h) r (t) = (cos t) i + (sin t )j + (sin 5t) ls
130 Chapter 3: 30 Vectors

Question 16 [Guided question to find the intersection of two surfaces]

The diagram below shows a parabolic cylinder z = x 2 - y and a plane z = y - l.

\
\

I
I \
\
I
I ~

\
\

1 2
2 (x + 1).
(a) Solve the surfaces simultaneously to show that y = What is the geometric significance
of this result?

(b) Set x = t for some t E lR and hence show that the intersection of the two surfaces has parametric
representation
r (t) = (t) i + ~ (t 2 + 1) 2 + ~ (t 2 - 1) J5.

(c) For the ~-component above, the surface z = y - 1 was used. Is it incorrect to instead use the
other surface z = x 2 - y?

(d) Suppose that the condition z ::;; 4 were introduced. Find a corresponding restriction for t and
hence find the endpoints of the curve of intersection.

MASTERING MATHEMATICS
3D Parameterising 30 Curves 131

Question 17 [A well-chosen initial parameter can result in a cleaner parametrisation]

The diagram below shows a cone z = ✓x 2 + y2 and a plane z = 1 + x.


-------
z
\
'
/
/

(a) Obtain a parametrisation of the curve of intersection by setting x = t.


(b) By setting y = t , show that a parametrisation of the intersection is

(c) Why do you think the second method is better than the first even though it results in the same
curve?

Question 18 The diagram below shows a cylinder x 2 + y2 = 1 and a plane z = 1 - y.

/
~y

X
/
(a) Find a suitable parametrisation for the cylinder.
(b) Substitute this into the plane to obtain a parametrically defined curve.
(c) Describe what this parametrisation represents geometrically.
132 Chapter 3: 30 Vectors

Question 19 The diagram below shows a paraboloid z = x 2 + y2 and a plane z = 2x - 2y.

- ~~~t~~~> I
I
I

I ----

(a) Eliminate z to show that (x - 1) 2 + (y + 1) 2 = 2.


(b) Find a suitable parametrisation for the equation above.

(c) Hence show that the curve of intersection has parametric representation

r (t) = (1 + v'2 cost )i + (- 1 + v'2 sin t )1 + ( 4 + 4 cos ( t + i)) Jc,.

Question 20 The diagram below shows a parabolic cylinder y = x 2 and a paraboloid z = x 2 + y2


for z :::; 12.

Find a parametric representation for the curve of intersection, including any restrictions on the pa-
rameter t.

MASTERING MATHEMATICS
3D Parameterising 3D Curves 133

0 Challenge Problems ----------------------------- ---------,


Problem 1 [A way to describe a familiar curve in 3D space]
Define two mutually perpendicular unit vectors in space as

Q = ad + a2 j__ + a3 ls
Q = bii + b2 j__ + b3 ls

Define the position vector


p_(t) = (r cos t)g + (r sin t)Q.

(a) Show that p_(t) · p_'(t) = 0 for all t ER

(b) Hence, what kind of curve does p_(t) trace out?


(c) Let £ = cd + c2 j__ + c3 ls represent the position vector of some fixed point (c1 , c2, c3).
Consider the curve defined by the following.

p_(t) = £ + (r cos t)g + (r sin t)Q

What kind of curve does this define?

Problem 2 [The restriction can sometimes be hard to spot!]

Bob is given the parametric vector equation

r(t) =
t+ -
[ t2
1\
+ -t2
l
and asked to produce a sketch.

(a) Show that the Cartesian equation is y = x 2 - 2.


(b) Bob submits the graph of y = x 2 - 2 and his teacher says that he is wrong! Explain why.
L _____________________________ _______________________ _
134 Chapter 3: 3D Vectors

Exercise 3E
Spheres and Circles

~ Fundamentals

Fundamentals 1
The sphere centred at (xo ,Yo, zo) with radius r is

Fundamentals 2
The position vector Q of any point on a sphere is given by

It represents a sphere centred at the position vector _ with radius _ .

Fundamentals 3
Let S represent a sphere. Describe the main steps to
(a) determine if a point P lies inside S, on the surface of S , or outside of S.
(b) determine where a line f, intersects S.
(c) find the Cartesian equation of S given the centre C and a point P on the surface.
(d) find the Cartesian equation of S given two diametrically opposed foes A and B on S.

Question 1

(a) Find the Cartesian equation of the sphere centred at (2, -3 , 5) with radius 6.

(b) Find the intersection of the sphere with each of the coordinate planes.

Question 2 Find the Cartesian equation of the sphere that passes through the origin O and has
centre C(-5 , 3, 4).

Question 3 Find the centre and radius of the following spheres.

(a) x2 - 4x + y 2 + 2y + z 2 = 4 (b) x2 + 2x + y 2 - 4y + z2 - 6z = 2

Question 4 Find the Cartesian equation of the sphere with centre C(6, -2, 3) that touches the

(a) xy-plane. (b) xz-plane. (c) yz-plane.

MASTERING MATHEMATICS
3E Spheres and Circles 135

Question 5 Find the Cartesian equation of the circles formed where the sphere
(x - 2) 2 + (y - 4) 2 + (z - 3) 2 = 36
intersects the xy , y z, x z-planes.

Question 6 Find the Cartesian equation of the largest possible sphere centred at C(3 , 5, 2) but is
still fully contained within the first octant.

Question 7 Define fixed points A(l, -2, 0) and B(l, 1, 3).


(a) Find the set of all points P where the distance of P to A is twice the distance of P to B, and
show that it is a sphere.

(b) Find the centre and radius of the sphere.

Question 8 Find the Cartesian equation of the sphere that passes through P(O , 3, 2) with centre
C(O, 1, 2).

Question 9 [Guided question for a line intersecting a sphere)

Consider the line represented by the vector equation

and the sphere centred at (2, -1 , 3) with radius 3.


(a) Write down the vector equation of the line in the form

(b) Write down the Cartesian equation of the sphere.


(c) Substitute the i , land!;;, components into the equation of the sphere to show that 7,\ 2 -8,\+ 1 = 0
and hence solve for ,\.

(d) Hence, find the two points where the line intersects the sphere.
(e) What can we say if the quadratic in terms of ,\ has no real roots? How about one real root?

Question 10 [Demonstrating a 3D analogy of a familiar result)

Let f be the line that passes through (2, -1 , 0) in the direction of Q = - j__ + ]s. Let S be the sphere
centred at C(l, 0, -1) with radius r. Let the point of contact between Sand f be P.
(a) Find the value of r so that S touches f.
(b) Substitute the vector representation off into the Cartesian equation of S and show that there
is only one solution of,\. Was this to be expected?

(c) Find the coordinates of P.

(d) Find the angle between GP and f. Explain briefly how this result was to be expected.
136 Chapter 3: 3D Vectors

Question 11 Find the coordinates of the points where the line represented by

intersects the sphere centred at the origin with radius 3.

Question 12 Consider the two spheres

S1 : (x - 1) 2 + (y + 1) 2 + (z - 2) 2 = 16
S2 : (x + 1) 2 + (y - 3) 2 + (z + 2) 2 = 4

(a) Show that S1 and S2 are tangential to each other.


(b) Find the coordinates of the common point.
(c) Find another possible radius for S2 such that the spheres will be tangential.
(d) Find the coordinates of the common point when this occurs.

Question 13 [Parametric equation of a circle in 3D space]

Consider a circle with radius r on some plane P containing two perpendicular vectors ']:!; and Yd both
with length r.
z

x-------
--
Let the centre of the circle have position vector£, and let 'Q represent the position vector of any point
on the circle.
(a) Write down the standard parametrisation for the circle centred at the origin with radius r in the
xy-plane.
(b) Use your previous answer to explain how

'Q = f + ']:!; cos 0 + Yd sin 0


represents a circle in 3D space.
(c) Find the radius of the circle and verify that it is r.

: I MASTERING MATHEMATICS
3E Spheres and Circles 137

Question 14 [Spherical coordinates]

Show that the set of equations


X = r COS 0 sin cp
y = r sin esin cp
z = r cos cp

represent a sphere centred at the origin with radius r.

0 Challenge Problems --------------------------------------,


Problem 1 Consider the line ,e defined parametrically by r(.\) = g + AQ and the sphere S
defined by lr - £1 = r.
(a) Suppose ,e is a tangent to S. Prove that this occurs when,\= Q · (£ - g).
(b) Deduce that the tangent to the sphere is perpendicular to the radius at the point of
contact.

Problem 2 Let g and Q be the position vectors of two fixed points A and B in space, and let
Q be a variable position vector that satisfies

(a) Show that

(b) Geometrically, what does the position vector Q represent, and find the main features of it .

(c) What is the geometric significance of the above results?

Problem 3 Consider the two mutually tangential spheres

S1 : lr - £1 I = r1
S2 : lr - £2 1= r2
Show that the point of contact has position vector
138 Chapter 3: 3D Vectors

Chapter 3 Review
3D Vectors

0 Review

Question 1 Find the angle between the vectors 1!, = 3i + j - 2 ls: and 'Q = 4i - 2 j + 5 ls:.

Question 2 Define A(l, 4, -1), B(-2, 8, 4) and C( - 1, 10, - 1). Find all possible points D such
that ABCD forms a parallelogram.

Question 3 Let 1!: = 2i - ls: and 'Q = 3i + 4j + 2Js:. Find a vector with length \/'l29 that is
perpendicular to both 1!: and Q.

Question 4 Consider a cube of side length a.

Prove that the acute angle between the two main diagonals as shown above is 0 = cos- 1 ( ~ )-

Question 5 Determine if the point P( a, b, c) lies on the line represented by

Question 6 Find the vector equation for the interval AB if A = (- 7, 3, 4) and B = (2 , 6, -1).

Question 7 Define the lines


3
P1 : ~(A)= [ ~ ] + A [ ~ 5]

£2 , Tj (µ) = m+ µ [ ~l]
Determine whether ,el and e2 are parallel or not.

MASTERING MATHEMATICS
Chapter 3 Review 139

h
Question 8 Define the lines

el ~(A)= ~1+ A [ 21 i
£2 : ~(µ) = [ iJ +µ [ ~
4
]

Determine whether .e1 and .e2 are perpendicular or not.

Question 9 Define the lines

R1 : ~ (A) = [ ~: l + A [ ~]

£2 : ~(µ) = [ l~l +µ [ ~ ~]

(a) Show that they intersect.


(b) Find the coordinates of the point of intersection.

Question 10 Define the line ,e

Find where the line ,e intersects the plane 2x - 3y + z + 2 = 0.

Question 11 Define the lines ,e1 and .e2 by

£1 : r (A) = [ ; J +A rn
P2 : r(µ) = [ iJ +µ m
(a) Prove that .e1 and .e2 are skew lines.
(b) Find the shortest distance between ,e1 and .e2.
140 Chapter 3: 3D Vectors

Question 12 The diagram below shows a parabolic cylinder z = y 2 and a plane z = x + 2y.

Find a parametric representation for the curve of intersection, including any restrictions on the
parameter t.

Question 13 The diagram below shows a cone z = ✓x 2 + y2 and the upper-half of the unit
sphere x 2 + y 2 + z 2 = 1.

Find a parametric representation for the curve of intersection.

Question 14 Find the centre and radius of the sphere

x2 + 4x + y2 - 2y + z 2 - 6z - 2 = 0.

Question 15 Find the equation of the sphere that has centre C(2, 3, 4) and touches the

(a) yz-plane. (b) x-axis.

Question 16 Find the coordinates of the points where the line passing through P(3 , -1, -2)
and Q(5 , 3, -4) intersects the sphere x 2 + y 2 + z 2 = 26.

Question 1 7 Consider the two spheres

S1 : (x - 1) 2 + (y + 1) 2 + (z - 2) 2 = 64
S2 : (x + 1) 2 + (y - 3) 2 + (z + 2) 2 = 4

(a) Show that S1 and S2 are tangential.


(b) Find the coordinates of the point of contact.

MASTERING MATHEMATICS
Chapter 3: Investigation Task 141

~ Investigation Task

Lunar laser ranging retro-reflector

One of the ways of precisely measuring the distance of the Moon from the Earth is to use some-
thing called a Laser Ranging Retro-reflector, which shoots a laser from the Earth to a series of
corner-mirrors on the moon that were planted during Apollo Programs 11 , 14 and 15.

This investigation task will allow the student to see how the study of 3D vectors can be used in
a practical physics scenario.

Write a two-page article that demonstrates how the mathematics behind 3D vectors was used in
the laser ranging retro-reflector and in particular the idea behind corner mirrors. Your answer
should include the following.

• A definition of an angle of incidence, an angle of reflection, and the relationship between


the two angles.

• A derivation and/or proof of the key result about reflections that allows a corner-mirror to
actually be useful.

• Relevant calculations that demonstrate an approximation of the distance of the Moon from
the Earth.

• All relevant diagrams.


142 Chapter 3: 3D Vectors

~ Investigation Task

Equation of a plane

In this chapter, we study lines in 3D space and work lightly with planes in 3D space. This
investigation task will allow students to study planes more carefully, which will enrich their
understanding of working in 3D space. This investigation task is also best accompanied by the
investigation task on the Cross Product for a complete pictu~e.

Question 1 In your study of linear functions, you needed a gradient and a point to uniquely
define a linear function. What do you need to uniquely define a plane?

Question 2 Write a one-page article, complete with diagrams, that shows the equation of a
plane and the derivation of that equation.

Question 3 Answer the following, and also provide an example to demonstrate your technique.

(a) How do you determine if two planes are parallel?


(b) If two planes intersect, how do you find the equation of the line of intersection?
(c) How do you find the angle between two planes?
(d) How do you find the distance between a point and a plane? What is the formula for it in
general?

(e) How do you find the distance between two parallel planes?
(f) How do you find the plane that passes through a particular point and is perpendicular to a
particular vector?

(g) How do you find the plane through a particular point that is parallel to a particular vector?
(h) How do you find the equation of a plane through three given points?
(i) How do you find the equation of a plane that contains two intersecting lines in space?
(j) How do you determine if two vectors are coplanar?

MASTERING MATHEMATICS
Chapter 3: Investigation Task 143

q Investigation Task

Cross Product

Any standard course on linear algebra will include a section on the cross product, which is an
incredibly useful tool when it comes to the study of 3D vectors. This investigation task, accom-
panied by the investigation task on the equation of the plane, will give a more complete study of
3D space and 3D vectors.

Question 1

(a) Define a cross product and give the formula for it.
(b) What is the output of a cross product?
(c) What does it mean when we say that a cross product is non-commutative?
(d) What is the right-hand rule, and why is it relevant to the cross product?
(e) What is torque, and why is it relevant to the cross product?

Question 2

(a) Give the formula for the determinant of a 2 x 2 matrix.


(b) How would you calculate the determinant of a 3 x 3 matrix?
(c) Give the definition of the cross product that involves the determinant of a 3 x 3 matrix.
(d) Make up two vectors, find their cross product , and show that the cross product is non-
commutative.

Question 3

(a) What would you expect to happen if you cross product a vector with itself? Why?

(b) Prove that 'Jd x 'Q is indeed perpendicular to both 'Jd and 1!_.

(c) For the dot product, we have the formula 'Jd · 'Q = l'JJ: l l'QI cos 0. There is a similar formula for
the cross product. What is it , and explain how it can be used to find the area of a triangle.

(d) For the dot product , two vectors are perpendicular if and only if 'Jd · 'Q = 0. There is a similar
property for the cross product. What is it , and provide derivations.

(e) What is the relationship between 'Jd x 'Q and 'Q x 'Jd?

Question 4

(a) What is a scalar triple product? Prove that Q · (!2 x £) = (g x 12) · £·


(b) How is the scalar triple product used to calculate volume? Give full derivations.
(c) How can the scalar triple product be used to prove that three vectors are co-planar? Explain.
FURTHER INTEGRATION

■ Integration by Substitution

■ Trigonometric Integrals

Trigonometric Substitutions

Harder Standard Integrals

Partial Fractions

t-formula Substitutions

Integration by Parts
146 Chapter 4: Further Integration

Exercise 4A
Integration by Substitution

~ Fundamentals

Fundamentals 1
Complete the following formulae.

(a) JJ' (X) (f (X) f dx (b) Jf'(x)


f(x)
d
X (c) j f' (x )ef( x) dx

Fundamentals 2
Complete the following formulae.

(a) j f' (x) sin (f (x)) dx (b) j f' (x) cos (f (x)) dx (c) j f' (x) sec 2
(f (x)) dx

Fundamentals 3
Complete the following formulae.

f'(x) d
(a)
J✓l - (f(x)) 2
X
(b)
J f'(x) d
1 + (f(x))2 X

Note from the author: Although these formulae are on your reference sheet, it is useful to have
them memorised to allow for quicker recognition and most importantly correct recognition of the cor-
rect form. It is easy to get the various forms mixed up , and many integrals can look very similar
despite having entirely different answers.

For example, the integrand of


1 d
/ x+ft X
looks similar to the integrand of
1 d
/ XyX+yX X
but the first is a log integral and the second is an inverse tan integral. It is very difficult to see this
if you rely too heavily on the reference sheet. Even worse , you may not even see it at all because you
were not able to develop that 'sixth sense' that allows you to immediately spot the correct form.

MASTERING MATHEMATICS
4A Integration by Substitution 147

Question 1 [Practising figuring out the substitution]

More often than not when doing integration by substitution, the substitution is of the form

u = the function inside another function


2
2 2
For example, for the integral / xex dx we would let u = x because x is 'inside' the exponential
function. State an appropriate substitution for each of the following integrals.

(a)
3
/ x 2 ex dx (b) j x sin(x 2) dx (c)
2
(d) j sin x ( 1 + cos x) 3 dx (e)
J x d
(1 + x2 )3 X
(f)
J sec x d
✓l + tan x x

Question 2 Find the following integrals using an appropriate substitution or the formula

j J'(x ) (f (x)f dx = -n+l-(f(x)f+


1 1
+ C.

(a) J✓ l + x d· (b)
j sinx - cosx dx (c) j x✓x+Tdx
x2 X (cos x + sin x )2

(d)
j (x2 + x3
1)3
dx (e) / (x + l) (2 - x )5 dx (f) J)1+x x dX

(g) j cossinxx dx (h)


j ex - e-x
2 dx (i) j l:x dx
3 (ex + e-x )

(j) j ✓3sinx- COSX


d
X
(k) 3
/ sin x cos x dx (1) j sec2 x tan x dx 3

Question 3 Find the following integrals using an appropriate substitution or the following formulae.

j J'(x ) sin (f (x)) dx = - cos (f (x )) + C


j J' (x ) cos (f (x)) dx = sin (f (x )) + C
j J'(x) sec2 (f(x)) dx = tan (f(x)) + C

(a)
2
/ x sin(x ) dx (b) / ex cos (ex ) dx (c) j: 2 sec
2
m dx

(d) / Jx sin (v'x) dx (e) j cos~nx) dx (f)


2
/ sec 2 x sec (2 tan x ) dx
148 Chapter 4: Further Integration

Question 4 Find the following integrals using an appropriate substitution or the formula

JJ' (x )ef(x) dx = ef(x) + C.

(a) J (2x + 1)ex +x dx


2
(b) J ecosx sin x dx (c) Jex-dx
x2
1

(d) fvxfa
- dx (e) f 2-dx
ex2 (f)
/ etanx
- -2d x
cos x

Question 5 Find the following integrals using an appropriate substitution or the formula

j f'(x)
f(x) dx = ln lf(x) I + C.

2
(a) / sinx d (b) / --1 dx / 1 - sec x dx
1 + COSX X
(c)
xlnx x - tanx

(d)
Jex - e-x dx (e) / 1 d (f) j 1 +sin2x d
ex+ e-x fa(l + fa) x cos 2 x x

Question 6 Find the following integrals using an appropriate substitution or the formula

j f'(x)
2
dx = sin- 1 (f(x)) + C.
J1 - (f(x))

(a) J x
✓l - x4
d
X
(b) J cosx
✓25 - 4sin 2 x
dx (c) J ✓l
ex
- e2x
dx

(d) /
✓16
sec
2
xd
- 25tan 2 x x
(e) /
1
xJl - ( lnx)
2 dx
(f) /
Rx 1 d
x2
2

Question 7 Find the following integrals using an appropriate substitution or the formula

J f'(x)
l+(f(x))
2
dx = tan- 1 (f(x)) + C.

(a)
J~
+ 1 x6
2
dx (b)
J cosx
2
4 + 9 sin x
dx
(c)

sec x d 1 d
(d) 2 (e) (f)
/ 9 + 4tan x x / fa(l+x) X

Question 8 Find the following integrals using an appropriate substitution.

(b) j ta:,r dx (c) j: 2 tan m dx

MASTERING MATHEMATICS
4A Integration by Substitution 149

Question 9 [Useful way to handle square roots]

Find the following integrals using the substitution x = u2 .

() J
a
1 d
l+ f i x
() J fa
C l+fi
d
X

0 Challenge Problems

Problem 1 [Euler substitution]

Find j x ✓x 2
1
+ 4x -4
dx using the substitution u = ✓x 2 + 4x - 4 - x.

Problem 2 [Verifying the mean of the normal distribution]

Recall that the probability density function of the normal distribution with meanµ and st andard
deviation CY is
1 l( x-µ)2
f( x) = - - e-2 - a
(Yv12ii
The formula for expect ed value in the discrete scenario is given by

E(X) = I: xp(x ).
X

The equivalent formula for the continuous case is

E(X) = 1_: x f( x ) dx.

Verify that E(X) = µ for the normal distribution, using the formula above.

Problem 3 Find the following using appropriat e manipulations and substitutions.

(a) J e ex +x dx (b)
J e 2x
e 2x -
+l
1
dx (c) / 1 d
x +y1x X

(d) / 1 d (e) / 1 d (f) j x4x ++ 11 dx


x ✓x 2 - 1 x x ✓x + x 2 x 6

(g) /
✓l -
1
e2x
d
X
(h) 1 dx
/ -5 -
x +x
(i) j l+,Ixx dx
(j) / 1 d (k) j x +1
2
dx (1) jfi5 - - dx
✓x 3 + x 2 - x- 1 x x ✓ - x 4 + 3x 2 - 1 1-x 3
150 Chapter 4: Further Integration

Exercise 4B
Trigonometric Integrals

~ Fundamentals

Fundamentals 1
The form below is the most important standard form when it comes to trigonometric integrals.

JJ' ( X) (j (X) f dx
Write down the formula for it .

Question 1 Find the following.

(a) 2
/ sin x cos x dx (b) j cos x sin x dx
3
(c) j tan x sec x dx
4 2

(d) j sec 5
x tan x dx (e) j sec x tan x dx
2
(f) j sec xtanx-/secxdx
3

Question 2 Find the following.

(a) j sin x dx
3
(b) j cos xdx
3
(c) j tan 3
xdx

(d) j tan xdx


2
(e) j sec x dx
4
(f) j tan 4
xdx

Question 3 Find the following.

(a) j sin 5
xdx (b) j cos 5
xdx (c) j tan 5
xdx

Question 4 [Double-angle formulae]

Find the following. For part (c) , try it without using previous parts.

(a) 4
/ sin xdx (b) j cos 4
x dx (c) / sin 4 x + cos 4 x dx

Question 5 Find the following.

(a) j tan x sec 3


x dx (b) j sec 2
x tan 2 x dx (c) j tan x sec x dx
3

(d) / sin 2 x cos 3 x dx (e) j sec 2


x tan 3 x dx (f) j sin x cos x dx
5 2

MASTERING MATHEMATICS
48 Trigonometric Integrals 151

Question 6 Find the following.


2 3
sec x dx sin x dx (c) sin x dx
(a) (b)
/ tan x / cos 3 x / cosx
3 4
sec x dx
(d)
J c~s3 x dx
Sln X
(e)
/
sin x dx
✓cosx
(f)
/ tan 2 x

Question 7

(a) Find / sin x cos x dx using t hree different methods.

(b) Explain why you obtained t hree different answers, despite all three techniques being valid.

Question 8 [Products to sums]

Find the following.

(a) / sin 2x cos 3x dx (b) j cos5xcos2xdx (c) j sin 7x sin 5x dx


Question 9 [Sums to products]

Find the following.


sin 6x - sin 2x dx sin 2x - sin x dx
(a) (b)
/ cos 6x + cos 2x / cos 2x - cosx
sin x + sin 2x d sin x + sin 2x + sin 3x dx
(c) (d)
/ 1+ COS X + COS 2x X / 1 + cos x + cos2x

0 Challenge Problems

Problem 1 Find the following.


1 1 d
(a) dx (b)
/ a + b2 sin 2 x
2 / a2 + b2 cos 2 x x
1 1
(c) dx (d) dx
/ a cos x + b2 sin 2 x
2 2 / ( a cos x + b sin x) 2

Problem 2 Evaluate r
Jo a -
bl
cos x
dx using the substitution (a-b cos x ) (a+b cos y) = a 2 -b 2 .

Problem 3 [Orthogonality relation for Fourier Analysis]

Prove that
Lt ) dx = { 02£
Lt) cos (mw
(n1r if n =m
1 L
_L cos if n #- m
152 Chapter 4: Further Integration

Exercise 4C
Trigonometric Substitutions

~ Fundamentals

Fundamentals 1
Write down an appropriate trigonometric substitution for x to simplify the following.
(a) ✓a 2 - x 2 (b) ✓a 2 + x 2 (c) ✓x 2 - a 2

Question 1 Evaluate the following.

(a) t _!_
v13
l
x2 ✓ 1 + x 2
dx (b)
/,2 ✓4 - x2
I X2
dx (c) In~ o (1 + x2)2
1
3 dx

Question 2 Find the following.

(a) / 1 d (b) / 1 d (c) / 1 d


x2 ✓4 - x2 x x2 ✓4 + x2 x x2 ✓x2 - 4 x

Question 3 Find the following.

(a) / 1 d (b) / 1 d (c) / 1 d


x ✓4 - x 2 x x ✓4 + x 2 x x ✓x 2 - 4 x

Question 4 Find the following.

(a) j ✓4x~x2 dx (b) j ✓4x: x2 dx (c) J✓x;2- 4 dx

Question 5 Find the following.

(a) j ✓4:x2dx (b) j ✓4:x2 dx (c) j ✓x:-4dx

Question 6 Find the following.

(a)
j x2
dx (b)
j x2
dx (c)
j x2
dx
✓4 - x2 ✓4+x 2
✓x 2 - 4

Question 7 Find the following.

(a) / 1 3 dx (b) / 1 3 dx (c) / 1 3 dx


(4 - x 2 ) 2 (4 + x 2 )2 (x 2 - 4)2

MASTERING MATHEMATICS
4C Trigonometric Substitutions 153

Question 8 Find the following.

(a) j )a 2 - x 2 dx (b) j )a 2 + x 2 dx (c) j )x 2 - a2 dx

Question 9 Find the following.

(a) / 1 d (b) / 1 d (c) / 1 d


✓a2 - x2 x ✓a2 + x2 x ✓x2 - a2 x

Question 10 Find the following.

(a) / 1 3 dx (b) / 1 3 dx (c) / 1 3 dx


(a2-x2)2 (a2+x2)2 (x2 - a2)2

0 Challenge Problems --------------------------------------,


Problem 1 Evaluate the fo llowing.

(a ) tfiEd
0 1- X X
(b)
l/fd
0 +
- - dx
1 X
(c) fVX
0
--dx
+ 1 X

(d) t0
,Ix d
--
1- X
X (e) fa' ✓2x - x 2 dx (f)
J,3
1
x2
✓4x - x 2
dx

Problem 2 The diagram below shows the ellipse :: + t: = 1 and the region inside it.

Prove that the area of the ellipse is 1rab by considering an appropriate integral and using a
I trigonometric substitution.
I
I
1 Problem 3 Let b > a > 0. Use the substitution x = a cos 2 0 + b sin 2 0 to evaluate
I
I
I
{b ~ dx.
I
I
I
la V~
I
L ____________________________________________________ _
154 Chapter 4: Further Integration

Exercise 4D
Harder Standard Integrals

~ Fundamentals

Fundamentals 1
Consider an integral of the form

J l
ax 2 + bx+ c x
d

and let 6 be the discriminant of t he quadratic in the denominator. Describe t he general technique
t o integrat e the above if
(a) 6 =0 (b) 6 <0

Fundamentals 2
W hen dealing wit h an integral of t he form

px +q
J ax 2
d
+bx + c x

first force out the form j ~(~


?dx to handle the J_ _ component of t he numerator , and t hen
it becomes t he same as t he above scenario.

Fundamentals 3
W hen dealing wit h an integral of t he form

J✓ax l d
2 +bx + c x

complet e t he square and it will often be an i _ ___ s _ _ integral. Ot herwise, at _ _ _ __


substit ution can be used aft er completing the square.

Fundamentals 4
W hen dealing wit h an integral of the form

px + q
J d
✓ax +bx + c x
2

first force out t he form _ ____ and t hen it becomes t he same as t he above scenario.

MASTERING MATHEMATICS
4D Harder Standard Integrals 155

Question 1 Find the following.

(a) J - X- dx (b)
j - x2- dx (c) j 2xx -+l1 dx
2x -1 x +l

(d)
j - x-2 dx (e) j x 2 +x -4xl- 3 dx (f) j - x4-dx
2x -1 x -l

Question 2 Find the following.


1 1 d 1 d
(a) / (b) (c)
x2 + 2x + 2 dx / 4x 2 + 4x +5 x / x2 + x +l x

Question 3 Find the following.


+1 d 2x + 1
(a) / x +2 d
x 2 + 6x + 10 x
(b)
J 9x 2
6x
+ 12x + 4 x
(c)
J d
x 2 + 6x + 10 x

Question 4 Find the following.


x2 +1 2
+l 2x 2 - 1
(a) J x 2 + 2x + 2 dx (b)
J x - x
x2+ x +l x
d
(c)
J -2- - - dx
x + 4x + 5

Question 5 Find the following.


1 d 1
(a) / J6 xl x2 dx (b) (c) dx
/ v- x 2 + 4x - 3 x / v-2x - x 2

Question 6 Find the following.


x + l dx
(a) J v x - x2 (b)
J x d
v2x - x 2 x
(c)
JV5 + x
X - x2
d
X

Question 7 Find the following.

(a)
Jv~ F x dx (b)
Jv~ ~ dx (c)
Jv~ ~dx

Question 8 Suppose b > a > 0. Find the following.


1 1 d
(a) / dx (b)
v-a - 2bx - x 2
2 / x 2 + 2ax + b2 x
156 Chapter 4: Further Integration

0 Challenge Problems ------------------------------ --------,


Problem 1 [Alternating Harmonic Series]
Define the integral

where n is a positive odd integer.

(a) Show that


1 1 1 1
I = l - - + - - - + · · · + - - ln 2.
n 2 3 4 n

(b) Prove that


1
0 <In<--.
n +l

(c) Hence, write down the exact value of

1 1 1
1--+---+ ...
2 3 4

22
Problem 2 [Stretching 1r ~ to the limit]
7
Define the integral
l X4n (1 _ X )4n
In =
ln
o l+x 2
dx

where n is a positive integer.

(a) Show that


X4n (l _ x)4n n 22n
1_
__ +_x_2_ = p (x) + (-1 ) -1 _+_x_2

where P(x) is a polynomial of degree 8n - 2.

(b) Prove that In ➔ 0 as n ➔ oo.

(c) Show that

where Rn is some rational number.

(d) Hence, explain the significance of the number

for large values of n.


L ______________________________ ______________________ _

MASTERING MATHEMATICS
4E Partial Fractions 157

Exercise 4E
Partial Fractions

~ Fundamentals

Fundamentals 1
There are three main categories of partial fraction problems. For each of them, write down the
standard decomposition. A template for the first one has been done for you.
(a) Simple linear factors .

J___ l_ _ _ dx =
(x+a)(x+b)
f-7-
x+a
+ _?_ dx
x+b

(b) Irreducible quadratic factors.

(c) Repeated linear factors.

Question 1 [Simple linear factors]

Consider the partial fraction decomposition

J__ x_-_l__ dx = !-A-


(x+l)(x-2) x +l
+ _B_ dx
x -2 ·

(a) Show that


A(x - 2) + B(x + 1) =x - l.

(b) Equate the coefficient of x to find an equation in terms of A and B.


(c) Equate the constant term to find another equation in terms of A and B.
(d) Solve the expressions simultaneously to find A and B.

J__
(e) Hence, find
x_-_l__ dx.
(x+l)(x-2)
158 Chapter 4: Further Integration

Question 2 Find the following.

(a) J 5
( 2x + 1) ( X + 3)
d
X
(b) / 1 d
3x 2 + 18x + 24 x
(c)
J (X
3x + 2
+ 1) (2x + 1)
d
X

(d) / 1
x (x + l)(x-3) x
d (e) f 2
5
(x-4)(1- x)
dx (f) J,
1
2 X - 8
- - - - - dx
( 2x - 1) (X + 2)

Question 3 [Useful for resisted motion and the logistic equation]

Show that
1
/ x 2 - a2
dx = -1
2a
1x-a1+ C.
ln - -
x +a

Question 4 [Irreducible quadratic factors]

Consider the partial fraction decomposition

J x +l
(x 2 + 1) (x - 1)
d x- !Ax+B
-
-2- - + -Cd
x +1
- X
x - 1 ·

(a) Show that


(Ax + B) (x - 1) + C (x 2 + 1) =x + l.

(b) Substitute x = 1 to find the value of C.

(c) Equate the coefficient of x 2 to find A.

(d) Equate the constant term to find B.

(e) Hence, find

J x+ 1
(x 2 + 1) (X - 1)
d
X.

Question 5 Find the following.

(a)
J__ 2
x_-_2- - dx
(x + 4)(x + 1) (b) J 4x +2
(x + l)(x + 3) x
2
d
(c)
J x 2 + x + 11 d
(x+2)(x 2 +9) x

Question 6 [It 's easier than you think!]

x2
Find
J (x 2 +l ) ( x 2 -2 ) dx.

MASTERING MATHEMATICS
-I
4E Partial Fractions 159

Question 7 [Repeated linear factors]

Consider the partial fraction decomposition

J___ x_ __ dx =!--A-+ _B_


(x - 1) 2 (x + 1) (x - 1) 2 x- l
+ _C_ dx
x+1 ·

(a ) Show that
2
A(x + 1) + B(x - l)(x + 1) + C(x - 1) = x.

(b) Substitute x = 1 to find A.


(c) Substitute x = - l to find C.
(d) Substitute any appropriate value of x to find B.
(e) Hence, find

J___ x_ _ _ dx.
(x - 1) 2 (x + 1)

Question 8 Find the fo llowing

()
a
x +9 d
/ x(x -3)2 x
(b)
J (x 2 -
x
1) (X + l)
d
X
(c)
J x+l d
(x- l)(x + 2) 2 x

Question 9 [Rationalising the integrand]

Use substitutions of the form u = xr to find the following.

(a) / x ~~ dx (b) / l~ 3 dx

Question 10

(a) Prove that x 4 + x 2 + l = (x 2 - x + l)(x 2 + x + l).


(b) Hence, evaluate
roo 1
lo x4 + x2 + l dx .

Question 11

(a) Prove that x 4 - x 2 + l = (x 2 - v-13x + l)(x 2 + vf3x + 1).


(b) Hence, evaluate
roo 1
lo x4 - x 2 + l dx.
160 Chapter 4: Further Integration

Question 12 [Avoiding partial fractions]

Let R > l. Define the integral

R x
I =
J l_
R
- 3- - dx.
X + l

(a) Prove that x 3 +1= (x 2 - x + l)(x + l) .


1
(b) Using the substitution y = -, show that
X

R l
I =
J 1--
R
- 3- - dy.
y +l

(c) By considering the average of the two expressions for I , find I.


(d) Hence, evaluate
oo X

lo --dx
o x3 + 1 ·

Question 13
(a ) Let A (x) and B (x) be of t he form p cos x + q sin x for p, q E lR in the following expression

12 A(x) B(x)
9 cos 2 x - 4 sin 2 x = 3 cos x - 2 sin x + 3 cos x + 2 sin x ·
Find A(x) and B(x).

(b) Hence, evaluate


(i 12 d
Jo 9cos 2 x - 4sin 2 x x.

Question 14
(a ) Prove that x 4 +4 = (x 2 + 2x + 2)(x 2 - 2x + 2).
(b) Find A and B such that
16 2x+A 2x +B
x4 +4 x2 + 2x + 2 x2 - 2x + 2 ·

(c) Hence, show that

2
k 16 dx = In kk2 + 2kk + 2 1+ 2 tan- 1 (k + 1) + 2 tan- 1 (k -
In
I
- --
4 1).
0 X +4 -2 +2

(d) Hence, evaluate


00
16
1 --dx
o x4 + 4 ·

MASTERING MATHEMATICS
4E Partial Fractions 161

0 Challenge Problems

Problem 1 [Residue method for partial fractions]

Let P(x) be a monic polynomial of degree n with real distinct roots ak, where k = 1, 2, 3, · · · , n.
1
Using partial fractions , p (x) can be expressed in the form

_l_
P(x)
= t
k=l
Ck
X - ak
'

for some reals ck.

(a) Show that

(b) Hence, show that


1 n 1
P( x ) = ~ P'(ak)(x - ak)"

Problem 2 [Generalised linear decomposition]

Let n be a positive integer and consider the decomposition

1 co c1 c2 Cn
- - - - - - - -- = - + - - + - - + • · • + - -
x(x+l)(x +2)···(x+n) - x x+ l x +2 x+n·

(a) Show that


(- ll
ck = ~
(n) for all
k k = 0, 1, 2, • • • , n.

(b) Hence, show that


162 Chapter 4: Further Integration

Exercise 4F
t-formula Substitutions

~ Fundamentals

Fundamentals 1

If t =tan(;) , write down the t-formula expansion of the following.

(a) sin 0 (b) cos 0 (c) tan 0

Fundamentals 2

If t =tan(;) , then write down an expression for d0.

Question 1 Find the following.


1 1 1
(a ) (b) (c)
/ sin0
+ tan0 d0 / 5
+ 4cos0 d0 / 1 + cos 0 d0
1 1 1
(d) (e) (f)
2 cos 0 d0
/ 3+ / 2 - cos 0 d0 / 5 + 3 sin 0 -
4 cos 0 d0

Question 2 [Partial fractions involved]


Find the following.
5 1 1
(a ) / (b) (c)
3 sin 0 + 4 cos 0 d0 / 4
+ 5 cos 0 d0 /
3 + 5 cos 0 d0
1 1
(d) / (e) (f) / l d0
3 - 5 sin 0 d0 / 12
+ 13 cos 0 d0 8 - 17 sin 0

Question 3 Find the following.

(a) f l - sin ~ + cos 0 dB (b)


1
/ 1 + sin 0 - cos 0
d0 (c)
1
/ sin 0 + cos 0
d0

Question 4 Consider the integral


I= j sec0d0.
(a) Find I using t-formula substitutions.
(b) Find I using a different method, by first multiplying the top and bottom by (sec 0 + tan 0).
(c) Show that the two answers are equivalent.

MASTERING MATHEMATICS
4F t-formula Substitutions 163

0 Challenge Problems

Problem 1 Find

J l
1 + sin 20
d0.

Problem 2 Find the following.

(a)
J l
1+4cos 2 0
d0 (b)
J l
1 + sin 0 cos 0
d0

(c)
J l
sin 4 0 + cos 4 0
d0 (d)
Jvtan0
sin20
d0

Problem 3

(a) Prove that a6 + b6 = (a 4 - a 2 b2 + b4 )(a 2 + b2 ).

(b) Hence, find j sm. 0 +1 cos 0 d0.


6 6

Problem 4 Find

J l
a sin 0 + b cos 0
d0.

Problem 5 Let a> l. Find the value of

r
Jo a -
1
cos 0
d0.

Problem 6 [Differentiation Under The Integral Sign]

(a) Let a > l. Find the value of


r 1
} o ( a - cos 0) 2
d0
·

(b) By differentiating the answer to the previous question, show that

1 1
!!:__r d0=r~( )de
da } o a - cos 0 } o da a - cos 0 ·

L ____________________________________________________ _
164 Chapter 4: Further Integration

Exercise 4G
Integration by Parts

~ Fundamentals

Fundamentals 1
Complete the following formula for integration by parts.

j uv' dx =
Fundamentals 2
Complete the following formula for integration by parts.

Fundamentals 3
(a) When integrating an isolated function using integration by parts, it is often fruitful to set
v' = _ to introduce an x term.

(b) When selecting what goes into the v' term, it is important to ensure that it will be easy to

Question 1 Find the following using integration by parts.

(a) j xex dx (b) j xlnxdx (c) j vxlnxdx


(d) j xsinxdx (e) j xsec2 xdx (f) j x sin - x) dx
l (

(g) j x tan- (x) dx


1
(h) j lnx
x2
dx (i) j x sin x cos x dx
Question 2 [Integrating isolated functions]

Find the following.


(a ) j lnxdx (b) j sin- (x) dx
1
(c) / tan- 1 (x) dx

(d) j ln(x 2 + 1) dx (e) j eVXdx (f) / sin (ln x) dx

Question 3 [Two applications of integration by parts needed]

Find the following.


(a) j ex sinx dx (b) j x 2 sinx dx (c) j (lnx) 2 dx

MASTERING MATHEMATICS
4G Integration by Parts 165

Question 4 [Prioritising the v' term]

Find the following.

(a) / x 5 )1 + x 3 dx (b)
J +x ✓1
x7
4
dx (c) j x cos ( x dx
3 2
)

(d)
2
/ x 3 ex dx (e) Jx e6 sin ( e3 x) dx (f) j x ~dx 3

Question 5 Find / sec 3 x dx using integration by parts.

Question 6 [Definite integrals using integration by parts]

Evaluate the following.

(a) lo"' xcosx dx (b) le xlnxdx (c) lo


1
tan - 1
( x) dx

(d) lo~ e - x cos x dx (e) fo'f. x tan 2 x dx (f) fo 1 x 3 tan -l (x) dx

(g) 1o= xe-x dx (h) l ,


0
x3e- x dx (i)
lnoo ln(l + ex) dx
0 eX

0 Challenge Problems

Problem 1 Find the following.

(a) j ✓4x~ x2 dx (b) 1~dx (c) j lnx


2
d
X
(1 + ln x)
1
/ sin- x dx / tan-1 ,Ix dx
(d) / ln ( x + J x2 - a 2 ) dx (e) (f)
fi+x fi+x

Problem 2 [Application to the Laplace transform]

The Laplace transform is an advanced technique used to solve differential equations, usually
taught in universities. It is an operation on J(t) defined as

The output is a function in terms of s.


1 s
(a) Show that £(t) =2 . (b) Show that £( cos at) = .
s s 2
+ a2
L ____________________________________________________ _
166 Chapter 4: Further Integration

Exercise 4H
Reduction Formulae

~ Fundamentals

Fundamentals 1
(a) A reduction formula is a recurrence formula where an integral In is expressed in terms of a
similar integral, but with a lower i _ _ such as In -I or In _ 2 .

(b) Once the recurrence is obtained , the integral may be calculated by r _ ___ substituting
the formula into itself until the integral reduces to either 10 or fi.

(c) Calculate lo or Ii directly using the definition of _ _ . Do not use the reduction formula
to try to calculate them because this will not work.

Fundamentals 2
Most reduction formula problems require the use of integration by P - , though there are some
problems that do not require it. Be mindful that with some problems , you will need to use
integration by p _ twice.

Fundamentals 3
The following identities are often useful for reduction formula problems.

(a) sin 2 x = (b) cos 2 x =


(c) tan 2 x = (d) sec 2 x =
(e) xn =l+(-- )- (f) xn = l - ( - - ) -

Question 1 Define for n 2 0

(a) For n 2 1, show that

(b) Hence, find / x 2 ex dx.

Question 2 Define for n 2 0


In= j x (ln xf dx.

(a ) For n 2 1, show that

2
(b ) Hence, find/ x (lnx) dx.

MASTERING MATHEMATICS
4H Reduction Formulae 167

Question 3 Define for n 2 0

(a) For n 2 2, show that

(b) Hence, find lo 'I; x 2


cos x dx.

Question 4 [Integration by parts is not always needed]

Define for n 2 0

For n 2 2, show that


1
In= - - - In-2·
n- 1

Question 5 Define for n 2 0

For n 2 2, show that


1
In= - - - In-2·
n- 1

Question 6 Define for n 2 0

(a) For n 2 1, show that


2n
In= --In-l·
2n + 1
1
(b) Hence, find fo (1 - x 2 )3 dx.

Question 7 Define for n 2 0

(a) For n 2 1, show that


3n
In= --In-2 ·
3n+2
1
(b) Hence, find fo x(l - x 3 ) 4 dx.

Question 8 Define for n 2 0

(a) For n 2 1, show that


2n
In = - --In-l·
2n + 3
(b) Hence , find lo 1
x 3 v'f="x dx.
168 Chapter 4: Further Integration

Question 9 Define for n 2:: 0

(a) For n 2:: 1, show that


I = 2/2 _ 2n I
.n 2n + 1 2n + 1 n- l ·
l X2
(b) Hence, find
ln
0
y1l+x dx.
1+X

Question 10 Define for n 2:: 0

(a) For n 2:: 2, show that

(b) Hence, find la~ sin 5 x cos 2 x dx.

Question 11 Define for n 2:: 0


xn
In =
J ~ dx.
vl - x 2
(a) For n 2:: 2, show that
1 n-1 ~ n- 1
In= --x V 1 - x~ + --In-2·
n n
x3
(b) Hence, find
JvT=x2 1- x 2
dx.

Question 12 Define for n 2:: 0

(a) For n 2:: 1, show that

In = 2 (n l- 1
) ( (
x2 +X1) n- 1 + (2n - 3)In-l) .
1
(b) Hence, find j (l + x 2 ) 3 dx.

Question 13 Define for n 2:: 0


l xn
In=
ln
0
(
X + 1) 2 dx.
(a) For n 2:: 2, show that
1 n
In= ( ) - --In-1·
2n - l n-1
l X3
(b) Hence, find
ln
0
(
X +1
) 2 dx.

l MASTERING MATHEMATICS
4H Reduction Formulae 169

Question 14 Define for n ~ 0

J2n-2 n- 2
(a) For n ~ 2, show that In = - - - + - - In-2·
n-1 n- 1

(b) Hence, find la'!, sec 6


x dx.

Question 15 [Glimpse into the definition of the factorial for non-integer n ]

Define for integer n ~ 0


In = 1a=xn-le-x dx.
You may assume that for all real n > 0, lim x ne-x = 0.
X --t CX)

(a) Show that Ii = 1.

(b) Show that In+l = nin.


(c) Deduce that In+l = n!
l n In (-lr
(d) = --.
Let Jn =
fno (lnx) dx. Prove that -
h n

Question 16 [Wallis Product]

Define for n ~ 0
In = fo'!x sinn x dx.
n-1
(a) For n ~ 2, show that In = - -In-2·
n
(b) Hence , show that
2n - 1 2n - 3 3 1 1r
hn = --X X .. •X - X - X -
2n 2n - 2 4 2 2
and
2n 2n - 2 4 2
12 - -- x - - - x .. •x-x - xl
n+l - 2n + 1 2n - 1 5 3

(c) Explain why h > h+l ·


(d) Deduce that
7r ( 2n ) 22 X 42 X ... X (2n) 2 7r
2 2n + 1 < 1 X 32 X 52 X . . . X ( 2n - 1) 2 ( 2n + 1) < 2.

(e) Write down the value of the Wa llis Product


170 Chapter 4: Further Integration

l·U@iiiiHl¥MH+--------------------------------------,
Problem 1 Define for n 2 0

In = la"' cos n x dx.


2

(a) For n 2 1, show that


2n -1
In = --In-1·
2n

(b) Hence, show that In = 22 :+1 (2,.,~) ·


Problem 2 Define for n 2 0
In = j sincosxnx dx.
(a) For n 2 2, show that
2
In= - - cos(n - l)x - In-2·
l - n

(b)
~cos 5x sin x dx.
H ence, fi n d
In
O cosx

Problem 3 Define for n 2 0


In = r si~smxnx dx.
Jo
(a ) For n 2 2, show that In= In-2·
(b) Hence, find all possible values of In and state when they occur.

Problem 4 Define form , n 2 0

(a) For n 2 1, show that


n
I(m, n) = - - I ( m + 1, n - l) .
m+l
m!n!
(b) Hence, show that I(m, n) = ( )'.
m+n+l .
1
(c) Find the value of fo (x - x 2 f dx.

Problem 5 Define for n 2 0

MASTERING MATHEMATICS
41 Further Substitutions 171

Exercise 41
Further Substitutions

&fu Fundamentals
Fundamentals 1
The variable in a definite integral is called t he d _ _ _ variable, since t he final answer does
not contain t hat variable. Hence, what can we say ab out t he following integrals?

t f( x) dx t f( u) du

Fundamentals 2
In general, let
u = expression inside t he bracket
when proving identities involving integration. Write down a suitable substit ution for the following.

(a) t f(- x) dx (b) t f( ax) dx

(c) t f (a - x) dx (d) t f (a + x) dx

Question 1 Prove that


[ f( x) dx = la"f( a - x) dx .

Question 2 Use the identity from t he previous question to calculat e t he following.

(a)
{~
Jo
3
sin x
sin 3 x + cos 3 x
dx (b) t0
yfilllX
yfilllX + y'cosx
dx (c) 11r x sin x
o 1 + cos 2 x x
d

(d) (e) la'!, sin 2 x dx (f) In", 1 -


sin 2x dx
o 1 + sin 2x

Question 3 Prove that


{ af( x) dx = [ f( x ) + f(- x ) dx.

Question 4 Use the identity from the previous question to calculat e the following.

(a) /_
-1
1 x2
- - dx
1 + eX
(b)
~ ex sin 2
/__.zi:_ -
2
-- dx
1 + eX
x
(c) t l
-¾ 1 + sin x
dx
172 Chapter 4: Further Integration

Question 5 Show that


l 1
l o
x (1 - x t dx = ( ) (
n+l n+2
)"

Question 6 Prove that


¾1 a 1
1 ~dx=-1 ~dx
and state what familiar logarithm law comes from this .

Question 7 Prove that


l
loo f (ax) dx = -a
1 la
o
f (x ) dx.

Question 8
(a) Prove that

t f(a+b-x)dx= t f(x)dx

(b) Hence, evaluate


hjcj l+ ~ dx.
Question 9

(a) Prove that foa f(a - x) dx = foa f(x) dx.


(b) A function f(x) has the property that f(x) + f(a - x) = f(a). Prove that
a a
l
o
J(x) dx = - f (a).
2
Question 10

(a) Prove that foa f(a - x) dx = foa f(x) dx.


(b) Hence, evaluate
foa In (1 + tan a tanx) dx
7f 7f
for - - <a<-.
2 2

Question 11

(a) Prove that foa f(a - x) dx = foa f(x) dx.


(b) Prove that foa f(a + x) dx = 1a 2
f(x) dx.

(c) Hence, evaluate

lo ½sin - (12 - x) + sin - (12 + x) dx.


l l

MASTER ING MATHEMATICS


41 Further Substitutions 173

0 Challenge Problems - - - - - - - - - - - - - - - - - - - - - - - - - - - - - - - - - - - - - - - 1

Problem 1 Use the identities and techniques from this exercise to evaluate the following.

(a) fo'I ln (1 + tanx) dx (b) lo'"' ln(sin x) dx


~ 1 1r X
(c)
j_'!, (1 + tan 4 x) (1 + 1rx) dx
(d)
fo 0 a+
b 2
COS X
dx for a , b > 0.

Problem 2 Let O < 0 < 1r and n E R Evaluate the following integral.

oo tan- 1 (xn)
-2 - - - - - dx
foo x + 2x cos 0 + 1

Problem 3 Suppose a> 0 is a constant.


(a) Prove that

fa f (-Wax) dx = fa f (X) dx.


lo 1+x 2
lo 1 + x2

l ln(l + x)
(b) Hence, evaluate dx.
foo l+x 2

Problem 4 Suppose that f(x) is an even function and a> 0 is a constant.


(a) Prove that
dx = fa f(x) dx.
J_-aa 1f(x)x
+e lo

l (1 x)2
(b) Hence, evaluate - e dx.
fo
0 (1 + eX) 3

Problem 5 Let <p be the positive solution to x 2 = x + 1, and n be a positive integer.


Define the integral
n1r X
In= foo - ---dx.
<p - cos 2 x

n1r fon1r 1
(a) Show that In =- dx.
2 O <p - COS 2 X

n21r
(b) Show that In = - - Ji.
2
(c) Hence, evaluate In.
I
0 Review

Question 1 Mixed problems ( easy)

(a) / cos 3 x dx (b) / 2x tan - l (x ) dx (c) / x✓f+xdx


1
(d)
J _ x_ dx
1 - x4
(e) la'!, ex sin x dx (f)
/ 1 - sin x
dx

2
+x +3 d
(g) 1
/ x 2 - 6x + 13
d
x
(h) 2
/ ln Ix - 1 I dx (i)
J x
(X - l) (x 2 + 4) X

Question 2 Mixed problems (medium)

(a) / sin xdx


4
(b) j ✓1 + x 2 dx (c) j cosvxdx
1
(d) sin2x dx
/ 1 + sin 2 x
(e)
/
- - dx
ex+ l
(f)
J _4_x_-_3_ dx
x 2 + 2x + 5

(g)
lo
(~
3+
l
s cos e
d0 (h)
/ l-fi
1 d
x
(i)
J x2
J9-x 2
dx

Question 3 Mixed problems (hard)


1 2 1
(a) / dx (b) / x dx (c) dx
25 cos x + 4 sin 2 x
2 (x 2 + l)(x 2 - 2) / sinx - cosx
1 1 d 1 d
(d) --d x (e) (f)
/ X + x6 / fi+ijxx / e2x - ex - 2 x

(g) j J l + sin x cot x dx (h)


Jv~ rx-dx (i)

Question 4 Mixed problems ( very hard)

(a)/ 1 d (b) (lln(l+x)dx (c) (i l d


xfi + VX x lo 1 + x2 l I!-
6
tan x + cot x x
2
x 2 -1
(d) /
(
~)
1 - ex dx (e) / ln(l: lnx) dx (f)
J -4 - - - - dx
x + 3x 2 + 1
(~ a + bsinx dx 1
(g)
lo (b + asinx) 2
(h)
/ cos xJsin 2x
dx (i) / ln (x + ~ ) dx
Question 5 Mixed problems ( competition-level elementary techniques)

J---
1 sinx dx
(a) - - dx (b) (c)
(1 - x )vT"=x2 / sinx + cos x

J___
00
ln x
(d) x_2___ dx
( X sin X + cos X) 2
(e) (oo __x__ dx
lo ~
(f)
1 --d
o 1 + x2
x
Question 6 Evaluate the following definite integrals.

(a)
jl - - dx x2
(b) lo1x3 /I=x2 dx (c) !/'
1

- 1 1 + x6 0 1 - 1y'x d X

(d) fo'f tan 4


x dx (e) foi cos 5
x dx (f) foi sin 4
xdx

1J4 -
(g) fo x
2
dx (h)
t~
0
--dx
4- X
(i)
lo (1 + x2)2
x2
3 dx

(j) lo x2 - x + l
X dx (k) Jo
-1
x +l
x2 + x + l
dx (1)
f2 ✓ -8 + 6x - x 2
X
dx

(m) t1 (2x + 1) (X + 2)
l dx (n)
12 x2 - 1
o (x + l) (x 2 + 4)
dx (o)
11 x2
o (x - 2) 2(x+l)
+ 2x dx

(p)
t0
1
2 + cosx
dx (q) Jn 1 - cos x1 + sin x dx
E.
(r)
t 1
o 4 cos x + 3 sin x
dx
2

le lo1x tan - 1(x) dx


1

(s) x
2
ln xdx (t) fo 2
sin- 1 (2x) dx (u)

Question 7
oo 1
(a) Simplify (x + l)(x 2 - x + l). (b) Hence, evaluate
lO
- 3- - dx.
X + l

Question 8 Define for n 2: 0


In = J( ln x f dx.

(a) For n 2: 2, show that


In= x (ln xf - nin-l·

(b) Hence, find h.


j I ~ -.

Question 9 Define for n 2: 0


In = J cosn x dx.

(a) For n 2: 2, show that


1 n-l . n- l
In= - cos xsmx + --In-2 ·
n n
(b) Hence, find fs.

Question 10 Define for n 2: 0

(a) For n 2: 1, show that


2n
In = - - - - In-l·
2n+3
(b) Hence, evaluate h-
176 Chapter 4: Further Integration

Question 11 Define for n ~ 0

(a) For n ~ 2, show that


n- l
In = - - In-2·
n + 11
(b) Hence, evaluate Is

Question 12

(a) Prove that


la" f (x ) dx = la" f (a - x) dx.

1
(b) Hence, evaluate fo x2~ dx.

Question 13

(a) Prove that


{a f(x) dx = la" f(x) + J(-x) dx.
~ ex cos x
(b) Hence, evaluate /_ - - - dx.
_ _Z!: 1 + eX
2

Question 14

(a) Prove that

(b) Hence, evaluate h :


a
. ( -1 -
a Sln X )
dx.

Question 15

(a) Prove that


00
f (¼) loo J(x)
1o - 2 - dx =
x +1
- - dx
o x2 + 1 ·
oo l
(b) Hence, evaluate
lO
(
X
2
+l
)2 dx.

·I MASTERING MATHEMATICS
Chapter 4: Investigation Task 177

q Investigation Task

Further Applications of Integration

In the RSC course, we learn how to use integration to calculate area and volume. Another thing
that can be calculated using integration is arc length. This investigation task will allow students
to explore and get some hands-on experience with calculating arc length.

Question 1 Write a two-page article about arc length. Your response should include answers
the following questions.

(a) State the formula for the arc length of a Cartesian equation, and prove it. Give an example.
(b) How do you find the arc length of parametrically defined curves? Give examples and include
a verification that the circumference of a circle is 1rr 2 .

(c) What assumptions are made in the derivation of the arc length formula?

Question 2 Write a two-page article about polar curves. Your response should include answers
the following questions.

(a) How are such curves defined?


(b) Why do we need this method of defining curves?
(c) What are some examples of polar curves?
(d) What does the equation of a standard curve like a line or parabola look like when it is
defined using polar coordinates instead of Cartesian coordinates?

(e) How do you find the arc length of such curves? Give examples including the cardioid.
(f) How do you find the area inside such curves? Give examples including the cardioid.
(g) What is a real-life example of arc length and/or area inside a curve defined using polar
coordinates?

Question 3 Write a one-page article about line integrals. Your response should include answers
the following questions.

(a) What is a line integral?


(b) How is it related to 'area under a curve' that we learn in the RSC course?
(c) Give an example of a calculation.
(d) Why does orientation matter?
(e) What is a real-life example or application of a line integral?
178 Chapter 4: Further Integration

q Investigation Task
,. ' •: •. :"'I 1t
1
... it ... ~
~ 'I.'-,·,•.. ..... r- ,. ..... ~..... .> ' ~ ~·4

I ~- ~ • , • i.£ ~ .;C°'\°' · •·

"}ti•f- l
~- leli Silbstitat"ion' ~· '. ; .\ 't) ~~/ .-:
i.,,••,~• •'l-:'11~ I ,J

'L. :ri.;1..,. l. .,_<- )_':,,· .._.v

In this chapter, we learn many integration techniques such as integration by parts and partial
fractions. Another very useful technique that was mentioned in Exercise 2A Problem 1 is called
the Euler substitution. _:~1::{~~~. .,.~.-·,~,., ':~~l ~}7.~':f~ -r:\:•,. ~;~> :.~~<.~ r·•i~:::;•~:,,:t~>,::~ -~;·, --~. 'i>'-1.'.· '.~~- •', _",,. '
Write a three-page article on Euler substitutions. It should contain theory, proofs, a discussion
on any special cases, and a number of questions and answers. Your response should also include
a discussion of the following. ; ,..' ir.,. . " .. ti 1•. . , : -· •. , , • .._,t•_ . • . , •• ,;,, • , o ,...
1 • ••
~ ! .l~ ~"f.-ft" -.1•• tt "f'' it t' ~ ~• ~ j" ',! ••, ': ••, f ' • ": • - <" ';: 1: " IFJ •, l ...1/, .., J•• t .,. -{

., : · ,. What are tbe different substitutions and for what scenarios do you use which?
•, I' •....,._..,,. -

1l ,'

..• What is the intuition behind using it?


..
l
......... :.-.'
. Give an example of an integral done using an Euler substitution, with the same integral
done without it .

• I
Your lesson should be sufficiently detailed so that a typical Extension 2 student will be able to
independently perform an Euler substitution to solve a problem.

MASTERING MATHEMATICS
Chapter 4: Investigation Task 179

q Investigation Task
' .
,, 1•'

~ ~.:. ,,~ . .•
• \J

\· ,_i: "'-

~ ifferentiati ng u~dE!r.the ;~te~ra ,,


' ~ '"1 : I; ..~ ;.. . \,1
. . ·;: :, /'..· .:,,:;, e,
I!,... I 1: I ".I: : J.~ ..
:;
r,:· ,.(j, ,. ' ( ,: ' ~ll

A lesser-known trick in integration is to differentiate under the integral. Although not well-known, :·
it is a very powerful tool that can be used to make quick work of integrals that are otherwise
inaccessible using standard techniques. ::: _, ,:·:-- ,'t-;·-.: '/':·~;::~..~;\ :.< __ ,:·( '/-·: , ,, ~-/ <'."': ·.~,:,•?.;.t .;(;~:./)
Give a 5-10 minute presentation on this trick that is targeted towards an audience of Extension
2 students. Your answer should include the following. ~,.~:-:.•'it:,_·,.,.,.:\,, /'!:j;< •.~,,- -:. , .. _, --~~·,_:--:~_, ·-,~~" i~}_: y-! 1

• What is the difference between differentiating under the integral, and what is commonly
referred to as Feynman's Trick? , , ,. , , . - r , ; _, ,
'r. l T•

• Worked examples using the trick, and the same problem without using it.

• How is the Fundamental Theorem of Calculus relevant to this trick?

Your presentation should be sufficiently detailed so that a typical Extension 2 student watching
it will be able to solve simple problems independently.
180 Chapter 4: Further Integration

q Investigation Task

Bunch of Integrals

At the bottom of the first page of the chapter review is a section on 'competition-level elementary
techniques ' integral problems. Do them all and show full working out.

Question 1

(a) / 1 d
(1 - x ) ~ X
(b) lo~Jtanxdx (c) j sinxsinx
+
dx
cosx

(d) j (X sin X
x
2

+ cos X) 2
dx (e) lo~ov0-=-I
X dx (f) fooo -lnx
-dx
o 1 + x2
Many of these integrals have multiple solutions, so bonus points for alternative methods!

MASTERING MATHEMATICS
~--Resistea Horizontal Motion
Resisted Vertical Mo~ r:1~

Resisted Pa:ojectile Motion

Inclined Planes a
182 Chapter 5: Mechanics

Exercise SA
Velocity-Displacement Equations

~ Fundamentals

Fundamentals 1
Complete the following.
dv
(a) a= -

dx dv
(c) V=- (d) a=v-

Fundamentals 2
Prove that

Fundamentals 3
Describe the steps to obtain an expression for x in terms oft given the following.

(a) v = J(x) (b) a= J(v) (c) a=f(x)

Note: Unless stated otherwise, all distance units are in metres and all time units are in seconds.

Question 1 A particle is initially at the origin and moves such that

Find the following as functions of time t.


(a) X (b) V (c) a

Question 2 A particle is initially at rest at x = l and moves such that


1
a=---.
2x +1
Find v 2 as a function of x.

Question 3 A particle moves with a constant acceleration of 8 m s- 2 . When the particle is 5 metres
to the right of the origin, it has a velocity of 12 m s- 1 . Initially, the particle is at the origin.
(a) Find an expression for velocity in terms of displacement.
(b) Hence, find an expression for displacement in terms of time.

MASTERING MATHEMATICS
SA Velocity-Displacement Equations 183

Question 4 The velocity of a particle moving in a straight line is given by

V = 3x + 1,
where x is the displacement from the origin.
(a) What is the acceleration of the particle when it is 2 metres to the right of the origin?
(b) If the particle is initially at the origin, when does it reach x = 27

Question 5 A particle moves along the x-axis with velocity given by

V = -2x 2 .
Initially, the particle is 2 metres to the right of the origin. Find an expression for
(a) a in terms of x. (b) x in terms oft.

Question 6 The acceleration of a particle is given by

.. 9
x= - - - -3
(x + 2) ·

3
Initially, the particle is at the origin with a velocity of v = m s- 1 .
2
(a) Express v in terms of x. (b) Express t in terms of x.

Question 7 A particle moves in a straight line with acceleration


x3
x=--.
2
The particle is initially at rest at x = 2.
1
(a) Prove that v 2 = x 4 ).
4(16 -
(b) Find the value of the greatest speed, and where this occurs.
(c) Describe the motion of the particle.

Question 8 The acceleration of a particle is

x= 2x - 6.

Initially, the particle is at rest at x = 4.


(a) Show that v 2
= 2(x 2
- 6x + 8).
(b) In which direction does the particle begin to move? Explain your answer.
(c) Show that the particle does not pass through the point x = 3.
(d) Sketch v 2 against x , and hence explain why x 2:: 4.
(e) Describe the motion of the particle.
184 Chapter 5: Mechanics·

Question 9 A particle has velocity

v2 = (4x 2 - l)(x 2 - 9).

Initially, the particle is at the origin with a velocity of 3 m s- 1 . Show that at all times , the particle
1
lies within the interval lxl ::; .
2
Question 10 The acceleration of a particle is given by

··
X = --l e -4x
2
1
Initially, the particle is at the origin with a velocity of v = m s- 1 .
2
1
(a) Show that v = - e- 2x. (b) Find x as a function oft.
2
(c) What happens to x and v as t -+ oo? (d) Describe the motion of the particle.

Question 11 A particle has an acceleration-displacement equation given by

It is initially at the origin and moving to the right with a speed of 4 m s- 1 .

(a) Find v as a function of x. (b) Describe the particle's eventual behaviour.

Question 12 A particle moves with equation of motion given by

a= v 2 + 1,

and it is initially at rest at the origin.

(a) Show that v = tan t. (b) Show that x = -lnl cost!.


1
2 ln(l + v
(c) Show that x = 2
). (d) Show that a= e2 x.

Question 13 A particle moves with velocity-displacement equation

V = 2- X,

and it is initially at the origin.


(a) In which direction does the particle travel initially?
(b) Find a as a function of x.
(c) Find x as a function of t.
(d) Hence, describe the behaviour of the particle as t-+ oo.

MASTERING MATHEMAT ICS


SA Velocity-Displacement Equations 185

Question 14 A particle has displacement-time equation

Show that a = 4(x - 4) using two different methods.

Question 15 A particle has acceleration equation

and is initially at the origin with a velocity of 2\/'2 m s- 1 .

(b) Find x as a function oft.

0 Challenge Problems

Problem 1 [Escape velocity]

A particle is fired from the Earth's surface with initial velocity u, and the acceleration of the
particle is given by
k
a = -2 ,
X

where x is the distance of the particle from the centre of the Earth and k E ffi.+. Let R be the
radius of the Earth.

(a) If acceleration due to gravity has a magnitude of gm s- 2 on the Earth's surface, show that

gR2
a= --2-·
X

(b) Show that


v 2= u 2- 2gR 2(1R - ;;1) .
(c) Show that if u 2 = 2gR, then v #- 0.
(d) Explain the significance of this result.

Problem 2 [General formula for simple harmonic motion]

A particle is initially at the origin and moves to the right with acceleration equation

..
X = - n2 (X -
)
Xo.

By solving the differential equation, prove that the displacement-time equation is

x = Acos(nt + a) + xo ,

where A , a , xo and n are constants.


186 Chapter 5: Mechanics

Exercise 58
Simple Harmonic Motion

~ Fundamentals

Fundamentals 1
The differential equation for simple harmonic motion centred about x = x 0 is

x= -----

Fundamentals 2
A particle moves according to the displacement-time equation

x = A cos (nt +a)+ xo.

Describe how each of the parameters A, n, a, and x 0 affect the motion of the particle.

Fundamentals 3
(a) A particle moving in simple harmonic motion about the origin satisfies

v2= - - - - - -

(b) A particle moving in simple harmonic motion about the point x = x 0 satisfies

v2= - - - - - - -

Fundamentals 4
The formula for period is T = _ _ .

Fundamentals 5
A particle moves about the origin O with amplitude A. Write down the position(s) of the particle
when it attains

(a) maximum speed. (b) zero speed.


(c) maximum acceleration. (d) zero acceleration.

MASTERING MATHEMATICS
58 Simple Harmonic Motion 187

Question 1 [Picking an appropriat e model]

State whether you would use a sine or cosine equation to model the following scenarios. Also, state
whether you would pick the positive or negative version of the function you chose. Assume that the
centre of motion is the origin x = 0.
(a) A particle is initially at the origin and then moves to the left.
(b) A particle is initially at the origin a nd then moves to the right.
(c) A particle is initially at rest at x = 5.
(d) A particle is initially at rest at x = -3.

Question 2 A particle moves according to the displacement-time equation

x = 3 sin ( 2t + i) + 1.

Find the
(a) amplitude. (b) period. (c) centre of motion.
(d) range of motion. (e) maximum speed. (f) maximum acceleration.
(g) initial position. (h) initial speed. (i) initial acceleration.

Question 3 A particle moves according to the displacement-time equation


2
X = 2 COS ( 3t + ; ) - 1.

Find the first two times when the particle


(a) passes through the origin. (b) passes through the positive endpoint.
(c) has velocity 3ms- 1 . (d) has maximum speed.

Question 4 A particle moves according to t he displacement-time equation

x = 6 - 4 sin ( 2t - i) .
Find the velocity of the particle when it passes through
(a) x =2. (b) x=lO .
(c) x = 6 for the first time. (d) x = 4 for the second time.

Question 5 For each of the following , verify that the motion is simple harmonic motion by showing
that they satisfy the differential equation
..
X = - n
2(
X -
)
Xo '

for constants n and xo .


(a) X = 2 COS 3t (b) x = 2 sin 2t + 3 cos 2t - 1
(c) v 2 = 16 - 4x 2 (d) v 2 = 27 + 18x - 9x 2
188 Chapter 5: Mechanics

Question 6 A particle moves in simple harmonic motion about the origin with a period of 10
seconds. It is initially at the origin and moves to the right. In what direction is the particle moving

(a) 8 seconds later? (b) 15 seconds later? (c) 24 seconds later?

Question 7 [Be careful!]

Find the amplitude, period and centre of motion of the following particles.

(a) x = sin 2 t (b) x = cos 2 t


(c) x = 3 + 2 cos 2 t (d) x = 2 - 3 sin 2 2t

Question 8 [Using auxiliary-angle formulae]

Find the amplitude, period and centre of motion of the following particles.

(a ) x = 3 cos 2t + 4 sin 2t (b) x = 4 sin 4t - 3 cos 4t


(c) x = 12cos3t + 5sin3t + 7 (d) x = cos6t - sin6t + 1

Question 9 The motion of a particle is modelled by

x(t) = 3 cos ( 2t + i) - 3.
Sketch the following for t ~ 0.

(a) x(t) (b) x(t) (c) x(t)

Question 10 [Tide problem]

The tide at a harbour can be modelled using simple harmonic motion. At the harbour, high tide is
12 metres and low tide is 2 metres. It takes 4 hours to go from low tide to high tide. Initially at 2am,
it is at low tide. Let t be measured in hours.

(a) Show that x = 7 - 5cos C:).


(b) A ship needs at least 5 metres of water in order to safely enter the harbour. Find the earliest
time that the ship may enter the harbour.

(c) Find the latest time that the ship may leave the harbour after entering at the earliest time.

MASTERING MATHEMATICS
58 Simple Harmonic Motion 189

Question 11 A particle moves in simple harmonic motion with acceleration equation

x= -9x.

When the particle is at x = l , it has a speed of 4 m s- 1 .


(a) Show that the velocity is given by

(b) Find the speed of the particle when it is at the origin.


(c) Find the amplitude and period of the motion.
(d) Write down a possible displacement-time equation.

Question 12 A particle moves with displacement-time equation x = 3 sin(21rt).


(a) Prove that the motion is simple harmonic motion.
(b) Find the initial velocity and acceleration of the particle.
(c) Find the maximum speed and acceleration of the particle.
(d) How far will the particle travel in the first 0.5 seconds of motion, and at what speed is it travelling
at this point in time?

Question 13 A particle moves in simple harmonic motion. W hen it passes through the centre of
motion x = 0, it has a speed of 3-/2 m s- 1 .
(a) Find the amplitude in terms of n.
(b) Find the speed of the particle when it is halfway between the centre of motion and maximum
displacement.

Question 14 A particle moves in a straight line so that the velocity is given by

v
2
= 4(5 - x )(x + 1).
Initially, the particle is at x = 5.
(a) Show that the motion is simple harmonic motion.
(b) Find the amplitude, period, and centre of motion.
31r
(c) How far will the particle travel in the first seconds of motion, and find its position then.
4

Question 15 A particle moves in a straight line so that the acceleration is given by x= n 2 ( 4 - x) ,


where n is a constant. The particle initially moves from rest at x = 0.
(a) Show that v 2 = n 2 (8x - x 2 ) .
(b) Hence, show that the particle never moves outside of a certain interval.
190 Chapter 5: Mechanics

Question 16 A particle moves in simple harmonic motion. The velocity of the particle is respectively
3 m s- 1 and Jg m s- 1 at distances J3 and 2 metres away from the centre of motion. Find the amplitude
and period of motion.

Question 17 Two particles move in simple harmonic motion with displacement-time equations

X1 = 3 COS 2t
x2 = 5 + 3 sin 2t

Find the shortest distance between the two particles, and find the time when this first occurs.

Question 18 A particle moves in simple harmonic motion with endpoints x = 2 and x = 14. It has
a maximum speed of 18 m s- 1 . Find the amplitude and period of the motion.

Question 19 A particle moves in simple harmonic motion with period 0.5 seconds and maximum
speed 4 m s- 1 . Find the amplitude of the motion.

Question 20 A particle moves in simple harmonic motion with displacement-time equation

(a ) Expand (a+ b)(a 2 - ab + b2 ).


(b) Find the centre, amplitude and period of the particle.

Question 21 A particle satisfies the displacement time equation

x = Acos
2
Ct) + Bsin (~t), 2

where O < A < B.

(a) Show that


x (A+B)
= - - + (A-B)
- - cos(nt).
2 2

(b) Hence, show that A ::; x ::; B.

Question 22 A particle moves in simple harmonic motion given by the model

x = Asinnt.

Initially, the particle is at the origin and moves towards the positive extremity. Bob claims that when
1 1
the particle has displacement k of the positive extremity, the velocity is also k of the maximum speed,
where k > l.
(a) Use a counter-example to demonstrate that this statement is not always true.
(b) Find the value of k for which the statement is true.

MASTERING MATHEMATICS
5B Simple Harmonic Motion 191

I
0 Challenge Problems - - - - - - - - - - - - - - - - - - - - - - - - - - - - - - - - - - - - - - - 1

Problem 1 A particle moves in simple harmonic motion about the origin 0. When it is p
units from 0, it has speed u. Similarly, when it is q units from 0, it has speed w. Prove that

(a) (b) the amplitude is A=

Problem 2 [Total mechanical energy]

The force equation for simple harmonic motion is

Fnet = -kx(t),
where k E ffi. and x(t) is the distance of the particle from the centre of motion at time t.

The kinetic energy of a particle is given by

where m is the mass of the object and v(t) is the velocity of the object at time t.

The potential energy of a particle is given by

(a) Show that x(t) = A cos ( ff,; t) satisfies the force equation for simple harmonic motion.
(b) Hence, show that the total mechanical energy

M(t) = K(t) + P(t)


depends only on the amplitude of the motion.

Problem 3 Two particles P and Q move in simple harmonic motion such that

xp = A cos(nt + a)
xq = B cos(nt + /3)
A third particle moves with displacement-time equation x = xp + XQ· Prove that the third
particle also moves in simple harmonic motion with amplitude R satisfying

R 2 = A2 + B 2 + 2AB cos(a - (3).


192 Chapter 5: Mechanics

Exercise SC
Projectile Motion

&fu Fundamentals
Fundamentals 1
A particle is projected from the origin with initial velocity V m s- 1 and initial angle 0. Let gm s- 2
be the acceleration due to gravity.
(a) Write down the standard time-equations of motion in vector form.
(b) Prove that the equation of the trajectory is

. gx2
y = xtan0 - 2v2 (1+tan 2 0).

Fundamentals 2
Consider a particle projected from the origin with initial angle 0 and initial speed V m s- 1 . Find
the following in terms of V, 0 and g.

(a) Time of flight. (b) Maximum height.


(c) Horizontal range. (d) Maximum range.

Note from the author: Although the above results are not to be memorised, they are the bread-
and-butter of projectile motion 'proof' questions, so their derivations should be practised heavily.

MASTERING MATHEMATICS
5C Projectile Motion 193

Question 1 A projectile is launched from the origin with fixed speed V m s- 1 and some angle 0.

0 X

V 2 sin 20
(a) Prove that the horizontal range of the particle is - -- -
g

(b) When the particle is aimed at an angle of 15°, it travels a horizontal distance of 40 metres. Show
that V 2 = 80g.

(c) Show that the equation of the trajectory is

x 2 sec 2 0
y = x tan 0 - - - - -
160

(d) The particle is now aimed so that it hits an object 40 metres away horizontally at a height of 20
metres. Find the angles of projection required to hit the particle, rounded to the nearest degree.

Question 2 A particle is projected from the origin with initial speed V m s- 1 and initial angle 0.
The particle has height hat x = x1 and x = x2.

y
V

0 X

Show that
194 Chapter 5: Mechanics

Question 3 [The famous 'Monkey and Hunter' problem]

A zoo-keeper aims a tranquilliser dart with some initial velocity V m s- 1 at an escaped monkey that
hangs from a tree d metres away horizontally and h metres high. The initial velocity is always enough
so that the dart will at least make the tree. The moment the zoo-keeper fires the dart, the monkey
lets go of the tree and falls vertically downwards.

0 d
X

Prove that regardless of V, the dart will always hit the monkey.

Question 4 A particle is projected with variable initial angle 0 and fixed velocity V m s- 1 at a pole
d metres away horizontally. It hits the pole at a height of h metres.

(d , h)
V

0 d
X

You may assume that the equation of the trajectory is


gx2
y = - V 2 sec 2 0 + X tan 0.
2
v2
(a) Show that his maximised when tan 0 = - .
gd
(b) Prove that the maximum height at which the particle can hit the pole is given by

V4 _ g2d2
h=----
2gV2

(C) Deduce that V 2 > gd.

MASTERING MATHEMATICS
5C Projectile Motion 195

Question 5 A particle is fired from the ground with initial angle 0 and initial velocity V m s- 1 .

0 X

When the particle has height h, the speed of the particle is S.

(a) Prove that when the particle has height h, the vertical speed satisfies ii = V 2 sin 2
0 - 2gh.
1
(b) Deduce that h = -(V 2 - S 2 ).
2g

Question 6 A pole has an angle of elevation of 0 from the projection point of two cannons. Two
projectiles are fired with initial angles of a and /3, and initial speeds of u and w respectively.

0 X

The projectiles hit the pole simultaneously at the base of the pole and at the top of the pole.

(a) Show that u cos a = w cos /3.


(b) Hence, prove that tan 0 = tan a - tan /3.
196 Chapter 5: Mechanics

Question 7 A particle is launched from the base O of a plane inclined at an angle of a from the
horizontal plane.

X
0 1-----d-----

Initially, the particle has a speed of V m s- 1 and an angle of inclination of 0. You may assume the
standard equations of motion in terms of time.
gx2
(a) Show that the equation of the trajectory is y = x tan 0 - V 2 sec 2 0.
2
(b) Show that when the particle hits the ramp , it has travelled a horizontal distance of

d = 2V 2 cos 0 sin (0 - a) .
gcosa

(c) Hence, show that the range of the particle up the inclined plane is

2V 2 cos 0 sin(0 - a)
R = - -- - - ---
g cos 2 a ·

(d) Prove that the range R up the ramp is maximised when the angle of projection is halfway
between the vertical and the angle of the plane.
1
(e) Let T be the time of flight when this occurs. Show that R = gT 2 .
2

Question 8 A particle is projected from the origin with initial speed V m s- 1 and initial angle 0.
The particle passes through the point P(p , q), and has a horizontal range of R.

Show that
qR
tan0 = p(R - p)"

MASTERING MATHEMATICS
SC Projectile Motion 197

Question 9 A particle is projected from the origin with initial speed V m s- 1 and initial angle 0.
Assume that acceleration due to gravity is gm s- 2 .
v2
(a) Show that the maximum possible height is h =- .
2g
(b) Show that the equation of the trajectory can be expressed as

x 2 tan 2 0 - 4hx tan 0 + (4hy + x2) = 0.

(c) Show that the point (X, Y) where X #- 0 can be hit using two projection angles 01 and 02 if

X 2 < 4h(h - Y).

(d) Deduce that no point above the x-axis can be hit using two different projection angles 01 and 02
7f

4.
if both are less than

Question 10 A projectile is fired from the top of ah-metre tall building with a fixed muzzle velocity
of Vms- 1 and initial angle 0.

I
h

1 f - - - -- - - d -----------i

Let d be the horizontal distance of the particle from the building base when it lands on the ground.
(a) Show that

(b) Let D be the maximum horizontal distance of the particle from the base of the building , and let
v2
a be the angle when this occurs. Show t hat tan a = - .
gD
(c) Show that
D = V
g
Jv 2 + 2gh.

h
(d) Deduce that cot 2a = D.
198 Chapter 5: Mechanics

Question 11 [Application of the sum and product of roots]

A particle is projected from the origin with initial speed V m s- 1 and initial angle 0.

y
V
h

h
2

0 X

There are two points x 1 and x 2 where the particle's vertical height is half of the maximum height h.
Let gm s- 2 be the acceleration due to gravity. You may assume that the Cartesian equation of the
trajectory is
gx2 2
y = xtan0 - 2v2 (1+tan 0).

(a) Show that x 1 and x2 are solutions of the quadratic

R
(b) Show that the horizontal distance between x 1 and x2 is vl2, where R is the horizontal range of
the particle.

Question 12 A particle is projected from a point h metres above the ground with initial speed
V m s- 1 and initial angle 0. Assume that acceleration due to gravity is gm s- 2.

y V

1
h

X
0 i+--- - - - - R - - - - - - - - - d -

The particle clears a h- metre tall fence R metres away, and lands at a point d metres away from the
base of the fence. Show that
hR
tan 0 2: d( d + R)

MASTERING MATHEMATICS
SC Projectile Motion 199

Question 13 A particle is projected from the ongm with initial speed V m s- 1 . Assume that
acceleration due to gravity is gm s- 2 . When the particle is projected with angle 0, it attains a
horizontal range R. When it is projected with angle a , it falls short of R by a metres. Similarly, when
it is projected with angle /3, it overshoots R by b metres.

. y

0 X
R
- - a - - -- - b - ---

v2
Show that a + b = - ( sin 2tJ - sin 2a).
g

Question 14 A particle is projected horizontally from a point h metres above the ground with
initial speed Um s- 1 . Simultaneously, another particle is projected from the ground with initial speed
V m s- 1 and initial angle 0.

V
1
h

0 X

Assume that acceleration due to gravity is gm s- 2 .

(a) Show that if the particles collide, then V > U.


1
Show that if the particles collide on the ground, then V 2 U2 =
(b) -
2gh.
200 Chapter 5: Mechanics

Question 15 A particle is projected from the origin with initial speed V m s- 1 and initial angle 0.
Assume that acceleration due to gravity is gm s- 2 .

0 X
d

When the particle reaches maximum height, it hits a wall d metres away at height h.

(a) Show that 0 = tan- 1 c:). (b) Show that V 2 = :h (d 2 + 4h 2 ).

Question 16 A projectile is launched from a point O on a plane inclined downwards at an angle of


a from the horizontal plane. Initially, the particle has speed Vms- 1 and an angle of 0.

0 X

Prove that the range R down the ramp is maximised when the angle of projection is halfway between
the vertical and the angle of the plane .

. 1- - - - - - - - - - - - - - - - - - - - ~ -
MASTERING MATHEMATICS
5C Projectile Motion 201

0 Challenge Problems

Problem 1 A particle is projected horizontally from a point h metres above the ground with
initial speed V m s- 1 . It hits the ground R metres away from the base of the cliff after travelling
for T seconds. The same point can also be hit using the same initial speed, but with angle of
projection 0 -/=- 0. Assume that acceleration due to gravity is gm s- 2 .

y
V

1
h

0 X
- - - - - - - - -- - - - R - - - - -- - - - ----

1
Show that R = gT 2 tan 0.
2

Problem 2 [Envelope of parabolas]

A hose sprays water with a fixed muzzle velocity of V m s- 1 .

The angle of inclination is allowed to vary from -1r :::; 0 :::; 1r so that anything inside the curve
traced out by the stream can be hit by water.
V 2 sin 20
(a) Prove that the horizontal range is given by - - - -
g
v2
(b) Show that the maximum horizontal range is given by R =- .
g
(c) Prove that anything inside the parabola

x2 + 2Ry - R2 =0
can be hit by water.
202 Chapter 5: Mechanics

Exercise 5D
Resisted Horizontal Motion

&& Fundamentals
Fundamentals 1
Write down at least three expressions for acceleration.

Fundamentals 2
(a) When a particle is projected horizontally through some medium, it experiences a resistive
f _ _ that is a function of the v _ ___ of the particle.

(b) For horizontal motion, we ignore the effects of g _ __ _


(c) Sometimes, the particle may have a limiting displacement and will never exceed this. The
limiting displacement , if one exists, can be found by letting _ approach infinity.

Question 1 A particle with mass 5kg is projected to the right with an initial velocity of 40 m s- 1 ,
and it experiences a resistive force of 20v, where v is the velocity of the particle after t seconds. Let
a be the acceleration of the particle after t seconds.

(a) Show that a= -4v.

(b) Show that v = 40 - 4x.

(c) Show that v = 40c 4 t.

(d) Combine parts (b) and (c) to show that x = 10(1 - e- 4t).
(e) Explain how you would obtain x = 10 ( 1 - e- 4 t) using either parts (b) or (c) only.

(f) Write down the limiting displacement of the particle.

Question 2 A particle with mass 5kg is projected to the right with an initial velocity of 10 m s- 1 ,
and it experiences a resistive force of 20v 2 , where v is the velocity of the particle after t seconds. Let
a be the acceleration of the particle after t seconds.

(a) Show that v = 10e- 4 x_

(b) Hence, show that x = ln(l + 40t).


(c) Find how far the particle travels after 2 seconds.

(d) Find how long it takes for the particle to travel ln 2 metres, and its speed at this time.

MASTERING MATHEMATICS
5D Resisted Horizontal Motion 203

Question 3 A particle with mass 4kg is projected to the right with an init ial velocity of 8 m s- 1 ,
and it experiences a resistive force of kv 2 , where v is the velocity of the particle after t seconds. Let
a be the acceleration of the particle after t seconds. Find the value of k if

(a) after 4 seconds, the particle has a speed of 2 m s- 1 .


(b) the particle has a speed of 2 m s- 1 when it has travelled 4 metres.
(c) aft er 2 seconds, the particle has travelled 8 ln 3 metres.

Question 4 [Generalised linear drag]

A particle with mass m kg is projected to the right with an init ial velocity of um s- 1 , and it experiences
a resistive force of mkv, where v is the velocity of the particle after t seconds . Let a be the acceleration
of the particle after t seconds .

(a) Show that v = u - kx.


(b) Show that v = ue-kt.
(c) Show that x = ku (1 - e-kt ).

(d) Hence st~te the limiting position of the particle.


(e) After how many seconds does the particle reach half of its limiting position?
(f) Use two different methods to show that when this occurs, the particle has half its initial velocity.

Question 5 A particle with mass 10kg is projected to the right with an init ial velocity of 20 m s- 1 ,
and it experiences a resistive force of mkv, where v is the velocity of the particle aft er t seconds. The
limiting displacement is 40 m . Let a be the acceleration of the particle after t seconds.

(a) Find the limiting displacement of the particle in terms of k.


(b) Hence , find the value of k.

Question 6 [Generalised quadratic drag]

A particle with mass m kg is projected to the right with an initial velocity of um s- 1 , and it experiences
a resistive force of mkv 2 , where vis t he velocity of the particle after t seconds. Let a be the acceleration
of the particle after t seconds .
u
(a) Show that v = ue-kx. (b) Show that v = ---
1 + ukt
1
(c) Show that x = k ln(l + ukt). (d) Does this particle have a limiting position?
204 Chapter 5: Mechanics

0 Challenge Problems

Problem 1 A particle with mass m kg is projected to the right with an initial velocity of
um s-1, and it experiences a resistive force of mkv, where v is the velocity of the particle after
t seconds. Let a be the acceleration of the particle after t seconds.

Prove that when the particle is r % of the way to its limiting position, it will be moving at
(100 - r) % of its initial velocity. For example, when the particle is 30% of the way to its
limiting position, it will be moving at 70% of its initial velocity.

Problem 2 A particle with unit mass is projected to the right with an initial velocity of
um s- 1 , and it experiences a resistive force of v + v 3 , where v is the velocity of the particle after
t seconds . Let a be the acceleration of the particle after t seconds.
(a) Show that

X = tan -1 (1u-v)
+
UV .

(b) Show that

(c) State the limiting position of the particle.


7r
(d) Show that the particle can never travel past x = with any initial speed.
2
(e) State the limiting speed of the particle.

Problem 3 Bob claims that if an object is projected horizontally through a resistive medium,
then it must eventually stop, assuming that there are no further forces pushing the object
forwards. Is Bob correct in saying this?

Problem 4 Suppose that two particles with differing mass are projected through the same
resistive medium with linear drag, and with the same initial velocity. Bob claims that the lighter
particle will travel further than the heavier one. Is Bob correct in saying this?

.1 MASTERING MATHEMATICS
SE Resisted Vertical Motion 205

Exercise 5E

Resisted Vertical Motion

~ Fundamentals

Fundamentals 1
Resisted vertical motion could be going up or down. Separate equations will need to be produced
for each direction of motion.
(a) When the particle goes up, weight force is considered positive/negative (circle one) and
resistive force is considered positive/negative (circle one).

(b) When the particle goes down, weight force is considered positive/negative (circle one) and
resistive force is considered positive/negative (circle one).

Fundamentals 2
(a) When an object is released in the air, it experiences a w ____ force and a resistive force.
(b) For a particle falling , we consider the w ____ force to be p ____ since it pulls in
the same direction as our motion. However , we consider the resistive force to be n _ _ __
because it opposes our motion.

Fundamentals 3
(a) When a particle falls vertically through a resistive medium, the speed will approach a limit
called the t ____ velocity.

(b) The t ____ velocity can be found be letting either _ or _ approach infinity.

(c) However, the easiest way to find an expression for it is to let a = _ since this is when the
particle has constant speed .

Question 1 A particle of mass 4kg is dropped from a tall building. It experiences a resistive force
of 20v, where vis the speed of the particle after t seconds, and a gravitational acceleration of 10 m s- 2 .

(a) Show that a= 10 - 5v.

(b) Show that v = 2(1 - e- 5 t) .

(c) Hence, find the terminal velocity of the particle using two different methods.

(d) Find the time that it takes for the particle to reach half of its terminal velocity.

(e) Find the displacement when this occurs.


206 Chapter 5: Mechanics

Question 2 [Generalised upwards motion with linear drag]


A particle of mass m is projected vertically upwards through a resistive medium. It experiences a
resistive force of mkv and a weight force mg. Suppose the particle has an initial velocity of um s- 1 .

(a) Show that velocity is given by

(b) Show that the time taken to reach maximum height is

ku)
9 .
1
T=kln ( 1+

(c) Show that displacement is given by

x= !k [u - v- 2-1n
k
(gg ++ ku)
kv

(d) Show that the maximum height is

u ku)
H = k - kg2 ln ( 1 +
9 .

Question 3 [Generalised downwards motion with linear drag]

A particle of mass m falls vertically downwards through a resistive medium. It experiences a resistive
force of mkv and a weight force mg. The particle has a terminal velocity of w.

(a) Show that w =f.


(b) Show that

x= -¼[ + fIn ( :v)]


v 1-

(c) Hence, show that

x=i [w1n(w:J-v].
(d) Show that
t= !k In ( ~ )
w-v

MASTERING MATHEMATICS
5E Resisted Vertical Motion 207

Question 4 [Generalised upwards motion with quadratic drag]

A particle of mass m is projected vertically upwards through a resistive medium. It experiences a


resistive force of mkv 2 and a weight force mg. Suppose the particle has an initial velocity of um s- 1 .

(a) Show that

x =
1
2k ln
(g ++ kukv2
g
2
)
.

(b) Show that the maximum height is

2
H = 1k ln ( 1 + ku
2
g )
.

(c) Show that

(d) Show that the time taken to reach maximum height is

7f
(e) Show that the particle will never take longer than r::r:. seconds to reach maximum height.
2vgk

Question 5 A particle of mass m is projected vertically upwards through a resistive medium. It


experiences a resistive force of mkv 2 and a weight force mg. Suppose the particle has an initial velocity
of um s- 1 . If the particle is dropped from rest instead, the terminal velocity of the particle is w.

(a) Show that w 2 = f


(b) Show that

x 1
= -ln
2k
(ww 2
+u )
2 + v2
2

(c) At some point in the upwards movement, the particle will have speed w. Show that when this
ln2
occurs , the particle will have to travel k metres further before reaching maximum height.
2
208 Chapter 5: Mechanics

Question 6 [Generalised downwards motion with quadratic drag]

A particle of mass m falls vertically downwards through a resistive medium. It experiences a resistive
force of mkv 2 and a weight force mg. The particle has a terminal velocity of w.

(a) Show that

(b) Show that


g (1
v 2 -_ k - e -2kx) .

(c) Find the terminal velocity of the particle using two different methods.

(d) Show that


2
x = -21k ln ( w 2w- v 2 ) .

(e) Show that


t = _ l_ lnlw + vl
2-Jg!i W - V .

(f) Show that

v=
e 2 tyigk _
w ( e2tylgk + l
1) .

Question 7 A particle of mass m falls vertically downwards through a resistive medium. It experi-
ences a resistive force mkv and a weight force mg. The particle has a terminal velocity of w.

(a) Show that the terminal velocity is V = f


(b) Show that the speed at time t is w(l - e-kt).

(c) An identical particle is projected upwards from the same point of release as the first particle ,
but now with initial velocity u. Show that the second particle reaches maximum height when

uw
(d) Show that when the second particle reaches maximum height, the first particle has speed - - .
u+w

MASTERING MATHEMATICS
SE Resisted Vertical Motion 209

2
Question 8 A particle of mass m kg falls from rest and experiences an air resistance of mkv . Let
the acceleration due to gravity beg m s- 2 and let the terminal velocity of the particle be V.
(a) Show that the equation of motion is x= g- kv 2 .

(b) Find V 2 in terms of k and g.


(c) The particle impacts the ground with velocity W. Show that the distance travelled is given by
2

D = 1 ln (
2k v2 v- w2 ) .

(d) Prove that if the particle is projected from the ground with initial velocity U and air resistance
is still mkv 2 , then the maximum height reached is given by

_~
H - 2k In
+ v2) .
(u2 v2

1 1 1
(e) Show that W 2 = U 2 + V 2.

(f) Show that

(g) Hence , state whether the impact speed is lesser or greater than the initial projection speed.

Question 9 [What happens to simple harmonic motion when sufficiently damped]

A particle of mass m was originally intended to move in simple harmonic motion at time t according
to the displacement equation x = A cos(nt) for positive constants A and n.

However, once the particle was released from rest it experienced a resistance force of 2mnv, where v
is the velocity of the particle.
(a) Explain why the acceleration equation of the particle with the resistance is

(b) Let y = v + nx. Show that y + ny = 0.


(c) Hence, show that the displacement equation of the particle is given by

x = A(l + nt)e-nt.

. d
Hint: Consider the fact that dx (e xf( x)) = ex (f( x ) + J'(x)).

(d) Sketch the displacement equation of the particle and describe its behaviour over time.
210 Chapter 5: Mechanics

0 Challenge Problems ---------- ---------- ---------- --------,


Problem 1 A particle A of unit mass is projected horizontally from the origin through a
medium with initial speed u. It experiences a resistive force of kv 2 , where v is the velocity of
the particle after t seconds .

Simultaneously, a second particle B of unit mass is projected vertically through the same medium
with the same initial speed u. It experiences a resistive force of kw 2 , where w is the velocity of
the particle after t seconds. Let gm s- 2 be the acceleration due to gravity.

(a) Let V be the velocity of particle A when particle B is at rest. Show that if VT is the
terminal velocity of particle B when allowed to fall indefinitely, then

1 1 1 -1 ( u )
V =;,+VT tan VT ·

(b) Deduce that if u is sufficiently large, then Vis approximately 64% of the terminal speed.

Problem 2 A particle of unit mass is projected vertically upwards through a resistive medium
with initial speed u. It experiences a resistive force of kv and a weight force g . When the same
particle lands on the ground, it has speed w. Prove that u > w.

Problem 3 A particle of unit mass is projected vertically upwards through a resistive medium
with initial speed u. It experiences a resistive force of kv and a weight force g. When the same
particle lands on the ground, it has speed w.

u+w
Show that the particle takes in total T = - - seconds to land back on the ground.
g

Problem 4 A particle A is dropped from a weather balloon. Its equation of motion is

where gm s- 2 is the acceleration due to gravity, k is a positive constant and VA


is the speed
of particle A. After T seconds, an identical particle B is projected downwards from the same
balloon with initial velocity um s- 1 . Its equation of motion is similarly

where v B is the speed of particle B. Let w be the terminal velocity of both particles.

(a) Show that T = ! ln ( -1- x w- VA).


k w-u W-VB

(b) Show that particle B's displacement is given by xs = ¼[u - VB+ w In(;--v:) l·
(c) Deduce that if particle B catches up with particle A, then particle B must have been
u
projected no more than - seconds after particle A was dropped.
g

MASTERING MATHEMATICS
SF Resisted Projectile Motion 211

Exercise SF
Resisted Projectile Motion

~ Fundamentals

Fundamentals 1
For each of the following , express your answers in the form x = x(t) and y = y(t).
(a) Write down the acceleration equations for projectile motion assuming no air resistance.
(b) Write down the acceleration equations for projectile motion but now with linear drag,
assuming that the drag constant is the same in both x and y directions.

Fundamentals 2
For each of the following, express your answers in the form f(t), where r(t) is t he position vector
of the particle at time t.

(a) Write down the acceleration equations for projectile motion assuming no air resistance.
(b) Write down the acceleration equations for projectile motion but now with linear drag,
assuming that the drag constant is the same in both x and y directions.

Fundamentals 3
When a particle undergoes projectile motion with air resistance, the motion is no longer
p ____ . Instead, the particle will have a l ____ horizontal displacement that can be
found by letting t approach i _ _ __

Fundamentals 4
The two diagrams below show projectile motion. Label the one that has no air resistance, and
the one that has air resistance.

(a) (b)
y y

X X
212 Chapter 5: Mechanics

Question 1 [Standard derivations for linear drag]

A particle is projected from the origin with initial speed V with an angle of inclination of 0. The particle
is subject to gravity and an air resistance proportional to the velocity such that the accelerations in
the horizontal and vertical directions are given by

x = - kx
y= - g- ky
where k is a constant and gm s- 2 is the acceleration due to gravity. Derive the following results.

(a) x = V e-kt cos 0 (b)

(c) x =Vcos0(
-- - l - e - kt) (d)
k

Question 2 [Some key results for linear drag]

A particle is projected from the origin with initial speed V with an angle of inclination of 0. The
particle is subject to gravity and an air resistance proportional to the velocity such that motion is
modelled by

r = -kr~ -
~
gJ·~'

where k is a constant and gm s- 2 is the acceleration due to gravity.


(a) Show that the limiting horizontal displacement of the particle is

Vcos0
X =---
k

(b) Show that the time taken for the particle to reach maximum height is

1 ln ( 1 + kV sin 0) .
T = k
9

(c) Show that when the particle reaches maximum height , the horizontal displacement is

2
D = V sin20 .
2(g + kV sin 0)

(d) Let a be the angle so that D is maximised. Show that

/5-1
sin a = - - -
2

MASTERING MATHEMATICS
SF Resisted Projectile Motion 213

Question 3 [Quadratic drag does not work in the way you may think it does]

A particle of unit mass is proj ected from t he origin with init ial speed V with an angle of inclination
of 0. The particle is subj ect to gravity and an air resistance with a magnitude proportional to the
square of the velocity, but acting in a direction opposite to the particle's motion.

Bob claims that the particle is modelled by the systems of equations


x = -kx 2
y.. = -g - k y.2

where k E JR+ for the upwards journey and gm s- 2 is the acceleration due to gravity.
1
(a) Show that x = k ln (1 + ukt cos 0).

(b) Assume that gravity is absent. Write down a similar expression for y.
(c) Hence, show that the Cartesian equation of the trajectory is

if the effect of gravity is to be ignored.

(d) Explain why this demonstrates that Bob's model does NOT accurately reflect projectile motion
with quadratic drag.

Question 4 A particle of unit mass is projected horizontally from a building of height h with initial
speed V. The particle is subj ect to gravity and an air resistance proportional to the velocity such that
the accelerations in the horizontal and vertical directions are given by
x = -kx
y= -g - ky
where k is a constant and gm s- 2 is the acceleration due to gravity.

Let the terminal velocity in free fall be w, and let the limiting horizontal displacement be R.

(a) Show that x = : (1 - e-kt) and hence show that k = ~.

(b) Show that y = : ( 1 - e- kt ) - wt + h.


(c) Hence, show that the equation of the trajectory is

y - ln ( 1 - -X) .
= h + -Wx + W
V k R

R
(d) Show that if the particle hits the ground when it is at x = , then
2
h = 3!!_ ( 2 1n 2 -
2k
1).
214 Chapter 5: Mechanics

0 Challenge Problems ----------------------------- ---------,


Problem 1 [The correct model for projectile motion with quadratic drag)

(a) Explain why the correct model of projectile motion with quadratic drag is

f = -g_t - klt lt

where k E JR+ and gm s- 2 is the acceleration due to gravity.


(b) Show that this leads to the systems of equations

x = -k± ✓± 2 + iJ 2
jj = -g - ky ✓x2 + y2

Problem 2 [The Cartesian equation of the trajectory)

A particle of unit mass is projected from the origin with initial speed V with an angle of
inclination of 0. The particle is subject to gravity and an air resistance proportional to the
velocity such that the accelerations in the horizontal and vertical directions are given by

x = -kx
jj = -g - ky
where k is a constant and gm s- 2 is the acceleration due to gravity.

(a) Prove that the Cartesian equation of motion is

y= ( - -g - ) x+-
g ln ( 1 - -kx
- -) .
kV cos0 k2 V cos0

Vcos0
(b) Show that the particle's limiting horizontal displacement is R = k

(c) Show that the particle's terminal speed is w = f·


(d) Let X be the proportion of the particle's horizontal displacement with respect to R. Show
that the Cartesian equation of the trajectory can be expressed as

y = : ( X + ln(l - X)).

MASTERING MATHEMATICS
5G Inclined Planes and Pulleys 215

Exercise 5G
Inclined Planes and Pulleys

~ Fundamentals

Fundamentals 1
The following force diagram shows a normal force N , a tension force T and a weight force mg
acting on an object.

mg

(a) Resolve forces in the horizontal and vertical directions.


(b) Resolve forces in directions that are perpendicular and parallel to the tension force.

Fundamentals 2
(a) If we want to find the acceleration equation of a particle that is travelling in a certain direc-
tion , we should resolve forces in directions P - - - - and P - - -- - to the direction
of motion to make the working out easier.

(b) An i _____ of forces causes acceleration, and hence, movement , so if a particle is


moving in a certain direction , it means there is a n_ force acting in that direction, so t he
acceleration in t hat direction can be studied.

Note from the author: For ramp problems in the following exercise, "Find the acceleration" is
intended to mean "Find the acceleration of the object acting along the direction of the ramp".
216 Chapter 5: Mechanics

Question 1 [Guided smooth ramp problem]


The following diagram shows an object of mass 20kg on a smooth ramp inclined at an angle of 30°
from the horizontal.

200

The particle experiences a weight force and a normal force N. Let g = 10 m s- 2 be the acceleration
due to gravity.

(a) Resolve forces in directions parallel and perpendicular to the plane.


(b) Show that there is a net force parallel to the plane with magnitude 100 Newtons .
(c) Recall that Fnet = ma. Show that the acceleration down the ramp is a = 5 m s- 2 .
(d) Find the speed of the particle if it is released from rest and allowed to travel for 4 seconds .
(e) Find how far the particle slides down the ramp in 4 seconds .

Question 2 [Generalised smooth ramp problem]


The following diagram shows an object of mass m kg on a smooth ramp inclined at an angle of 0 from
the horizontal.

mg

The particle experiences a weight force mg and a normal force N. Let a be the acceleration of the
particle down the ramp. Show that after t seconds,

(a ) a = g sin 0. (b) v = gt sin 0. (c)

'·- - - - - - - - - - - - - - - - - - - - -
1
MASTERING MATHEMATICS
I
SG Inclined Planes and Pulleys 217

Question 3 [Guided ramp problem with friction]

The following diagram shows an object of mass 20kg on a ramp inclined at an angle of 30° from the
horizontal.

N F

200

The particle experiences a weight force, a normal force N , and a friction force F O.lN. Let
g= 10 m s- 2 be the acceleration due to gravity.
(a) Show that there is a net force down the ramp with magnitude (100 - 10-/3) Newtons.
(b) Find the acceleration of the object.

Question 4 [Generalised ramp problem with friction]

The following diagram shows an object of mass m kg on a smooth ramp inclined at an angle of 0 from
the horizontal.

N F

mg

The particle experiences a weight force mg, a normal force N, and a friction force F = µN.
(a) Show that a= sin0 - µcos 0.
(b) Hence, show that if µ ~ tan 0, then the particle will not slide down the ramp.
218 Chapter 5: Mechanics

Question 5 [Guided double smooth ramp problem]

Two objects A and B with masses of 20kg and 10kg respectively are connected by a light inextensible
string that runs through a smooth pulley. The objects lean on a double-sided smooth ramp inclined
at angles of 30° and 60° from the horizontal, as shown below.

The system moves so that particle A slides down the ramp whilst particle B slides up the ramp.

(a) Explain briefly why the magnitude of tension is the same for both particles.
(b) Explain briefly why the magnitude of acceleration is the same for both particles.
(c) Resolve forces for particle A and show that

.. T
X = 5- - .
20

(d) Resolve forces for particle B and find a similar result for i.
(e) Hence, calculate the amount of tension in the string.
(f) Calculate the acceleration of particle A down the ramp.

MASTERING MATHEMATICS
5G Inclined Planes and Pulleys 219

Question 6 [Generalised double smooth ramp problem]

Two objects A and B with masses of m1kg and m2kg respectively are connected by a light inextensible
string that runs through a smooth pulley. The objects lean on a double-sided smooth ramp inclined
at angles of a and /3 from the horizontal, as shown below.

The system moves so that particle A slides down the ramp whilst particle B slides up the ramp.

(a ) Show that
m1 sin a - m2 sin /3)
a= g ( - - -- - - - .
m1 + m2

(b) Hence, show that


T = g (-m1m2 )
- - (sin a+ sin/3).
m1 + m2

(c) Show that if


sin a m2
sin /3 m1
then the system will remain at static equilibrium.
220 Chapter 5: Mechanics

Question 7 [Guided smooth pulley problem]

The following diagram shows two objects with masses of 10kg and 20kg on either end of a light
inextensible string that passes through a smooth pulley.

10 kg

20 kg

The heavier mass moves downwards whilst the lighter mass moves upwards.

T
(a) Show that for the heavier object a= 10 - .
20
T
(b) Show that for the lighter object a = - 10 + - .
10
(c) Explain why both particles have the same magnitude of acceleration at any time t.

(d) Hence , find the acceleration of both masses , and the amount of tension in the sting.

Question 8 [Generalised smooth pulley problem]

The following diagram shows two objects with masses of m1 kg and m2 kg, where m1 > m2, on either
end of a light inextensible string that passes through a smooth pulley.

Show that the acceleration of the heavier particle is

m1 --
a= (- - m2) g.
m1 + m2

MASTERING MATHEMAT ICS


SG Inclined Planes and Pulleys 221

Question 9 The following diagram shows two objects with masses of m kg and 3m kg on either end
of a light inextensible string that passes through a smooth pulley. Both particles are released from
rest simultaneously.

m 3m

Let a be the acceleration of the heavier particle in the downwards direction. Let g be the acceleration
due to gravity.

(a ) Show that a = ~-

1
(b) Hence, show that after 4 seconds, the heavier object travels 4g metres and has speed 2g m s- .

Question 10 [Minimal force problem on a flat surface]

An object of mass m rests on the surface of a table. It is attached to a rope inclined at an angle of 0
from the horizontal that pulls it to the right.

The object experiences a friction force F = µ N that resists the motion of the object.

(a) Resolve forces in the vertical and horizontal directions.

(b) Hence, show that the amount of tension needed to overcome friction is

T= µmg
cos 0 + µ sin0
222 Chapter 5: Mechanics

Question 11 [Minimal force problem involving a ramp]

An object of mass m sits on a ramp inclined at an angle of 0 from the horizontal.

The object is pushed up the ramp and it experiences a friction force F = µN.

(a ) Resolve forces parallel and perpendicular to the plane.

(b) Hence, show that the minimal amount of force needed to push the object up the ramp is

F push = mg(sin 0 +µcos 0).

Question 12 [Minimal force problem involving a ramp and a rope]

An object of mass m sits on a ramp inclined at an angle of 0 from the horizontal.

The object is dragged up the ramp by a rope that is angled a from the surface of the ramp , and it
experiences a friction force F = µN.

(a ) Resolve forces parallel and perpendicular to the plane.

(b) Hence, show that the minimal amount of force needed to pull the object up the ramp is

T = mg ( sin 0 + µ c~s 0 ) .
cos a+ µsma

MASTERING MATHEMATICS
5G Inclined Planes and Pulleys 223

Question 13 The following diagram shows two objects with masses of m1 kg and m2 kg on either
end of a light inextensible string that passes through a smooth pulley. The object with mass m2 is
hung off from the edge of a table whilst the object with mass m1 rests on the surface of a table. The
object on the table experiences a friction force F = µN with the surface of the table.

Prove that ifµ ~ mi, then there is sufficient friction to prevent the object on the table from moving.
m2

Question 14 [Threshold problem]

An object rests on a plane inclined at a variable angle 0 from the horizontal. It experiences a friction
force F and a normal force N. Let a be the maximal angle that the plane can be tilted before the
object begins to slide down the plane. When this occurs, the friction force is F = µN.

N F

mg

A horizontal force H is applied to the object so that if H is sufficiently large, then the particle will
begin to slide up the plane and if H is sufficiently small, then the particle will still slide down the plane.

Show that for the object to remain motionless, H must satisfy

mg tan(0 - a) :s; H :s; mg tan(0 + a).

Question 15 [Glimpse into vector calculus]

Let v = v( t) and a = a( t) be the velocity and acceleration vectors respectively for a particle in motion.
The speed of the particle at any time t is given by lvl , and it is continuously changing as the particle
moves around. Show that
d l
-(lvl) = -(v · a).
dt lvl
224 Chapter 5: Mechanics

Question 16 [Uniform circular motion]

The following diagram shows a particle travelling in a circle with constant angular velocity w = d0
dt,
which is measured in radians per second.

/
/

I
I
\

I I
I

e
_ _ _ _ _ _ _ _ _ _ _j
\
I

\
\

The position of the particle at time t is given by the position vector

r (t) = (r cos wt) i + (r sin wt) j.

(a) Show that the velocity vector is given by

t(t) = (- rwsinwt)i + (rwcost)j.

(b) Show that the acceleration vector is given by

t(t) = (- r 2 wsinwt)i + (-r 2 wcost)j.

(c) Show that at any time t, the linear speed of the particle is v = rw.
(d) Prove that the direction of the velocity vector t( t) is perpendicular to the direction of the position
vector r(t) at any time t.

(e) Show that at any time t , the linear acceleration of the particle is rw 2 .
(f) Show that the direction of the acceleration vector t(t) is opposite to the direction of the position
vector r( t).

(g) The centripetal force is the force that a particle in uniform circular motion continuously experi-
ences in order to keep it in motion.
2
Show that the magnitude of centripetal force is given by mrw 2 or equivalently -mv- .
r

MASTERING MATHEMATICS
SG Inclined Planes and Pulleys 225

0 Challenge Problems

Problem 1 [Application to Kinetic Energy]

The following diagram shows a bead of mass m released from rest at x = l on the curve y == ~ x %.
2

Let the position of the particle at time t be given by r(t) = [:/!\].


y

1
- ----7
X
1

The kinetic energy of a particle at t ime t is given by

The particle also experiences gravitational potential energy

Ep = mgh,
where h is the initial height of the particle from the ground.

(a) The law of conservation of energy states that the total energy of an isolated system remains
constant. Use this to explain why

is a constant function.
1
(b) Show that ± = X3 iJ.
(c) Show that
3 - 2y)
iJ2 = 3g ( 3 + 2y .

(d) Find how long it takes for the particle to fall to the ground.
L _____________________________ _______________________ _
226 Chapter 5: Mechanics

Chapter 5 Review
Mechanics

0 Review

Question 1 A particle has acceleration equation

x = 4x + 2,
and is initially at the origin with velocity 1 m s- 1 .

(a) Show that the velocity is given by i: = 2x + 1.


(b) Find the time taken by the particle to attain a velocity of 16 m s- 1 .

Question 2 A particle has acceleration equation

a= X - 2,

and is initially at rest at x = 3.


(a) Show that the velocity is given by v 2 = (x - l)(x - 3).

(b) In what direction is the motion?

(c) Will the particle ever be at x = 2? Explain your answer.

(d) Find the position and acceleration of the particle when v = 2\/'2.

Question 3 A particle has acceleration equation

1
x=x--
x3'

and is initially at x = 1 with velocity v = 2 m s- 1 .


1
(a) Show that the velocity is i: = x +-.
X

(b) Hence, show that x = J2e 2t - 1.

Question 4 A particle has acceleration equation

x.. = -e -x - e -2x ,

and is initially at the origin with velocity v = 2 m s- 1 .

(a) Show that the velocity is given by i: = 1 + e-x .

(b) Hence, show that x = ln (2et - 1).

MASTERING MATHEMATICS
Chapter 5 Review 227

Question 5 Prove that the following particles are moving in simple harmonic motion.

(a) x = 2 sin t (b) X = 3 COS 21rt (c) x = l - 2 sin3t

(d) x = 4 sin ( 2t + fl (e) x = 4cos 2 t (f) x = 3 cos 2t + 4 sin 2t

(g) x = 2 + 4 cos 3t + 6 sin 3t (h) v2 = 36 - 9x 2 (i) v2 = 128 - 32x - 16x 2

Question 6 A particle moves in simple harmonic motion with displacement-time equation

X = 4 COS ( 2t - fl + 2.

(a) Find the initial position of the particle.


· (b) Find the first time when the particle passes through the origin.

(c) What is the speed of the particle when this occurs?

Question 7 A particle has velocity equation

x= J6 x - x 2 .

(a) Show that the particle is moving in simple harmonic motion.


(b) Find the centre of motion, amplitude and period of the motion.

Question 8 A particle moves in simple harmonic motion and starts from rest at one of the
end-points x = 4. The centre of motion is the origin. If n = 3, find

(a) when the particle first reaches x = -2.


(b) the speed when x = -2.
7r
(c) how far the particle travels in the first seconds.
6
Question 9 The tides at a particular bay rise and fall in simple harmonic motion. Low tide
is at 5am with a depth of 8 metres, while high tide is at 1pm with a depth of 12 metres. A ship
needs at least 9 metres of water to pass through the bay safely. Let t be time in hours from 5am.

(a) Find the amplitude, period of motion, and the value of n.


(b) Write down an equation that models the above scenario.
(c) Hence, find the first range of times after 5am for when the ship can safely pass through the
bay.
228 Chapter 5: Mechanics

Question 10 A projectile is launched from the base O of a road inclined at an angle of a from
the horizontal plane.

0 X

Initially, the particle has a speed of 5 m s- 1 and an angle of inclination of 60°. Assume that
acceleration due to gravity is 10m s- 2 .
4
(a) Show that the path of the trajectory is y = xv'3 - -x 2 .
5
h
(b) Let T( d, h) be the point on the road where the projectile lands. Show that tan a = d.
5( v'3 - tan a)
(c) Let R represent the distance of OT. Show that R = - -- - - .
4cosa

Question 11 A projectile is launched from the base O of a ramp that is inclined downwards
7r
at an angle of from the horizontal plane.
6
y

0 X

7r
Initially, the particle has a speed of 10 m s- 1 and an angle of inclination of . Assume that
2
3
acceleration due to gravity is 10m s- . Find the distance that the particle travels down the ramp.

MASTERING MATHEMATICS
Chapter 5: Investigation Task 229

1
Question 12 A particle is projected from the origin on level ground with initial speed V m s-
2
and initial angle 0. Assume that acceleration due to gravity is gm s- . Let R be the horizontal
range attained by the particle when it lands.

V 2 sin 20
(a) Showt h at R = ---
g

(b) Hence, show that the Cartesian equation is y = x ( 1 - ~) tan 0.

Question 13 A particle with unit mass is projected horizontally with initial velocity of um s-1,
2
and it experiences a resistive force of k(v+v ) , where vis the velocity of the particle after t seconds.

. . . . .
Show that the time when the particle has velocity v 1s given by t = k1 ln (v(l+u))
u(l + v) .

Question 14 A particle is projected vertically upwards with initial velocity u. When the
2
particle falls back to the ground , it hits the original projection point with speed w. Let gm s-
be the acceleration due to gravity.

H= 2~ In ( 1 + k;
2

(a) Prove that the maximum height of the particle is ).

(b) Show that

Question 15 An object of mass m rests on a table and experiences a friction force F = µN.
Another object of mass 2m is suspended off the edge of a table and is connected to the first object
by a light inextensible string that runs through a smooth pulley.

2mg
(a) Prove that T = - -(1 + µ).
3

(b) Show that a= !(2 - µ).

(c) Describe what happens physically if µ 2 2.


230 Chapter 5: Mechanics

q Investigation Task

Counter-intuitive results

Earlier in the chapter, we studied horizontal resisted motion for both cases when the resistance is
proportional to velocity or proportional to the square of the velocity. This investigation task aims
to shed some light on how mathematical modelling can be delicate, and how a poorly constructed
mathematical model can cause confusion.

Question 1

(a) A particle of mass mkg is projected horizontally across two surfaces with initial speed
um s- 1 . One is made of ice, and the other is made of rubber. One of them has resistive
force mkv and the other has resistive force mkv 2 . Which surface do you think has which
resistive force model? Explain your answer.

(b) Along which surface do you think the particle is more likely to travel further , given a
sufficient amount of time?

(c) Show that for the surface with resistive force mkv, the displacement of the particle after t
u
seconds is given by x = k (1 - e-kt).

(d) Show that for the surface with resistive force mkv 2 , the displacement of the particle after t
1
seconds is given by x = k ln(l + ukt).

Question 2 It appears that one of the surfaces allows for a limiting displacement , whereas the
other one does not allow for that. Which surface did or did not allow for a limiting displacement
to occur? Does this match your intuition?

Question 3 Justify your findings carefully, and de-mystify your findings.

MASTERING MATHEMATICS
Chapter 5: Investigation Task 231

( \ Investigation Task

Uniform Circular Motion

When a particle moves in a circle at a constant angular speed, it is said to be moving in uni-
!arm circular motion. Many scenarios involving uniform circular motion can be analysed using
techniques learned in this chapter. This investigation task allows students to explore Mechanics
a little further for a scenario where the net force is non-trivial.

Question 1 Explain what centripetal force is, and provide a full derivation of the expression(s)
for centripetal force for a particle travelling in uniform circular motion.

Question 2 What is providing the centripetal force in the following scenarios where uniform
circular motion occurs?

(a) A car on a flat surface negotiating a bend.


(b) A mass tied to the end of a string moving in a circle.
(c) A satellite orbiting the Earth.
(d) A car on a banked surface negotiating a bend.
(e) A mass tied to the top of a cone by a rope, and rotating across the surface of the cone.

Question 3 A car negotiates a circular bend with radius r on a surface inclined at an angle of
0 from the horizontal.

N F

mg

It experiences a normal force , a friction force , and a weight force.

(a) What is the 'optimum speed' defined to be?


(b) Resolve forces vertically and horizontally.
(c) Resolve forces parallel and perpendicular to the plane.
(d) Prove that the optimum speed is given by v 2 = gr tan 0.
(e) Explain how the speed of the car is related to the direction of friction.
(f) Let F = µN. What happens ifµ 2:: tan 0?
232 Chapter 5: Mechanics

q Investigation Task

How long does it take to fall?

In Problem 1 of Exercise 5G , a guided problem was given to allow students to calculate the amount
of time that it takes for a particle to fall to the ground given a release point on a particular curve.

The task is to repeat the same problem, but with three different curves and release points of your
choice. For example, you may like to use one quadratic function , one exponential function and
one log function. More marks may be awarded for the following.

• Choosing curves and release points that result in neat values throughout the calculations.

• Choosing curves that require more advanced techniques from Mathematics Extension 2
Integration in order to solve for time.

MASTERING MATHEMATICS
Answers 233

Q5
1. The Nature of Proof (a) a~ b
(b) x ~ [a , b]
Exercise 1A (c) :3n E Z, 2n + 1 not odd.
Language of Proof
(d) 5 divides 2n + 1 for no values of n.
(e) :3m E Z such that n -1- m 2 , Vn ~ m.
Fl
(a) propositions (f) ~x E IR such that sin x = 0.
(b) implication, conditional
(c) converse, p Q6
(a) If I am in Sydney, then I am in NSW.
(d) if and only if
(b) If I am not in NSW, then I am not in
Sydney.
F2
(a) opposite (b) Not (c) rvp (c) It is true!

F3 Q7
I did not do well in my exam. The negation of a conditional statement is a
counter-example. The converse of a statement is
Ql just reading it the other way around .
(a) If Bob studies for his exam, he will get 99.95
ATAR. Q8
(a) If a number is divisible by 6, then it is
(b) If Bob got 99.95 ATAR, then he studied for
divisible by both 2 and 3.
his exams.
(c) If Bob doesn 't study for his exams, he will If a number is divisible by both 2 and 3, it is
get 99.95 ATAR. divisible by 6.
(d) If Bob got 99.95 ATAR, then he didn't (b) If a quadratic has two real roots , then
study for his exams. 6. > 0.
(e) Bob studied for his exam, but didn 't get
99.95 ATAR. If 6. > 0, then the quadratic has two real
roots.
(f) If Bob doesn't study for his exams, then
Bob won't get 99.95 ATAR. (c) If x = a is a zero, then (x - a) is a factor.

Q2 If (x - a) is a factor, then x = a is a zero.


(a) P ➔ q (b) q➔P
(d) rvq /\ p Q9
(c) p/\rvq
(a) If a+ bis even, then a and b are odd. Not
(e) rvp/\rvq (f) rv q --+ rv p
true since a and b could both be even.
(b) If a 2 is odd, then a is odd. True.
Q3
(c) If a2 > b2 , then a> b. Not true for a= -2,
(a) The cat is not black.
b = 1.
(b) A cat that does not have a tail.
(d) If a+ b E (Q then a, b E (Q. Not true for
(c) It doesn't have four legs, but it is a cat.
a = 1 + ~ and b = 1 - ~-
(d) I studied, but did not do well.
(e) If ab is even, then a and b are even. Not
true since we only need at least one of them
Q4 to be even, but not necessarily both.
(a) Bob dislikes either swimming or running. (f) If a number is odd, then it ends with the
(b) Bob likes neither swimming nor running. digit 5. Not true. Consider 23.
234 Answers

QlO Pl
(a) If it has perpendicular diagonals, then it is a (a) T he forward direction is false .
rhombus. Not true because it could be a Let a = $ = 2 + i . The converse is true.
kite.
(b) Neither direction is true.
(b) If exactly two sides of the triangle are equal,
then exactly two angles of the triangle are (c) Neither direction is true.
equal. True.
(c) If the triangle is right-angled, then P2
a 2 + b2 = c2 where c is the length of the
A function f approaches a limit L near x = a if we
hypotenuse. True.
can make fas close as we like to L by having x
sufficiently close to a.
Qll
(a) If f(a) does not exist , then f(x) has a This is also known as the 'Epsilon-delta definition of
vertical asymptote at x = a . Not true since the limit '. Almost every student studying their first
x = a could be a 'hole' discontinuity. mathematical analysis course at university will be
(b) If f(x) = b has no solutions, then f( x) has a confused by this for at least half of their degree.
horizontal asymptote at y = b. Not true like
P3
how y = x 2 can never be equal to y = -1.
It 's the complex conjugate root theorem.
(c) If J" (a) = 0, then there is a point of
inflexion at x = a. Not true for f (x) = x 4 at
X = 0.
Exercise 18
(d) If there is a stationary point at x = a, then
Direct Proof
f'(a) = 0. True.

Q12 Fl
(a) Vz EC, :3z EC such that z + z ER (a) Prove that n = 2m form E Z .
(b) Va, b E Z, a =J b, :3r E (Q such t hat either (b) Prove that n = 2m + 1 or n = 2m - 1 for
a < r < b or b < r < a. mEZ.
(c) Vr E JR, :3q E (Q such that lr - qi :S d, d E (Q. (c) n = ma for m E Z.

Q13 Ql
(a) For all m > n, there exists some real number (a) Let m = 2a , n = 2b.
a such that m - a = n. Basically it means
that if one number is larger than another, (b) Let m = 2a + 1, n = 2b + 1.
there exists a number that 'fills the gap'
(c) Letm=2a,n =2b.
between them.
(d) Let m = 2a + 1, n = 2b + 1.
(b) If P (x) is a polynomial with integer
coefficients and there exists some integer
zero a, then a is a factor of co. Basically this Q2
is the integer root theorem.
(a) Let n = 2m.
(c) If n is not a perfect square, fa is not
(b) Let n = 2m.
rational.
(d) If z is a non-zero complex number, then (c) Let n = 2m + 1.
there exists some complex number w such (d) Let n = 2m + 1.
that zw = 1.
1
(e) If a is a positive real number, t hen a + - is Q3
a
at least 2. For both parts, let m = 2a and n = 2b + 1.
Answers 235

Q4 P2
(a) Consider two cases where n is even or n is n! is divisible by all the integers from 1 to n
odd. Alternatively, recognise that inclusive, and so n! - 1 is not divisible by any of
n 2 + n = n(n + 1) , which is the product of those . So n ! - 1 is either itself prime, or it has some
two consecutive integers. prime factor greater than or equal to n + 1 since it 's
not divisible by 1, 2, 3, ... , n. Hence either
(b ) Recognise that n 3 - n = (n - l)(n) (n + 1) ,
n! - 1 = p or p 2'. n + 1. Note also p cannot exceed
which is the product of three consecutive n! - 1 since it is a prime factor. And so
integers. This necessarily contains one
multiple of 2 and one multiple of 3. (n + 1) S p S (n! - 1)
(c) Prove both directions separately.
and hence
(d) Prove both directions separately. n < p < n!

Q5 P3
n 2 = (2m + 1) 2 = 4m(m + 1) + 1 but m(m + 1) is Let the number be 1000a + 100b + 10c + d and the
even so n 2 = 4(2a) + 1 = Sa+ 1. last three digits form the number 100b + 10c + d.
Q6 Prove both directions.

n + (n + 1) + (n + 2) = 3n + 3 = 3 (n + 1) P4
q=7
Q7
4n + 4n+1 = 4n(l + 4) = 5 X 4n P5
(a) Expand the RHS.
Q8
(b) If n is even, the result is trivial since
Note that
everything is even. If n is odd, let n = 2k + 1
for 4n only and use the previous identity.

P6
So basically, we just need to prove that the Haha good luck!
numerator is divisible by 6. But
n 3 + 3n 2 + 2n = n(n + l)(n + 2) which is the
product of three consecut ive integers.
Exercise 1C
Q9 Contrapositive
Recognise that 2n + 1 = n + (n + 1) .
QlO Fl
(a) p (b) equivalent
gn - 1 = (8 + l) n - 1 then expand. When the 1
rv

cancels out , the remaining terms are multiples of 8.


F2
Qll
(a) contrapositive (b) contraposition
(2m + 1) 2 - (2m - 1) 2 = Sm.

Q12 Ql
Let the number be n = 100a + 10b + c where (a) If you cannot see me, t hen I cannot see you.
a , b, c E Z. We are given that a + b + c = 3m where
(b ) If Bob does not drive to work, then it is not
m E Z. Substitute back into n to eliminate c and
raining.
factorise out 3.
(c) If Mary does not attend, then Bob is not
Q13 attending.
Consider (n + 1) 2 - n 2 = 2n + 1. But this represents
(d ) If I am not late to school, then I did not
every odd integer.
miss my train.
Pl (e) If it does not rain , t hen it is not cloudy.
For both parts , let a= kn+ b. (f) If it does not have four legs , then it is not a
cat.
236 Answers

(g) If Bob did not do well, then he did not P2


study for his exam. The contrapositive is that if n is composite, then
(h) If you are not in NSW, then you are not in 2n - 1 is composite.
Sydney.

Q2 Exercise 1D
(a) If a number is even, then it does not end Proof by Contradiction
with 5.
(b) If a + b is odd, then either a or b are even. Fl
1 (a) negation (b) false (c) true
(c) If a+ - < 2 then a is negative.
a
(d) If ab is not an integer, then either a or b is Ql
not an integer.
(a) z (b) factors
(e) If a quadrilateral is not a rhombus , then it is (c) 2q 2
, even, even (d) z
not a square.
(e) 2n 2 , even, even (f) even, (b)
(f) If a triangle does not have three equal sides,
then not all angles are equal. (g) z
Q2
Q3 See full worked solutions.
(a) Let n = 2m.
Q3
(b) odd, odd See full worked solutions.

Q4
Q4
See full worked solutions.
See full worked solutions.
Q5
Q5 (a) Z (c) even , odd
(a) The contrapositive is that if n > 0, then
n 2 + 5n > 0.
Q6
(b) The contrapositive is that if a < b, then See full worked solutions.
a3 + ab 2 < a 2 b + b3 .
Q7
Q6 See full worked solutions.
The contrapositive is that if n is not divisible by 3, Q8
then n 2 is not divisible by 3.
See full worked solutions.
Q7 Q9
The contrapositive is that if either a or b are See full worked solutions.
divisible by 5, then ab is divisible by 5.
QlO
Q8 See full worked solutions.
Prove both directions. One can be done directly and
the other can be done by contraposition. Qll
See full worked solutions.
Q9
The contrapositive is that if a + b is odd, then Pl
a 2 + b2 is odd. See full worked solutions.

Pl P2
The contrapositive is that if neither a nor b are Let a = 2m + 1 and b = 2n + 1 and suppose
greater than 10, then a+ b < 20. a 2 + b2 = k 2 . Upon expansion k must be even so let
k = 2p and then obtain a contradiction.
Answers 237

P3 Q2
See full worked solutions. (a) n = 2 (b) n = 4
P4 (c) n=41 (d) n = 7
See full worked solutions.

P5 Q3
See full 'Yorked solutions. (a) X = -2
P6 (b ) a = 12, b = 6, n = 36

See full worked solutions. (c) n =6

Q4
Exercise 1E = 2, =3 = = J2
(a) X y (b) X y
Examples and Counter-examples

Q5
Fl
(a) J(x) = (x - a) 3 (b) f( x ) = (x - a) 4
negation

F2
Q6
Show that there does not exist any n such that the
statement is true. (a) True. a 2 + /3 2 + "( 2 < 0 is a sufficient
condition for non-real roots.
F3
(b) False. Let n be a perfect square.
Find some n E S where the statement is not true. In
other words, find a counter-example. (c) True. The odd integers have remainders 1 or
3 when divided by 4.
Ql
(d ) False. Leta=b=n.
(a) x=-1
'vx > 0 , 3x > 2x (e) True. The signed area bounded by the graph
of f'(x) is positive, as f'(x) is positive.
(b) x = 2
'vx E lR where x # 1, 2, 2x -/ 2x
Q7
(c) X =2
'vx > 4, 3x < 2x (a) The converse is false. Let a= b = 2.

(d) x = 0 (b) True.


'v x # 0 , ax = bx ==} a= b (c) The converse is false. Let a= 1, b = 2.
(e) X = -1 (d ) The converse is false. Consider 1.
\Ix 2: 0, J"x2 = X
21r
(f) X =- Q8
3
'vx where cos x 2: 0, J1 - sin 2 x = cos x (a) Let P(x) = x 2 , Q(x) = -x 2 .

(g) X = 1, y = -2 (b) Let a= b, P(x) = (x - a).


'vx, Y E lR, lxl + !YI 2: Ix+ YI X

(h) X=-1
(c) Let P (x) =
2- 1.

'vx > 0 , e1n x = x


Q9
(i) X = 2
(a) False. Consider a kite.
'vx E [-1, 1], sin ( sin - 1 x) = x
(b) True.
(j) X = -1
(c)
lfx > 0, tan- 1 (x) + tan - 1 G) = ~ True.
238 Answers

QlO F6
(a) True. lx +yl
(b) False. Let f( x) = ex sinx , b = 0. Then Ql
f(x) intersects b = 0 infinitely many times. (a) x = a, y = b (b) x = a 2 , y = b2
4
(c) False. Let y = (x - a) .
1
(d) True. Definition of a stationary point. (c) X = Vo,, y = \l'b (d) x=Ja,y= Ja

Qll
(e)
1
x=~,y=b
1
(f) X = l, y= fa
The claim is false. Let n = 3, a= c = 2, b = 4.

Q12
The claim is true. Let n = 100a + b where Q2
0:::; b:::; 99. See full worked solutions.
Pl Q3
(a) True. The condition cannot be satisfied by x (x - 1) + y(y - 1) ~ 0
real roots .
Q4
(b) False. Let a: = i3 = 2+i
(a) Use a + b > 2 v1alJ and ! + !b > ~ then
- a - vab
multiply.
P2
X y
(a) False. Let y = (x - a) 2 . (b) Expand and use - + - ~ 2.
y X
(b) False. Let y = (x - a)2.
Q5
P3 (a) Prove LBS - RBS~ 0.
(a) True.
(b) False. Let a= 0, b = 3, f(x) = x, g(x) = l.
(b) Instead prove (a+ b) ( ~+ D 2: 4.

(c) False. Let n = -1. 1


(c ) Instead prove (a+ b) 2 ( : 2 + b2 ) 2: 8.
(d) Prove LBS - RBS~ 0.

Exercise 1F
Q6
Algebraic Inequalities
[~] . [~] = ✓a2 +b2 ✓x2 + y 2 cos0 $ 1. But
Fl
(a) > (b) > [~] · [~] = ax +by.

Q7
F2
Expand LBS.
(a) > (b) < (c) > (d) <
Q8
Prove that LBS - RBS~ 0.
F3
(a) > (b) > (c) > Q9
(a) Instead prove that 4(a 3 + b3 ) - (a+ b)3 ~ 0.
F4 (b) Divide both sides of the previous inequality
> by 2, the cube root both sides.

F5 QlO
positive See full worked solutions.
Answers 239

Qll Q19
See full worked solutions. (a) Prove that RHS - LHS 2:: 0.

Q12 (b) Split the numerator and use ~ + '!!_ 2': 2


y X
See full worked solutions. repeatedly.
Q13 X y
(c) Expand the LHS and use - + - 2:: 2
(a) Prove that RHS - LHS 2:: 0. y X
repeatedly.
(b) Replace c with a 2 and d with b2 from the
previous inequality. (d) a(b - c) 2 + b(a - c) 2 + c(a - b) 2 2:: 0

Q14 Q20
(a) (a - b) 2 2:: 0 (a) (\IP - .jq) 2 2:: 0

(b) a2 + c2 2:: 2ac (b) Rewrite


a + b+ c + d
as
a!b + ctd and
4 2
b2 + c2 2::: 2bc apply (a) repeatedly.
(c) Add the three inequalities and divide both
sides by 2. Q21
(d) Replace a with xy, b with x z and c with yz. a+b+c
Let d = and raise both sides to the power
3
4
Q15 of-.
3
(a) Usual proof as done earlier.
Q22
(b) a 2 + b2 + c2 = (a+ b + c) 2 - 2( ab +be+ ac) 1
Recall that logb a = - --b.
1oga
Q16 Q23
(a) ( va - v1br 2:: 0 1 + a 2': 2fa and similarly for band c, then multiply.

(b) a+ c 2': 2y'ac Q24


b + c 2:: 2v'bc (a) Standard proof as done earlier.

V~
(c) Multiply all three inequalities. (b) Replace a with and similarly for the
yz
others.
Q17
(a) Standard proof as done earlier. Q25
(b) Expand the RHS. See full worked solutions.
(c) The result in (b) is positive because of part Q26

(1+ b1+ 1) 2':


(a) .
Instead prove that (a+ b + c) -;, ~ 9,
(d) Replace a with iji and similarly for band c.
which was done earlier.

Q18 Q27
(a) Prove that LHS - RHS 2:: 0.
(a) (~-fS ~o (b) Add three similar inequalities, factorise abc
a C and then use ~ + '!!_ 2': 2 repeatedly.
(b) -
C
+ -a -> 2 y X
(c) Suppose without loss of generality that a is
b C largest.
-+-b -> 2
C LHS - RHS 2:: -
s
a
(c) Multiply the LHS and apply (b) . where S = a 3 + b3 + c3 - 3abc.
240 Answers

Pl F3
First prove that diagram
Ql
(a) LHS = 24 = 16
and
ab be ac RHS = 4 2 = 16
-+-+->a+b+c
c a b -
and then combine the inequalities. (b) (k+1) 2

P2 (c) k2 , k-1, 3, 4
Suppose without loss of generality that a 2 b 2 c.
Observe that b(b - a)(b - c) is the only negative Q2
term. Replace it with an 'even more' negative term See full worked solutions.
b( b - a) (a - c). Prove that the first two terms are
positive. The third term is positive anyway. Q3

P3 (a) To = 2 x 3° - 1 = 1, as expected.
Expand the LHS and observe that you get n lots of
(b) 2 x 3k+1 - 1
'1 ' and a number of (Xi
Xj + Xj) pairs where i -:::J. j,
+ 2,
Xi (c) 3Tk 3k+l

each of which is at least 2. Precisely, there are (;)


Q4
such pairs so LHS 2: 2 x (;) + n = n2 . See full worked solutions.
Q5
P4
1 1 See full worked solutions.
Substitute a= - (-x + y + z), b = -(x - y + z) and
2 2 Q6
1 p q
c = - (x + y - z) and use - + - 2 2 repeatedly. See full worked solutions.
2 q p
P5 Q7
(a) P(x) is actually the sum of perfect squares, See full worked solutions.
so geometrically it would either be a
Q8
parabola on or above the x-axis i.e. it either
has one real root or no real roots. 1
(a) LHS = l2 =.1
(b) ~ :s; 0
1
RHS = 2- - = 1
(c) Let ak = ~ a n d bk= ✓ S~xk_ 1
1
(b) 2- -
k+l
(c) 1
Exercise 1G
Mathematical Induction Q9
See full worked solutions.
Fl QlO
(a) before (b) initial, initial (a) Prove that RHS - LHS > 0.
(c) closed, closed (d) closed, recurrence (b) See full worked solutions.

F2 Qll
(a) LHS - RHS (a) Prove that LHS 2 - RHS 2 > 0.
(b) LHS - RHS 2 0, LHS 2 RHS (b) See full worked solutions.
Answers 241

Q12 P5
See full worked solutions. See full worked solutions.

Q13 P6
See full worked solutions. See full worked solutions.
Q14 P7
See full worked solutions. (a) Calculate both using binomial probability
Q15 and use the symmetry of the binomial
coefficient.
See full worked solutions.
(b) See full worked solutions.
Q16
See full worked solutions.

Q17
See full worked solutions. Exercise 1H
Inequalities using Differentiation
Q18
See full worked solutions.
Fl
Q19
(a) The function is increasing, but it could start
(a) See full worked solutions.
negative.
(b) (2n)! = (2n) (b) The function could start positive, but then
n!n! n
it decreases.
Q20
See full worked solutions. F2
b
Q21
(a) See full worked solutions. Ql
(a) f(x) = ex - x - 1
(b) Pn - qn \l'2
(c) v'2 (b) f(x) = x - sinx

Q22 Q2
See full worked solutions. See full worked solutions.

Q23 Q3
See full worked solutions. See full worked solutions.

Q24 Q4
See full worked solutions. (a) (0,1) , (4, 0)
Q25 (b)
y
See full worked solutions.
Pl
See full worked solutions.

P2
See full worked solutions. 4 X

P3
(c) f(x) ~ 1
See full worked solutions.
(d) Substitute x = 1 and x = -1.
P4
See full worked solutions.
242 Answers

Q5 F2
(a) See full worked solutions.

(b) Let x = ~ - 1.
1' f (x) dx > 1' g(x) dx
e Ql
The area under the curve is between the rectangles.
Q6

(e, D
Q2
(a) The area under the curve is between the trapezium
and rectangle.
(b) f(1r) < f(e)
Q3
The area under the curve is less than the area of the
Q7 rectangle.
See full worked solutions.
Q4
Q8 (a) The area under the curve is between the
(a) See full worked solutions. rectangles.

(b) (b) See full worked solutions.


y

Q5
(a) The area under the curve is between the
upper and lower rectangles.
(b) 7 and 9.
n X
(c) No, since Hn grows asymptotically with ln n
(c) Let x = n + 1. according to (a) , and ln n diverges as
n-+ oo.
Pl
(a) See full worked solutions. Q6
(a) The area under the curve is between the
(b) ln x s; x - 1 then substitute x = npk and upper and lower rectangles.
then sum them.
(b) 60525 and 60571.
n
(c) First note that 2::)n(npk) ::; 0, then use log
k=l Q7
laws and manipulate.
(a) 1 ::; t + 1 ::; 2 then flip everything.
(b) Integrate everything from 0 to x.
P2
(a) See full worked solutions.
Q8
a1 + a2 + · · · + an
(b) Let x
ak
= - w h ere b = - ------ (a) 1 ::; 1 + t 2 ::; 2 then flip everything.
b n
Produce n such inequalities and multiply (b) Integrate everything from 0 to x.
them.
Q9
See full worked solutions.
Exercise 11 QlO
Inequalities using Integration See full worked solutions.

Qll
Fl See full worked solutions.
L< 1' f (x) dx < U Q12
See full worked solutions.
Answers 243

Q13 R5
(a) The area under the curve is between t he See full worked solut ions.
upper and lower rectangles . R6
(b) See full worked solut ions. See full worked solutions.
1 R7
(c)
3 See full worked solutions.

Pl R8

fo\ :x dx = ~
2
(a) False.
(d) True.
(b) True.
(e) False.
(c)
(f)
False.
False.
P2 (g) True. (h) False. (i) False.
(a) The area under the curve is between t he (j) True. (k) True. (1) False.
upper and lower rectangles. Then use log
laws to form t he factorials .
R9
(b) Evaluate the integral in the
double-ine quality from the previous part,
t hen algebraically manipulat e t he
G-if ~ 0

expression s. RlO
(c) For most calculators you get MA TH ERROR Xk + Yk 2: 2 and then sum n such inequalities.
Yk Xk
because the calculator tries to calculate the
th
100! first before taking the 100 root . RU
Prove that LHS - RHS 2:: 0.
(d) 37 and 39.
R12
(a) Prove t hat LHS - RHS 2:: 0.
(b) Expand t he RHS.
Chapter Review (c) Add three similar inequalities and use part
Rl the first inequality.
The statement is true.
R13
R2 See full worked solutions.
The statement is false for a= 2, b = -2.
R14
R3 See full worked solutions.
See full worked solutions.
R15
R4 See full worked solutions.
(a) The contraposi tive is if n divides a or b, R16
then n divides ab.
See full worked solut ions.
(b) The contraposi tive is that if neither a nor b
R17
are irrational, then ab is rational.
See full worked solutions.
(c) The contraposi tive is that if 5 is either a
factor of m or n, t hen 5 is a factor of mn. R18
See full worked solutions.
(d) The contraposi tive is that if a divides b,
then a divides be. R19
(e) The contraposi tive is that if eit her a or b are (a) f(x) = tanx - x
odd, t hen either ab or a + b are odd. 2

(b) f (X) = x - In ( l+ x + ~ )
(f) The contraposi tive is that if neither a nor b
are divisible by 5, then ab is not divisible by
5. R20
See full worked solutions .
244 Answers

R2 1 Q5
First show that (a) X = 4, y = 6 (b) x = 2, y = S
(c) x=3,y=-4 (d) x=4,y=3

and then integrate each expression from Oto x. Q6


z = -1 +i
R22
Q7
(a) The area under the curve is between the
See full worked solutions.
upper and lower rectangles .
(b) 87 and 89. Q8
Let z = x + iy and expand/simplif y the left-hand
side.
Q9
2. Complex Numbers Cross multiply and use the fact that z + z = 2Re (z)
and zz = lzl 2.
Exercise 2A QlO
Arithmetic of Complex Numbers Let z = x + iy and expand/simplif y.
Qll
Fl (a) a 2 - b2 + 2abi
(a) real, Re (z) (b) imaginary, Im (z)
a bi
(c) imaginary (d) real (b) a2 + b2 - a2 + b2
(e) a - ib (f) ✓a2 + b2 a2 + b2 - 1 2bi
(c) (a - 1) 2 + b2 (a - 1) 2 + b2

F2 Q12
(z) See full worked solutions.
(a) z+w (b) (z)(w) (c)
(w)
Q13
F3 k = ±1
(a) 2Re (z) (b) 2i Im (z) (c) lzl2 Q14
Let Re (z2 ) = 0 and show that Jxl = JyJ.
Ql Q15
(a) -1 (b) -i (c) 1 (d) i (z - l)(z - 1) = 1 and then use conjugate properties
(e) -i (f) 1 (g) i (h) (-l)n and expand.
Q16
Q2 Jz + wJ 2 = (z + w)(z + w) and then use conjugate
(a) S - 3i (b) 1 - Si properties and expand. Similarly for Jz - wJ 2 .
(c) 10 - Si
s Pl
(d) 3 +4i (e) (f) J5
1
(a) Let z = x + i y and let Im ( z - ) = 0 to
z +l
Q3
show that y = 0.
(a) 2 - 3i (b) 4
(c) 6i (d) 13 (b) Let z = x + iy so x 2 + y 2 = 1. Show that

Q4 Re ( z
z- 1)
+ 1 = 0.
(a) i (b) 2 - 3i (c) 0 + 4i P2
Square both sides, then use conjugate properties.
Answers 245

P3 Ql
(a) lzl = ✓x +y 22: H = lxl
2 (a) ±(2 + i) (b) ±(3 - i) (c) ±(3 - 2i)

(b) Consider lz + wl 2, use conjugate properties (d) ±(1 - i ) (e) ±(6 + i) (f) ±(5 - 4i)
and expand. (g) ±(4 + 3i) (h) ±(2 + 5i) (i) ±(1 + 5i)
P4
Q2
See full worked solutions.
(a) z = ±4i
P5 z2 + 16 = (z - 4i )(z + 4i)
Divide the LHS by lz 1 z2z3 I = 1, then recognise that (b) z = - l ± 2i
lzkl = 1 then
1
- = Z k- z2 + 2z + 5 = (z +1- 2i)(z + 1 + 2i)
since
Zk
1 . v'3
P6 (c) z =- - ±i-
2 2
(a) Take the modulus of both sides 2
z + z +l = (z+~ -~\) (z+~ +~i)
(b) lz - al= lz - _z__ I and similarly for Q3
1 + ki
lz - ,BI (a) z = l ± 2i (b) z = -3 ± 4i
(c) Consider the circle centred at the origin in
the complex plane that passes through a Q4
and ,B. The tangents drawn from a and ,B (a) z = l + i, 2 - i (b) z = -l - 3i, 3 + i
intersect at z .

Q5
P7
k = - 8+i
(a) See full worked solutions.
Pl
(b) z = l ± i l+i ± 1 + 5i
(a) ± - (b) ± 3 + i (c) ~
(c) See full worked solutions. ~ ~

P2
See full worked solutions.
Exercise 28 P3
Solving and Factorising Quadratics z = ±(2 + i) , ±(1 - 2i)
P4
Fl Expand (z - a)( z - a) .
(a) 2Re (a)
P5
(b) lal2 P(a) = aa 2 + ba + c = 0 then conjugate both sides
(c) (z -(a+,B) z+a,B ) to show that P(a ) = 0.

F2
(a) (z -a)(z -a)
Exercise 2C
Polar Form and the Argand Diagram
(b) P(z) = z2- 2Re (a) z + lal2
Fl
F3 (a) real , imaginary
(a) quadratic (b) discriminant
(b) Argand
(c) square (d) sum/ product
(c) real, imaginary
246 Answers

F2 Q5
(a) modulus, a 2 + b2 3 3
b
(a) v'2 cis ( - : ) (b) ~cis (- : )
(b) argument, -, 0 E (- 1r, 1r]
(c) polar, cis 0
a
(c) v'2 cis ( -
3
:) (d) 2cis (3;)
F3 Q6
A
(a) AB cis( a+ /3) (b) - cis(a - /3)
B (a) 12 cis (i) (b) 12 cis ( -i)
F4 (c) 3cis (12
llrr) (d) 2cis ( - : ; )
2
(a) lzllwl (b) 1 1

lwl Q7
(c) kl zl (d) argz + argw (a) ~ cis (i) (b) cis (i)
(e) argz - argw (f) 0 (c) ~ cis ( - ;2 ) (d) ~
~
cis (1h)
12

Ql Q8
(a) 2 cis (i) (b) 2cis (2;) (a) 5 + 5i
(b) arg ( (2 + i) (3 + i)) = arg(5 + 5i)
2
(c) 2 cis (-i) (d) 2cis (- ; )
Q9
(e) 2 cis (i) (f) 2cis (5;) (a) 2~ (b) 4~ (c) ~
21r
(g) 2 cis (-i) (h) 2c1s . (- 65rr) (d) 1 (e)
3
(f)
lb
- -
12
51r
(g) (h) 0
(i) (j) 6
~cis (~) ~cis (-~)
QlO
(k) v'2cis (3:) (1) ~ClS . ( 43rr)
-
(a) ( 1 - ~) + i (1 + ~)
(b) Equate real and imaginary components of
(m) 3 cis (i) (n) 4 cis ( -i) both expressions for zw .

(o) 5 cis(O) (p) 2cis(1r) Qll


See full worked solutions.

Q2 Q12
See full worked solutions.
(a) 4cis (5;) (b) 2v'2 cis ( -
2
;)
Q13
(c) Use cos0 = 2cos 2 -1 and

mm
~cis (~) (;)

Q3 sin0 = 2sin
(a) l+i~ (b) -3 + 3~i
(c) -1-i Pl
Let z = Rcis0
Q4
P2
(a) 2 cis (i) (b) 2cis (5;) (a) Add z and w, and then use the auxiliary
angle formulae.
3
(c) v'2 cis ( - : ) (d) 2~cis (-i) (b) 2 ~
'4
Answers 247

P3 Q3
(a) 2 + 6i (b) 4 - 2i
(a) (~+D i iy iy
(b) Calculate arg (zw) using two different 6 _• z1 + z2
4
methods.
X

P4
2 X

Use cos 20 = 2cos 0 - land sin20 = 2sin0cos0


2

P5 (c) 3 - 2i (d) 6
See full worked solutions. iy iy
P6
3
See full worked solutions. X
6 X
I
P7 - 2 - - - - - -• z1

See full worked solutions.

(e) 4i (f) 8i
Exercise 2D
Vector Representation iy iy

8 z2 - z-2

Fl
4 Z1 -Z-1

(a) b-a

(b) AB. X X

(c) position

Q4
F2 (a) (b)
(a) ?5f> by a factor of 2.
Lengthen

(b) Rotate ?5P anti-clockwise by a.


(c) Rotate ?5P clockwise by a.

(d) Rotate ?5f> anti-clockwise by i.

Ql (c) (d)
(a) -8-i (b) 8+ i

(c) 1 - 5i (d) -9+4i

Q2
(a) 11 + 6i (b) - 11 + 8i (c) 7 + 2i
248 Answers

Q5 Qll

(a) 2 cis (5:) (b) ~ cis ( -i) (c) cis ( -i) (a) R = 4vl3 - 6vl3i
(b)
iy
Q6
(a) Double the length, rotate anti-clockwise by
7r
-
3
7r
(b) Halve the length, rotate clockwise by .
4
(c)
.
Keep the length, rotate clockwise by
31r . (c) S1 = (12 + 4vl3) + i (8 - 6vl3)
4
S2 = (-12 + 4vf3) - i(8 + 6vl3)
Q7 S3 = - 4vl3 + 6vl3i
-6 +4i
Q12
Q8 (a)
(a) (b)

0 X
··Q2
Q1 = 1 + 5i
Q2 = 5 - i
(b)

(c) (d)

0 .... :-.x
Q2
Q1 = 2 + lOi
Q2 = 10 - 2i

Q13
Q9 (a)
iy
1 5. 9 1.
B=--+ - i D=-+-i
2 2 ' 2 2
B
QlO
(a) A.
iy
X

(b) C 1 = -4 + 6i, D1 = lOi


C2 = 4 - 2i , D2 = 8 + 2i
(c) C = 6i, D = 4 + 2i
(b) R1 = -2 + 2vl3i or R2 = 1- ivf3
Q14
B = 71r) , C = v'2 cis (lb)
.J2 cis (12
12
Answers 249

Q15 P4
(a) OW x cis (~) = OZ sow= iz. The other = ±
7r

2
so it forms a right-angled
result is because z and w could be swapped.
The diagram shows z = iw. triangle.
(b) Square both sides of either result , then
re-arrange. P5
See full worked solutions.
Q16
w = z x 2 cis ( ~) , then raise both sides to the P6
fourth power. See full worked solutions.
Q17
(a) z = w x cis (i) , then cube both sides and Exercise 2E
re-arrange.
Locus
(b) z 3 + w3
(c) z +w # 0 so it follows that
z2 - zw + w 2 = 0. Fl
(a) Argand
Q18
(b) set, line, curve, region
AS = iAC so it forms a right-angled isosceles
triangle. (c) locus

Q19
See full worked solutions. F2
Q20 (a) A circle centred at C with radius r.
Find arg ( z 3 - zi) and show that it is either O or (b) The interior, including the circumference, of
Z2 - Z1
1r.
a circle centred at C with radius r.

Q21 (c) The exterior, including the circumference, of


a circle centred at C with radius r.
See full worked solutions.
Pl (d) The straight line that passes through the
midpoint of AB and is perpendicula r to AB.
(a) Rotate one of the sides of the triangle to get
another side of the triangle. (e) The half-plane containing A, bounded by
(b) Produce a similar result , equate the the same straight line described in the
previous part. The locus includes the
cis ( ± i) part and then re-arrange. boundary line itself.

P2 (f) The ray starting from C with an open circle,


Draw out z, w , z + w and z - w and it forms a inclined at an angle of 0 from the positive
rhombus. Notice that horizontal axis.

z+w) 1r
- = arg(z + w) - arg(z - w) = ± - Ql
arg (-
z - w 2
(a) (b)
iy iy
since the diagonals are perpendicul ar, and so it is
purely imaginary.
P3 3
X
(a) They form a rectangle.
7r
(b) ±- 2 X
2
250 Answers

(c) (d) (c) (d)


iy iy iy iy
2
3

X
4 X X
-1 X

Q4
Q2
(a) (b)
iy iy
(a) (b)
iy iy

71" 2 X X
3
- - - - --1
- ------►
X

(c) (d)
iy iy
(c) (d)
iy iy
-1 X X

X X

Q5
(a) X= -1 (b) y =l
(e) (f)
iy iy iy iy
X

71"

.....4. ..... 3
-1 X X
-4
-4 X

(c) y=-x (d) y = 3-x


Q3 iy
(a) (b)
iy iy
X X
1
,,. '
I

X X
Answers 251

Q6 (e) (f)
iy iy
(a) (b) 1
iy iy -2
X
1 1 X
~ -2 2
X -2 ' ' ' X
-1
-1.2 '

....
Q9
(a) (b)
iy iy
Q7

(a) (b)
~ iy iy 1 1

1 X 1 X
\
\
271"
3
X 2 X

QlO
(a) (x - 1) 2 + y2 = 2 (b) x y = l

iy

L
(c) (d ) iy
iy iy I
~ I
I
1 I 11"

' 371" X ,\ 3
X X

\
' Q· · . v'3
-1 '~~- --- ,)_ .Z!:.
\ 3
\

-1 \ X
\
1
(c) y= 2x (d ) y = - x2
4
Q8 iy iy

(a) (b)
iy iy
2 ---- - --
X
X

3 X

2 X
(e) y = 0 excluding (f) (X - l )2 + y 2 = 1
(0, 0) . excluding (0, 0) .

iy iy
(c) (d)
iy iy
2
2
1 X

.5 X
- 1

X - 2
2
252 Answers

Qll (d) x = 0 and x 2 + y 2 = 1 excluding (0 , 0)


(a) lzl > 2 iy
7r
(b) 0 < arg( z) <
3
1 X
Q12
(a) 1 +J2 (b) 1 - v12
7r
(c) - (d) 0 Q16
2
(a) x2 + y2 = 1, where y > 0.
Q13
iy
(a) 1 (b) 7

(c)
2rr
+ sin- 1 (3)4 (d) ~- sin-
1
m
1 X
Q14

(a) (X - ff + y2 = l: iy
(b) x 2 + y2 = 1
3
v'3

Q15
(a) y = x - l excluding (0, - 1)
1 X

-1
iy

r
v'3

(b) (X - ~ + y2 = ~ excluding (0, 0) 1 X

iy
Q17
1
2 1
X < --
- 2
X
Q18
x = 0 if a> b, and x =a+ b if a < b.
(c) y = x +l excluding (- 1, 0) Q19
It is the circle with diameter being the interval
connecting a to b.

Pl
b .
w - a = - and take the modulus of both sides.
z
- 1 X
Answers 253

P2 Q4
(a) It is a straight line. More precisely, it is the (a) -4-4i (b) 64
perpendicul ar bisector of the interval from (c) Si (d) -64v13 + 64i
the origin to k on the complex plane. -512
(e) -4 (f)
(b) Let w = ~, where i = 1, 2, 3. (g) -64 (h) 24v13
Zi
(i) 128 - 128i
P3 Q5
(z +it= -( z - l)n then take the modulus of both Multiply the top and bottom by cos 0 - i sin 0
sides. The locus is the imaginary axis and so the
roots are imaginary. Q6
Use the fact that cos 0 - i sin 0 = cos(-0) + i sin(-0)
P4
x2 + y2 = 1 Q7
(a) cosn0 (b) sin n0
P5
See full worked solutions. Q8
See full worked solutions.
Q9
Exercise 2F cos 40 - i sin 40
De Moivre's Theorem
QlO
(a) n = 6k where k E Z
Fl
(b) n = 3 + 6k where k E Z
cos n0, sin n0

F2 Qll
(a) JznJ = JzJn (b) arg(zn)=n argz (a) n = 4k where k E Z
(c) zn (b) n = 2 + 4k where k E Z

Ql
Q12
(a) cos 60 + i sin 60
(a) n = 3k , where k E Z
(b) cos 50 - i sin 50 3
(b) n = - + 3k , where k E Z
(c) cos 60 - i sin 60 2

(d) cos 30 + i sin 30 Q13


(a) Notice that z is in the form
Q2 z = an + an = an + an = 2Re (an)
1 v13 . (b) No, because similarly it becomes
(a) --- - i (b) - 1 (c) -i z = an - an = an - an = 2ilm (an) so it is
2 2
actually purely imaginary.
Q3
7r
(c) Use the fact that z = 2~+ 1 cos (:w)
(a) JzJ = 2, arg z =
3
Q14
(b) 2 cis (i) See full worked solutions.
Q15
2
(C) ] 6 cis ( - ; ) (a) Turn everything to polar form then recognise
that we are adding two complex conjugates.
(d) -8 - 8v/3i 1
(b) The cosine term is either ± , or ± 1.
2
254 Answers

Q16 F3
cos n0 + i sin n0 (a) sin n0 (b) divide
Q17 (c) cosine (d) tan 0
Use cos 20 = 2 cos 2 0 - l and sin 2 = 2 sin 0 cos 0
Ql
Q18 Find z 3 using two different methods and equate
Express each term with a common denominator and real/imaginary components.
use de Moivre's theorem twice.
Q2
Q19
Use the sum of a geometric progression formula. Find z 3 using two different methods and equate
real/imaginary components.
Q20
See full worked solutions. Q3
Find z 4 using two different methods and equate
Pl
real/imaginary components.
(a) Use the sum of a geometric progression
formula. Q4
Find z 5 using two different methods and equate
(b) Factorise out cis ( ~) and use the fact that
real/imaginary components.
a - a = 2iim (a)
Q5
(c) Equate the imaginary components of the
series expression for Sn, and the closed-form Find z 6 using two different methods and equate
expression for Sn. real/imaginary components.

Q6
P2
cos0
(a) zn + z-n = cis(n0) + cis(-n0) = 2 cosn0
(a) Use cot0 = --:-- . 5
sm 0 (b) Consider (z + z- 1 )
(b) Let x = cot 0 and solve sin n0 = 0.
8
(c) Use the sum of the squares of the zeroes of (c)
15
P(x) = 0 by first expanding using binomial
expansions.
Q7
(d) Use cot 2 0 = csc 2 0 - l
(a) Consider (z + z- 1 )4
. 7r

2-
(b) Replace 0 with 0 and use
supplementary / complementary identities.
Exercise 2G
Applications of de Moivre's Theorem (c) Add the two previous results.

Fl Q8
b C d
(a)
a
(b) (c) (a) Consider(z + z- 1 )6
a a
1
F2
(b)
32
(10-15cos20 + 6cos40 - cos60)
(a) z = cos 0 + i sin 0
(b) de Moivre's Q9
(c) Pascal's, binomial (a) zn + z-n = cis n0 + cis(-n0) = 2 cos n0
(d) real (b) z = ±i, -2 ± J3
2
(e) cos 0, sin 0 = 2
l - cos 0
Answers 255

QlO Q15
(a) See full worked solutions. (a) Consider (cos 0 + i sin 0) 5

= ~ 51r 71r (b) See full worked solutions.


(b) 0
9' 9 ' 9 P(x) = l6x 4 + l6x 3 4x 2 - 4x +1
(c) X = COS m, COS (5;) , (7n)
COS
9
(c)
(d) a=2
-

(d) 0 -1 + vl5
(e) sin ( ; ) =
0 4
4
(e) --
3
Q16
1 (a) Consider (cos 0 + i sin 0) 6
(f) -
8

Qll
(b) ±cos(~) ,±cos(~;) ,±cos(~;)
(c) Use the product of roots.
(a) X = COS (2;) , COS (
4
;) , COS ( S;) = (d) Use the sum of the squares of the roots.
(e) Use o:2 + 132 = (o: + /3)2 - 20:/3
- cos(~)
(b) Use the sum of roots. Q17
(c) Use the product of roots . (b) 1
(c) 14
Q12 (d) 4
(a) Consider (cos 0 + i sin 0) 4 (e) x 2 - 4x +1= 0
(b) x = ±cos G), ±cos (3;) (f) tan ( ; ) = 2 - J3
2
(c) cos ( i) = ~ ✓2 + ~
tanG;) =2+~
cos (3;) = ~ ✓2 - ,/2 Q18
Q13 (a) Obtain sin 50 and cos 50 by usual means,
then divide.
(a) t = -1, tan ( ; 2) , tan ( ~;)
(b) t = ±tan G), ±tan (2;)
(b) tan ( ; ) = 2 - J3 (c) Use the product of the roots.
2
(d) Find the sum of the squares of the roots
tanG;) =2+~ first.
(e) Construct a quadratic whose roots are
Q14 2
(a) Consider (cos 0 + i sin 0) 5 tan ( ~) and tan ( ;) , then solve.

✓5 + 2Js
(b) cos (; ), cos
0
G~) , G~), G~)
cos cos
(f)

2
(c) Solve the quartic in terms of x by using the Pl
quadratic formula, and then take the square Let z = r cis 0 and consider the limiting sum formula
root again.
2 3 1
~✓ 5-vi5 l+ z +z +z +· ··=--
l- z
(d)
2 2
then equate real components.
256 Answers

(c) P(w 2 )= w6 -1 = 0
Use de Moivre's theorem.
(d) w 3 = 1 = ww, then divide both sides by w.
Expand (z + z- 1 ) 2 n.
(c) Integrat e both sides. The right-hand side Q3
has all cosine t erms except for the last term
(a) 1 (b) 9
(2:) . The cosine terms integrate to zero
(c) 1 (d) w

but the last term survives. Q4


P(x) = x 2 - x+1
P3
(a) (z 2 + l)n Q5
(a) (b)
(b) See full worked solutions. iy iy
(c) See full worked solut ions . w w
/ /,.,. - ..... '
I
I w2/ 1r/2 \

Exercise 2H w
2 - -
w3
Roots of Unity

Fl (c) (d)
(a ) 1 iy
(b) principal, argument

F2
(a ) 1 (b) 0

F3
(a) 1 (b) 0
Q6
F4
(a) a (b) polar (a) - 1, ± cis (i)
(c)
(e)
2k7r
conjugate
(d)
1
- , principal
n (b) 3i, 3cis (-iD , 5
3cis (- ; )

Ql
(c) 2 cis ( k;) where k= -5, 1, 7
(a) 1
(b) w 3 = 1 so w 3 - 1 = 0 and then factorise .
Q7

(a) cisVi) , cis ( ± 3: )


Q2
1 vl3 .
1 -- ± - i
(b) 2cis (k;) where k= -7,-3, 1,5
(a)
' 2 2
(b)
iy (c) cis ( ~:) where k= -19, -7, 5, 17
+ ../3i
_l
2 2
. .
Q8
2 4
(a) 1, cis ( ± ; ) , cis ( ± ; )

(b) - 2i , 2 cis ( :; ) where k= -9 , -1 , 3, 7


Answers 257

P2
(c) 2cis (~;) where k = -17, -9, -1, 7, 15
Expand it and simplify.

P3
Q9
+ l)n = -1
(a) ± 1, cis ( ± i) , 2
cis ( ± ; )
Re-arrange to instead solve ( _zz --1
P4
(b) cis ( :; ) where k = -9, -5, -1, 3, 7, 11
Group them in pairs

(c) 2 cis c;) where 1


--+- - -- = 1
- wn-k+l
1 - wk 1
1

k = -25 , -13, -1, 11 , 23 , 35


and recognise that we are just adding n such pairs.
QlO
P5
(a) w is a zero of P(x) = xn - 1. Show that
1
P(wk) = 0. -
2
(b) Use the sum of the zeroes.
P6
See full worked solutions.
Qll
(a) wn = 1 = ww
(b) wn = 1 = wkwk Exercise 21
(c) (w , w8 ), (w 2 , w7 ), (w3, w 6
) ,
5
(w4, w ) Applications of Roots of Unity

Q12 Fl
(a) w5 - 1 = (w - 1)(1 + w + w2 + .. . + w ) = 0
4
z 2 - 2Re (a) z + lal 2
(b) Use the sum of the roots. F2
(a) 1 + z + z 2 + z 3 + • • • + z n-l
Q13 (b) 1 - z + z2 - z3 + ... + z n-1
2
(a) w 9 - 1 = (w - 1)(1 + w + w + ... + w ) = 0
8

(b) Use the sum of the roots. F3


(a) integers (b) real
Q14

(a) -1, cis ( ±~) , c i + 3; ) , cis ( ±5; ) Ql


(a) z 2 - 2z + 2 (b) z2 - 6z + 13
(b) Use the sum of the zeroes.

Q15 Q2
X
2
+X +2= 0 (z 2 - 2Re (a) z + lal 2) (z 2 - 2Re (/3) z + J/31 2)
Q16 Q3
2 Expand the right-hand side and simplify.

Q17 Q4

Instead solve +
(~
1) 8
= 1
(a) (z + 1)(1 - z + z 2 - • • • + z 6 )
(b) (z -l)(l+ z + z 2 +-··+ z 6 )
Pl 2 4 6
(c) (z - l)(z + 1)(1 + z + z + z )
1 + wn + w 2n = 3 when n is a multiple of 3,
otherwise it is equal to zero.
258 Answers

Q5 QlO
P(z) = (z 3 - l)(z 6 + z 3 + 1) , so the zeroes of
(a)
2
cis ( ± ; ) , cis ( ± 4; ) (a)
z 6 + z 3 + 1 are a subset of the zeroes of P(z).
(b) Use (z - a)(z - a)= z 2 2Re (a) z + lal 2
(c) Equate the coefficient of z .
-
2
(b) cis ( ± ; ) , cis ( ± 4; ) , cis ( ± s; )
(d) Equate the coefficient of z 2 . (c) Use the result
(z - a)( z - a)= z 2 - 2Re (a) z + lal 2 three
(e) 4x 2 + 2x - 1=0 times.

(f) cos ( 27r) -_ -1 + J5 , (d) Substitute z = 1 into the previous identity


5 4 and use double-angle formulae.
4
cos ( : ) = -l ~ J5 (e) Substitute z = i to the same identity.
(f) Divide the two previous results, and note
(g) Substitute in z = 1 and use double-angle

formulae.
that cos (8;) = -cos m
Pl
Q6 (a) Find explicit expressions for wk and wk-l
Show that Pn(w) = Pn(w 2 ) = 0 so (x - w)(x - w 2 ) and calculate.
must be a factor , but this expands to become (b) The perimeter is n lots of the side length.
(x 2 +x+l).
(c) See full worked solutions.
Q7
(d) The circumference of the unit circle is 2w.
(a) Use de Moivre's theorem .
(b) Find all the roots of z 4 + 1 = 0 and then use
P2
(z - a)( z - a) = z 2 - 2Re (a) z + lal 2 two
times. (a) Use the distance formula.

(c) Divide both sides by z 2 and use the fact (b) First show that d~ = 2 - 2 cos k0 and then
that if z = cis 0 then zn + z-n = 2 cos n0 use the fact that wk + ~;k = 2 cos k0
(c) Sum the d~ terms and use the fact that
Q8 1+ w+ w2 + · · · + wn-l = 0
(a) Use de Moivre's theorem.
(b) Find all the roots of z 6 + 1 = 0 and then use
(z - a)( z - a) = z 2 - 2Re (a) z + lal 2 three
times.
Exercise 2J
Solving Polynomials
(c) Divide both sides by z 3 and use the fact
that if z = cis 0 then zn + z-n = 2 cos n0
Fl
Q9 conjugate
(a) P( z ) = + l)(z - z + 1) , so the zeroes of
(z 3 6 3
F2
z 6 - z 3 + 1 are a subset of the zeroes of P(z).
z2 - 2Re (a) z + lal 2
(b) cis ( ± iD ,cis ( ± 5; ) , cis ( ± 7; ) F3
b C d
(c) Use the result (a) (b) - (c)
a a a
(z - a)(z - a) = z 2 - 2Re (a) z + lal 2 three
times .
F4
(d) Divide both sides by z 3 and use the fact b C d e
that if z = cis 0 then zn + z-n = 2 cos n0 (a) (b) - (c) (d) -
a a a a
Answers 259

Ql (e) p=-1,q=4
(a) The coefficients of the polynomial equation
are real, so we can use the conjugate root Q9
theorem.
7x + 1
(b) x=-3
QlO
(c) P(x)=(x+3)( x - 2 - i)(x-2+i) 11

Qll
Q2
X= -1 ± i, -1 ± 2i
(a) x=l±i,-1 (b) x = ±2i, 3
Q12

Q3 (a) La = (Laf -2LafJ


2

(a) x = 1 - i, since the coefficients are real so (b) a 2 + ;3 2 + ,y 2 + b2 < 0 which cannot happen
we can use the conjugate root theorem. if the roots are all real.
(b) x 2 - 2x + 2 (c) a2 + ;3 2 + ,y 2 + 62 > 0 does not guarantee
(c) x 2 +x-12 that all four roots are real. For example if
a = i , jJ = -i and ,y = 6 = 5 then
(d) x = 1 ± i, -4, 3 2 2
a + JJ + ,y + b > O but the roots most
2 2

certainly were not all real.


Q4
(a) X = 1 ± 2i, 2 ± i Q13
(a) Solve for x 2 and notice that it 's always a
(b) x = 1 ± i, ±2i
negative number. Hence x is purely
imaginary.
Q5 (b) -ai, -/Ji
(a) P(l + i) = P'(l + i) = 0. Note that the
easiest way to calculate P(l + i) is to first (c) La = (Laf -2LafJ
2

convert to polar form.


(b) x = 1 ± i, 1 ± i, 1 Q14
(c) 2
P(x) = (x - 1 + i) (x - 1 + i) (x - 1) 2
(a) P(a) = jJ and then use conjugate properties
(d) P(x) = (x 2 - 2x + 2)(x - 1) to eventually deduce that P(a) = /3.
(b) The conjugate root theorem is a special case
of the property for when jJ = 0.
Q6
x 3 - 2x 2 + x + 1 = (x - i) (x 2 - (2 - i)x - 2i) + 3
Q15
Q7 (a) P(-1)=0
(a) See full worked solutions.
(b) Yes , the remainder theorem also works for
(b) PG) = 0 given P(a) = 0.
1
complex numbers. (c) The two non-real zeroes are a and -. But
a
(c) P(-i)=3+3i by the conjugate root theorem, this means
1
that a = - and so aa = 1. Hence all zeroes,
a
Q8 including the real root, have modulus 1.
(a) The remainder is at most one degree less
than the divisor. Since the divisor is Q16
quadratic, the remainder is at most linear (a) Show that a 2 + ;3 2 + ,y 2 < O
i.e. ax + b.
(b) Use the product of the zeroes.
(b) P(x) = (x 2 + l)Q(x) + ax + b
(c) Use the sum of the roots.
(c) P(i) = -5+3i
(d) Use the sum of the roots in pairs.
a= 3, b = -5
(d) 3x - 5
260 Answers

Q17 Exercise 2K
(a) They are the complex conjugates of a and Euler's Formula
ia.
(b) Either re-arrange P(z) or expand the given Fl
result. cos 0 + i sin 0, exponential
(c) Note that P (z) is the sum of perfect
F2
squares. So the only way it will be zero is if
1 (a) cos 0 (b) sin 0
both parts are zero, so k = k i.e. k = ±l.
(d) Consider two cases. One is if P (z) has real Ql
zeroes , in which case actually find the zeroes
(a) 2eT (b) 2)2e-if
and show that they are on the unit circle.
The other case is if P( z) does not have real
zeroes, in which case use the product of the Q2
zeroes and use the fact that lal 2 = aa. (a) 2 + 2iv'3 (b) -v'3 - i (c) 1-i
(e) Use the sum of the zeroes.
(f) Since the zeroes lie on the unit circle, they Q3
are in the form x + iy = cos0 + isin0. So 1
k = x - y = cos 0 - sin 0 and use
(a)
2 (cis( n0) + cis( -n0))
auxiliary-angle formulae. 1
(b) i (cis(n0) - cis(-n0))
2
Pl
Q4
See full worked solutions.
(a) 2 cos 20 - 2 (b) 2 cos 20 + 2
P2 (c) 2i sin 60 - 6i sin 20 (d) 2 cos 60 + 6 cos 20

(a) PG) = 0 given P(a) = 0. Q5


(b) Suppose all zeroes are inside the unit circle. (a) Factorise out eie

1 (b) Factorise out ei 0


But from the previous part, - must also be
a
1 Q6
a zero. But the modulus of - exceeds one
a (a) i tan 0 (b) e-~ x itan0
and so at least one root is outside of the
unit circle.
Q7
(c) Substitute z = a and re-arrange, then take ei0 _ e-i0
1
the modulus of both sides. (a) tan 0 = -:- X . .
e2 0 + e-i 0
i
(d) Use the fact that lal > 1. Substitute (b) See full worked solutions.
a = Ia I cis 0 into the previous part and
expand then simplify and factorise.
Q8
(e) Ia I > 1 and -1 :::; k :::; 1 so Expand and simplify
(lal 2 - 1) ( 1 - k2 ) 2:: 0. So any root lying ( ei0 ~ e- '0) + ( e'0 ~ie-i0)
4 4

outside of the unit circle will result in a


contradiction. Hence, the only possible
scenario left is that all the zeroes lie on the Q9
unit circle rather than inside or outside it. z = reie so zn = rnenie = rn ( cos n0 + i sin n0)
QlO
ei0 + e-i0
(a) cos 0 =
2
(b) Cube both sides and simplify.
Answers 261

Qll R8
Start from the right-hand side, convert to (a) cos 20 - i sin 20
exponential form , then expand.
(b) i cot 0
Q12
(a) Find two different expressions for
Re ( ei(a+,B)) R9
(b) Find two different expressions for (a)
Im ( ei(a+,B) ) iy

A
.
Pl
(a) Expand cos(A + B) + cos(A - B) where
X
A = e+ </> and B = e- </> . Similarly for the
2 2
sine expression.
(b) ei 8 + ei<P
= (cos 0 +cos </>) + i(sin 0 +sin</>) (b) C 1 = 6 + i, D 1 = 4 + Si, C2 = -2 - 3i,
then use the previous result. D2 = -4+i

P2
RIO
(a) Use the sum of a geometric progression.
i ( n+l) 0 (a) BC x i = BA
(b) Factorise e-2- from the numerator and
e14- from the denominator. (b) Z4 = Z1 + Z3 - Z2

(c) Find two different expressions for Im (Sn). (c) See full worked solutions.

Rll
Chapter Review B = 3 + Si, D = 5 + i

Rl R12
(a) 10-55i (b) 2+i
(a) vf2 cis ( ~)
R2 (b)
iy
See full worked solutions. z +w
R3
z
See full worked solutions.
/
/

R4
±(5 + 7i) X

R5
(c) See full worked solutions.
(a) z = 3 ± 2i (b) z = 3 - 2i , 4 + i
(d) l+v/2
R6
(a) - 1- i\/'3 (b) 2\/'3 - 2i R13
(a) y=l (b) x= O
R7
(c) (x - 2) 2 + y 2 = 4 (d) y = 2x
(a) /0
2v0cis (
- 21r) = 2v3e-~ /0 2i7r
x2 - y2 = 2
3 (e) xy = 2 (f)
262 Answers

R14 R20
(a) (b) (a) Consider (cos0 + isin0) 4 .
iy iy
(b) x =±cos(~) , ±cos G;)
2 X
X
( C) COS ( ~ )
12
= )3 + l
2yl2 '
COS (
5
1r) =
12
)3 - l
2yl2

-3 R21
(a) Consider (cos0 + isin0) 3 .
(c)
iy
(d)
iy (b) x = cos(2;), cos ( ; ) , cos
4
(8;)
(c) Use the sum of the roots , and then
complementary angle identities.
1
X

2 X R22
3
(a) -1 , cis ( ±i) , cis ( ± ; )

(e) (f)
(b) cis (k:) where k= -4, -1 , 2, 5
iy iy
R23
(a) w 7 = 1 so w 7 - 1 = 0 then factorise.
3 X 1 (b) Group the identity into conjugate pairs and
use the fact that a+ a= 2Re (a).
1 3 X

R24
(a) 9 (b) 3
R15
x2 + y2 = 1 R25
(a) Find all the roots of z 6 - 1 = 0 and then use
R16 the result
(a) 2yl2 + 1 (b) 2yl2 - 1 (z - a)(z - a)= z 2 - 2Re (a) z + lal 2 three
times.
(b) Divide both sides by z 3 and then use the
fact that a - a = 2ilm (a).
R17
R26
Multiples of 6

R18 (a) (z + 1) ( z2 - 2z cos ( i) + 1)


1 5
Consider (z + z- ) to prove a result for cos 0, and 5
7r
then replace 0 with
(z - 5
2
- 0. Alternatively, consider
z- 1 ) . Note that the alternative method does
(z -
2 3
2z cos ( ; ) + 1)
(b) (z + 1)(1 - z + z 2 - z3 + z4 )
not work for even powers of sine.
(c) Equate the coefficient of z .
R19
(d) Equate the coefficient of z 2 .
(a) zn + z- n = cis nf + cis(-n0) = 2 cos n0
(e) 4x 2 - 2x - 1 = 0
(b) z =±i, 2±)3
(f) cos ( ~) = l + v15 cos ( 3
5 4 ' 5
1r) = l - 4v15
Answers 263

R27 Q2
(a) x = l ± 3i , 2 (b) x =2±i, l±i (a) 6i + 4j__ + 4}s (b) lli - 2j__ + ls
(c) -2 (d) 12
R28
2x - 7 Q3
R29 (a) 60° (b) 101 ° (C) 70°
(a) a 2 + /3 2 + ,. y2 = - 52 < 0, so all three roots
cannot be real.
(b) x = 2 ± 3i, 4

R30
(a) x = l ± i , 1 ± 2i Q5
(b) P(x ) = (x 2
- 2x + 2) (x 2
- 2x + 5) a = l , b = 2, c = 3

Q6
R31 82°, 33°, 65°
(a) 2cos30 + 6cos0
Q7
(b) 2 cos 80 - 8 cos 40 +6 (1 , -2, 2) , (-3 , 10, 2) , (3 , 2, -8)

Q8

q~
R32

~ rn
e i0 _ e-i0
(a) sin 0 =
2
i
(a) 2] (b)
(b) Cube both sides and then convert to sines.

(c) A (d) A
3. 3D Vectors Q9
(a) U=V (b) lul = lvl
Exercise 3A
Introduction to 3D Vectors
QlO

20
Fl (a)
(a) + ka3}s
ka ii + ka2j__ J30
(b) 5\/'2
(b) (a1 + b1)i + (a2 + b2)j__ + (a3 + b3)}s
a1b1 + a2b2 + a3b3
(c)
(d) ar + a~ + a~
(c) f¥
(e) ✓ ar + a~ + a~ 10
(d) -JIT
3
1
(f) ✓ ( aii + a2 j + a31,) 1
2
a 1 + a2
2
+ a 23 (e) Area = -ab sin 0
2
F2
U ·V U·V Qll
(a) (b)
lul 1u12u
Ql
(a) 4 (b) 3 (c) 7
~=M rn
Q12
(d) v'65 (e) 5 (f) v1s8
See full worked solutions.
264 Answers

Q13 Exercise 38
(a) IABI = IACI = 6 Proofs using 3D Vectors
(b) Show that AB · AC f. 0
Fl
(a) Show that their dot product is zero.
Q14
(a) lgl 2 + 11!,12 + 2g · r (b) Show that they are scalar multiples of each
other.
(b) lgl 2 + lrl 2 - 2g ·r
(c) Show that the vectors that pass through the
(c) 21gl 2 + 2r1 2 points are parallel, but using a common
(d) 4g · Q point.
U·V
(d) Use the formula cos 0 = ; ;
Q15 1 11 1
(e) Find the mid-point M of one interval. Show
(a) g ·r = lgllrl cos 0 ::; lgllrl that the vector that makes the other interval
also passes through M.
(b) Prove instead the squared version of the
required result.
Ql
(c) This is the triangle inequality.
Use Pythagoras' Theorem twice.
Q2
Q16 7r
7r Let all the angles be equal to and observe that
(a) 3
2 a= b = c.
7r
(b) Both are equal to
2 Q3
2 (a) (1,1 , 1)
(c) LAPE= cos- 1 (
✓b2
c
+ c2 ✓a2 + c2
)
(b) G,~,D
LBPC = cos- 1 a2 ) (c) The exact angle is cos- 1 ( - ~ ) "' 109.5°
. ( ✓a2 + b2 ✓a2 + c2
b2 ) Q4
LAPC = cos- 1
( ✓a2 + b2 ✓b2 + c2 (a) (0, 0, av'2)
(b) tan - 1 ( ✓2)
Pl
Use the formula for the vector projection.
Q5
See full worked solutions.

Q6
(a) lg-QI=
P3 ✓ (u1 - v1) 2 + (u2 - v2) 2 + (u3 - v3) 2
(b) Square both sides then use the fact that
lg - rl 2 = (g - 1!) · (g - 1!)
P4 Q7
Find two vectors g and Q along the plane that are First show that g = cos a i ± sin a i_, and
not parallel. Show that '!} is perpendicular to g and Q = ± sin /3 j + cos /3 15
Q.
Q8
See full worked solutions.

Q9
See full worked solutions.
Answers 265

QlO P3
(a) Since it is perpendicular to the face opposite (a) Instead, minimise i?:YJ 2 = Jg - tQJ 2
l:::.Q RS, it points vertically downwards. The (b) This proves that the minimum distance
area is just the area of the right-angled between a line f, and a point P is the
l:::.QRS. perpendicular distance between them.

P4
Show that 'Q · ~ = ci(~ · ~) and re-arrange.
s= (½pr) i + (½ pq) l + (½qr) k
(c) Add the previous expressions and everything
cancels. Exercise 3C
Vector Equation of a Line
Qll
(a) g=ai-bj-ck Fl
Q=ai - bj+ck (a) parameter (b) parameter (c) position
(b) g . 'Q = a 2 + b2 - c2, which
is zero iff the
diagonals are perpendicular. F2
(c) P(3, 4, 5) (a) 0, 1 (b) Q (c) Q

Q12 Ql
(a) (r_ - r,0 ) lies on the plane and n is
perpendicular to the plane, which means (a) r_(,\) =
that it is perpendicular to any vector on the
plane. So their dot product is zero. - 3- 2,\]
(b) Expand (r_ - r_o) · n = 0
(b) r_(.\) =
[ 15++4,\,\
(C) 3x - 2y + Z = 13
2 + 4,\ ]
(c) r_(,\) =
(d) Ja 2 + b2 + c2 [- 3 ~ 3,\
Q13 (d) r_(.\) =
See full worked solutions.

Q14 Q2
See full worked solutions. (a) (b)
y y
Pl
(a) See full worked solutions.

(b) (E1 + E2 + E3 + E4) · (E1 + E2 + E3 + E4) = O


4 6 X
(c) Find a general expression for
=--=+ =--=+ =--=+
JPPkl 2 = PPk · PPk and then sum them and
use the previous result.

P2 Q3
b
Show that the given expression is equivalent to (a) m=-
a
Y:,·'Q
JgJJ'QI (b) r_(.\) =
266 Answers

Q4 Q13

(a) r(A) = [ r] [!
+A
5
]
(a)
(b)
(4,4 , 2)
(1 , 0, 2)

(b) r(A) = [ r] + A [ ~O;]


Q14

(a) r(A) = A[~]


Q5

(a) r(A) = [ ~ 2] +A [ !:] wherd E [O, 1] r(µ) = [!] + µ [ ~ 1]


(b) r(A) = m ~2] +A [ wherd E [0, 1]
(b) Show that A=µ and A+µ= l , hence
1
A=µ=2
Q6
Q15
(a) Collinear.
(b) Not collinear.
Iii
Q16
Q7 (a) Their direction vectors are scalar multiples
(a) Yes (b) No (c) Yes of each other, and hence are parallel.

Q8
(b) ff
(a) >. = -1
Q17
(b) (- 4,8,0)
(c) (-1, 2, 12) r(µ) = [ ~2] ?o]+µ [

Q9 Pl
(a) (1, -2, 3), (3, 1, 2) (a) Obtain x = x 0 + >.a and similarly for y and
(b) Does not lie on .e. z. Equate their A expressions.

(c) Lies on .e. (b) This gives the Cartesian equation of the
line. More specifically, the x y, y z and
(d) (0 , -3.5, 3.5) xz-proj ections can be obtained from this by
excluding one of the expressions.
QlO
P2

(a) PQ = [µ : ; ~ 1]
->.-1
Qll (b) 1 unit .
Skew.
P3
Q12
(a) r(>.) = E + >.1?,
3
(a) ~
r(A) = [ ] + A [ 6] ~ (b) Let r.,
(c)
= Q and use the fact that
.e is tangential to S
g ·g
if and only if there is
= 1Yol 2

(b) (1, 0, 3) exactly one point of contact i.e. A = 0 is the


only solution. This occurs if and only if
l?, · (E - £) = o
Answers 267

Exercise 3D Q4
Parameterising 3D Curves x 2 y2
(a) -+-=1
b2
a2
(b)
Fl y
parametrical ly, xy, xz , yz, perspectives
F2
(a) parameter, segment
a X
(b) positive, quadrant

(c) Replace t with -t.


Ql
• 7r
(d) Replace t with t + .
(a) y = 2x + 5 2
1
(b) y=-(x+1) 2 Q5
4
8a 3
(c) y = x + 1 where x 2'. 0 (a) y = x2 + 4a2
(b)
x2 y2 y
(d) -+-=1
16 9
x2 y2
(e) ---=1
9 4
X
1
(f) y = - where x >0
X
Q6
Q2 The first shows the full curve. The second only
Plane xy Plane xz shows the first quadrant since x 2'. 0 and the third
excludes the origin since neither the x nor the
y-coordinate can be zero.

Q7
(a) The spiral stretches out in the z-direction as
X
t increases. Also it is only defined for z 2'. 0.
1 X

2 (b) It is the same as the spiral from the previous


z = (x - 1)
part , except it spirals down the z-axis
Plane y z instead of up.

z (c) The spiral stretches out even faster in the


z-direction as t increases.
(d) The spiral gets 'compressed ' in the
z-direction as t increases. As t ➔ 0, the
y spiral has a 'tail ' that shoots down to
- 1
negative infinity.
(e) The spiral starts at z = 1 when t = 0, then
Q3 spirals downwards indefinitely towards z = 0
(a) x 2 + y2 = 2 but never touches the xy-plane.
(b) x 2 + y 2 = 2 (f) The 'spiral' oscillates up and down in the
(c) The first one starts at ( 1, 1) and goes z-direction whilst still maintaining a circle
clockwise as t increases. The second one in the xy-plane. So actually it is an ellipse
starts at ( v'2, 0) and goes anti-clockwi se as t that is slanted with respect to the xy-plane.
increases.
268 Answers

Q8
(a) x 2 + y2 = 1
Q14
(b) z = 1- y
(a) r,(t) = (2sint)i + tj + (2cost)]5
(c) x2 + (y - 1) 2 = 1
(b) r,(t) = (3 - 5t)i + (6t - 6)j__ + (3t + 1))5
(d) It represents a slanted ellipse with respect to
(c) r,(t) = ti + (t 2 )j__ + (2t 2 )}5
the xy-plane.
(d) r,(t) = (cost)i + (sint)j__ + (2 - sint)k
Q9
(a) T his represents a straight line. Q15
(b) This represents a slanted parabola. (a) (iii) (b) (i) (C) (vii) (d) (viii)
(c) This represents a spiral that wraps around (e) (v) (f) (vi) (g) (iv) (h) (ii)
the x-axis.
(d) This represents a slanted ellipse. Q16
1
z. The
2 (x + 1)
(a) Equate parabola y = 2
is
QlO
(a) y = x 2 , z = x 3 , y = z l the projection of the curve of intersection
(b) onto the xy-plane.
Plane xy Plane xz
(b) See full worked solutions.
(c) No , it is not wrong to use the other surface.
We choose z = y - 1 because the algebra
will be simpler.
X
(d) -3 :S t :S 3. Intersections at (-3, 5, 4) and
X (3,5 , 4).

Plane yz Q17
(a) r,(t) = (t)i± ✓ 1 + 2t2, + (1 + t) ~

(b) See full worked solutions.


(c) The first parametrisation works, but it has a
y
plus/minus in it. So it can only give one
branch of the parabolic intersection at a
time. However, the first parametrisation
gives the entire parabola in one go.
Qll
The first particle begins at (1, 0) and moves Q18
anti-clockwise at a certain speed. The second
(a) ( cos 0)i + ( sin 0)j
particle does the same thing, but traverses the circle
at twice the speed. The third particle begins at (b) ( cos 0) i + ( sin 0)j + (1 - sin 0) J5
(0, 1) but travels clockwise at the same speed as the (c) It represents the curve of intersection
first particle. between the two surfaces, which is an
Q12 ellipse. From the birds-eye view it is the
The particles will collide when t = 1. t his is found unit circle. In the yz-plane, it is the straight
by equating any component , solving for t , and then line z = 1 - y.
testing the other components.
Q19
Q13 (a) See full worked solutions.
(a) 32,(t) = (6t 2 + l)i - (2t)j__ + 3)5
g(t) = (12t)i - 21
(b) (1 + vf2 cos t) i + ( - 1 + vf2 sin t )j
(c) See full worked solutions.
(b) 112,(l)I = v162
jg(l)I = 2yll45
Answers 269

Q20 Q3
d t) = t i + t2 i + (t2 + t4 H (a) C(2, -1 , 0), r = 3
-v13 s t s v13 (b) C(-1 , 2, 3) , r = 4
Pl
(a) See full worked solutions.
Q4
(b) It traces out a circle.
(a) + (y + 2) 2 + (z - 3)2 = 9
(x - 6) 2
(c) It defines t he same circle as before, but now
centred at the fixed point represented by£· (b) (x - 6) 2 + (y + 2)2 + (z - 3) 2 = 4

(c) (x - 6) 2 + (y + 2) 2 + (z - 3) 2 = 36
P2
1
(a) x2 = t2 + t2 + 2 = y + 2
(b) i
Recall the inequality t + ;2: 2 fort > 0. So
Q5
(x - 2) 2 + (y - 4) 2 = 27
the curve is actually the parabola but for (y - 4) 2 + (z - 3) 2 = 32
x ;2: 2 or x S -2. (x - 2) 2 + (z - 3) 2 = 20

Q6

Exercise 3E (x - 3) 2 + (y - 5) 2 + (z - 2) 2 = 4
Spheres and Circles Q7
(a) (x - 1) 2 + (y - 2) 2 + (z - 4) 2 = 4
Fl
(x - xo) 2 + (y - Yo) 2 + (z - zo) 2 = r2 (b) C(l, 2, 4), r = 2vl2

F2
£, r Q8

F3
x2 + (y - 1) 2 + (z - 2) 2 = 4
(a) Find the distance between P and the centre Q9
of S and compare it to the radius .
(b) Find the vector equation of Rand substitute
it into the Cartesian equation of the sphere.
(a) r( A) = [!~];A]
Solve the resulting quadratic for the
parameter A and substitute the solutions (b) (x - 2) 2 + (y + 1) 2 + (z - 3) 2 = 9
back into R to find the point of intersection.
(c) ,\ = 1, ~
(c) Find the distance from C to P to find the
radius, and the Cartesian equation can now
10 12
be obtained since the centre and radius of S (d ) (4 0 1) and (
' '
-
7 ' 7 '7
!)
are known.
(e) If the quadratic had no real roots , then no
(d) The length of AB is the diameter and so the
real value of A exists i.e. the line does not
radius can be found. The midpoint of the
intersect the sphere. If there was only one
interval is the centre of S . Find the
real root , then it means the line is tangential
Cartesian equation from here.
to the sphere and hence intersects it at
exactly one point.
Ql
(a) (x - 2)2 + (y + 3) 2 + (z - 5) 2 = 36
QlO
(b) (x - 2)2 + (y + 3) 2 = 11
(x - 2) 2 + (z - 5) 2 = 27 (a) r =1
(y + 3) 2 + (z - 5) 2 = 32 (b) A = -1. This was to be expected since R is
tangential to S , so there should only be one
Q2 point of intersection.
(x + 5) 2 + (y - 3) 2 + (z - 4) 4 = 50
(c) P(2 , 0,- l)
270 Answers

(d) The dot product is zero, so the two lines are Chapter Review
perpendicular. This is to be expected since
this is just the 3D analogy of the property Rl
"The tangent and radius are perpendicular 1T
at the point of contact" from circle
2
geometry.
R2
Qll (0, 2, 4) , (- 4, 14, 4), (2 , 6, - 6)
(-2 , -1 , 2) and (2 , 1, 2)
R3
Q12
(a) Show that the distance between their
centres is equal to the sum of their radii.

R4
(b) (-! ~ -~)
3 ' 3' 3 Find expressions for the vectors and find the angle
(c) between the vectors.
10

(d) u 3'
_11
3 ' 3
14)
R5
(2, -4, 12)

R6
Q13
(a) Q = (r cos 0)i + (r sin 0):2
(b) The parametrisation above effectively
r(A) = [ Y] +A [] J where A E [O, l]

consists of two perpendicular vectors of R7


length r and combines cos 0 'amount' of one Parallel.
of them with sin 0 'amount' of the other one.
The parametrisation at hand here does the R8
same thing, except the basis vectors are now Perpendicular.
11:, and Yd instead of i and j__. The.£ vector just
R9
shifts the whole thing to be centred at_£.
(a) Equate two components and solve
(c) Show that l11:,cos0 + 7l:!Sin01 2 = r 2 simultaneously for µ and >-.. Test the
solutions with the third component to check
that it works.
Q14
Show that the coordinates satisfy x 2 + y2 + z2 = r 2 . (b) (-2,-4,-1)

Pl
See full worked solutions. RIO
(4, 1, -9)
P2
(a) See full worked solutions. Rll
(a) Equate two components and solve
(b) It describes a circle with diameter AB.
simultaneously for µ and >-.. Test the
(c) It proves t hat the set of all points that form solutions with the third component and
perpendicular vectors with respect to two show that it does not work.
fixed points is a circle. In other words it
proves the familiar 'angle in a semi-circle '
property otherwise known as Thales '
Theorem.
R12
r.,(t) = (t 2 - 2t)i + tj + t 2 }s
P3
See full worked solutions. R13

r(t) = (~cost) i + ( ~ sint) ,t+ 1-&


Answers 271

R14 Q2
C(-2 , 1, 3) , r=4
(a) ✓ l+ x 2 +C
R15
(a) (x - 2) 2 + (y - 3) 2 + (z - 4) 2 = 4 (b) l . +C
cos x + sm x
(b) (x - 2) 2 + (y - 3) 2 + (z - 4) 2 = 25 2 5 2 3
(c) (x + 1)2 - (x + 1)2 + C
5 3
R16 (d) 1 1 +C
(4, 1, -3) and (1 , -5 , 0) 4(x 2 + 1) 2 2(x 2 + 1)

R17 (e) -~(2 - x) 7 - ~(2 - x)6 +C


7 2
(a) Note that the centre of S 2 lies inside S 1 . 2
- 2v'x+l + C
Show that d + r2 = r 1 , where d is the (f)
3(x + l)v'x+l
distance between their centres.
1 +C
(b) ( - ~ 13 _
3' 3 '
10)
3
( )
g 2 cos 2 x

1 2
(i) - (ln x) + C
4. Further Integration 2

(j) 2✓3 - cos x + C


Exercise 4A
Integration by Substitution

Fl
(a) _l_(f(x)f+l+C
n+ l Q3
(b) ln IJ(x)I + C 1 2) + C
(a) -
2 cos(x
(c) e f( x ) +C
(c) -tanG) +c (d) - 2 cos (fi) + C

1
sin ( ln x) + C tan x) + C
F2 (e) (f)
2 tan (2
(a) - cos (f (x)) + C
(b) sin(f (x)) + C
(c) tan (f( x)) + C (b) - ecos x +C

(c) -e½+c
F3
(a) sin - 1 (J (x)) + C (b) tan - l (J (x)) + C 1
(e) --
2 e x2
+c (f) etan x +C

Ql
Q5
(a) u = x 3 (b) u = x2 (a) - ln I1 + cos x I + C (b) ln I ln x I + C
(c) u = 1 + ex (d) u = 1 + cos x (c) ln Ix -tan x l + C (d) ln !ex + e-x l + C
2
(e) u = 1 + x (f) u = 1 + tan x (e) 2lnjl+fil+C (f) -lnjl+cos 2 x l+C
272 Answers

Q6 P3
1
(a) eex + C
2 sin- (x )
1 2
(a) +C

(b) ~ sin- 1 (2s~nx) + C (b) ln !ex - e-x I + C

(c) sin - l (ex ) + C (c) 2lnll+Jxl +c

(d) 1 . _ 1 ( 5 tan x )
5sm - 4- +
C G)+c
(d) -sin - 1

(e) sin - l (ln x) + C (e) -2J1:x+c


(f) -sin- 1 m +C (f) tan- 1 (x) +

11 - ✓1 - e x
1
3
tan- 1 (x 3 ) + C
2
(g) -1 ln I
+C
Q7 2 1+ ✓1 - e2x
1 1
-
4 ln Ix- + 1I + C
4
(a) tan- 1 (x 3 ) + C (h)
3
(b) ~ tan - 1 (3 s~n x) + C (i) 2ft - 2 tan- 1 ( Jx) + C

(c) -tan- 1 (e-x) + C (j) v'2tan- 1 ( ✓:,; 1) +C


(d) ~ tan - 1 (2 x) + C
t~n (k) sin- 1 ( x - D+ C

2
(e) tan- 1 (lnx) + C (1) sin- 1 (xJx) + C
3
(f) 2 tan- 1 ( Jx) + C

Exercise 48
Q8 Trigonometric Integrals

(a) - ln I cos (ex) I + C (b) - 2 ln I cos (vx) I +


Fl
C
_ l_(f(x)r+l + C
n+l
Ql
1 1 4
(a) sin 3 x + C (b) -- cos X +C
2ft - 2 ln I1 + vxl + C 3 4
1 1
-2Jx- 21n 11 - vxl + C (c) 5
x+C 5
5 tan (d)
5 sec x+C
X - 2ft + 2 ln 11 + vxl + C 1 2
2 3
(e) (f)
2 tan x + C 7 sec xJsecx+C

Q2
1 3
(a) x - cos x + C
3 cos
1
(b) sin x - sin 3 x + C
See full worked solutions. 3
1
2 tan x + In Icos x I + C
2
(c)
Answers 273

Q7
(d) tan x - x + C 2 2
sin x C cos x C cos2x C
1 (a) -2- + ' - - 2 - + ' - - 4 - +
(e) tan x + tan 3 x + C
3 (b) The three 'different ' answers differ by
1 constants, which can be proven using the
(f) x - tan x + - tan 3 x + C double angle identities.
3

Q3 Q8
1 2
(a) - - cos 5 x + - cos 3 x - cos x + C
5 3
2 1
(b) sin x - - sin 3 x + - sin 5 x + C
3 5
1
(c) - sec 4 x - sec 2 x + ln I sec x I + C
4

Q4 Q9
1
2x I + C
2 ln Icos
(a) ~(12x - 8sin2x + sin4x) + C (a) -
32

(b) ~(12x + 8sin2x + sin4x) + C


32
1 .
(b) -Hsin(Hl+c
(c) (12x + sm4x) + C (c) ln I sec x I + C
16

Q5 (d) - 2 cos x + C
1 3
x + C
(a)
3 sec Pl
1 3 2 2
x+C
(b)
3 tan ( )
1
tan- 1 (Ja + b tanx) + C
a aJa2 + b2 a
1
(c) - sec 3 x - sec x + C
3 1
(b) tan- 1 ( a tanx) + C
2 + 2 Ja 2 + b2
(d) -1 sin 3 x - -1 sin 5 x + C aJa b
3 5

(e)
1
tan 4 x + C
(c) ~
ab
tan- 1 (~ tanx)
a
+ C
4
1 2 1 cosx C
(f) - - cos 3 x + - cos 5 x - - cos 7 x + C (d)
3 5 7 b(acosx + bsin x) +

Q6 P2
7r

(a) ln I tanxl + C Ja2 - b2

1 P3
(b) 2 co s 2 x +C See full worked solutions.
cos 2 x
(c) - - + ln I secxl + C
2
sin 2 x Exercise 4C
(d) ln I sinxl - - - + C Trigonometric Substitutions
2

(e) ( ~ cos 2
x- 2) ✓ cos x +C
Fl
(a) a sin 0 (b) a tan 0 (c) a sec 0
(f) tan x - cot x + C
274 Answers

Ql Q7

(a) 2 - vf2 (b) ~ - ~ (c) -~ ( ) X +C ( ) X +C


3 2 a 4J4- x 2 b 4J4 + x 2

Q2 (c) X +C
4Jx 2 - 4
J4-x 2 Jx2+4 C
(a) 4x +C (b) Q8
4x +
Jx 2 -4 (a)
2
-xJa- -- x + -sin-
2
a (x) +c
2
1
-
(c) 4x +C 2 2 a

½(x✓x 2 +a 2 +a2 ln1 x+ ✓:


2
+a I) +C
Q3 2
(b)

(a) -H2+~1 +c (c) ½(x✓x2 -a2-a 2 Jn1 x+ ✓:


2
-a I) +C
2

(b) -Hx+7 l+ c Q9

(a) sin - 1 ( ~) +C

Q4 (b) ln Ix+ ✓:2 + 2I + a C

~ . 1(X) + C
(a) - x - sm -
2 (c) ln Ix+ ✓:2 - 2I + a C

(b) ln1x+71 +C
QlO

(c) ~+lnlx+~l+c (a)


a2Ja2 - x2
x +C (
b
)
a2Ja2
X
+ x2
+C

(c)
Q5

2
Pl
(a) J4 - x 2
- 2 ln 2 + J4-
x x I

+C 7r ~ 7r
(a) --- (b) - 1
1
6 4 2
(b) J 4 + x2 - 2 ln 2 + J4
x + x2 + C I
(c) 2- -
7r
(d) 2 ln(l + v12) - vl2
2
1

7r
(e) - (f) 21r - ~
4

P2
Q6
See full worked solutions.

(2x) - x +C
2
(a) 2 sin - l xJ4 - P3
2 7r
-(b - a)
2
Ix+ v~
x +I4 + C
2
(b)
xJ4 + x 2
- 2 ln
2

(c) 2 ln x+~1+ 2
x~
2
+C
l
Answers 275

Exercise 4D Q4
Harder Standard Integrals
(a) x - ln lx 2 + 2x + 21 + tan- 1 (x + 1) + C

Fl 1
(a) Express as a perfect square and use the (b) x+ ~tan- 1 (2x; )-lnjx 2 +x+ll
reference sheet formula for
J J'(x) (f (x) f dx.
(c) 2x + 5 tan - l (x + 2) - 4 ln Ix 2 + 4x + 5 I + C

(b) Complete the square and it is an inverse tan


integral. Q5
(c) Use partial fractions 3
(a) sin - l ( x ; ) +C
F2 (b) sin- 1 (x - 2) + C
logarithmic
F3 (c) sin - 1 ( x + 1) + C
inverse sine, trigonometric
F4
JJ' ( X) (J (X) r dx
Q6

(a)
3
sin- 1 (2x - 1) - ✓x - x2 + C
Ql
2
1 (b) sin- 1 (x-1)- ✓2x-x 2 +C
(a)
4 (2x - 1 + ln I2x - 11 + C)
x2
(b)
2 - x + ln Ix+ 1 I + C

(c) 2x + 3 ln Ix - 1 I + C Q7
2
d) x + x ln I2x - 1 I
( -4-+ s +c (a) sin- 1 (x) + ~ + C
x2
(e)
2 + 5x + 2 ln Ix - 1 I + C (b) 2sin- 1 (x; 2 )-V4x x+C 2

x4 x3 x2
(f) + + + x + ln Ix - 1 I + C
4 3 2 (c) sin- 1 (x-1)- ✓2x-x 2 +C

Q2
Q8
(a) tan- 1 (x + 1) + C
(a) sin - 1 x + b ) +C
D+c
(
1
✓b2 - a2
(b) ~tan- ( x+
1
(b) tan - l ( x+ a ) + C
2 + 1) + C
_ 1 (2xv3 ✓b2 - a2 ✓b2- a2
(c) v3 tan

Pl
Q3 xn 1 + xn - 1
(a) Use the fact that - - = - - -
1 2 + 6x + 101 - tan- 1 (x + 3) + C 1+ x l+x
(a)
2 ln lx xn .
(b) - -<xnforxE[O , l] .
1 2 1 +x
2I + C
(b)
3
x+
2
+
3 ln I3x + (c) ln 2

(c) ln lx 2 + 6x + 101- 5tan- 1 (x + 3) + C


276 Answers

P2 Q3
(a) Use the same technique as in Chapter 2, Factorise the denominator and use partial fractions.
Exercise 2J , Question 8.
1 Q4
4.
(b) Notice that O :S x (l - x) :S (a)
(c) When P(x) is integrated from Oto 1, it x+l Ax+ B C
- -+--
yields a rational number since it has integer (x2+1)(x - 1) - x2-+1 x- l
coefficients.
then multiply both sides by (x 2 + l)(x - 1)
(d) It is a rational approximation of 1r that
increases in accuracy as n gets larger. (b) C = 1
(c) A= - l
(d) B = 0
Exercise 4E 1 2
ln Ix - 11 -
Partial Fractions (e)
2 ln lx + 11 + C

Fl Q5

(a) j~+_!!__dx
x+a x+b
(a) i tan - l ( ~) -
3
~ ln Ix + 1I+ 10 ln Ix2+ 41 + C
(b) J Ax + B + _!}____ dx
x2 + a x+b
(b) tan- 1 (x) - ln Ix+ 31 +
1 2
2 ln lx + 11 + C
A B C
(c)
J - --2 + - - + - -dx
(x + a) x+a x+b (c) ~ tan - l G-) + ln Ix + 21 + C
Ql Q6
(a) 1 1
x -l - tan- 1 (x) + (() (ln Ix - v'21- ln Ix+ v'21) + C
- - - -- = -A- + --B
3 3v2
(x + l)(x - 2) x+l x-2
Q7
then multiply both sides by (x + l)(x - 2) (a)
(b) A+ B = l
A(x + 1) + B(x - l)(x + 1) + C(x - 1) 2 = x
(c) - 2A + B = - l
then multiply both sides by (x + l)(x - 1) 2

(d) A=~ B = ~ (b) A=~


3' 3
2
2 1
(e)
3 ln Ix + l I + 3 ln Ix - 2I + C (c) C = -
1
4
Q2 1
(d) Substituting x = 0 is easy and yields B =
4
(a) ln I2x + 1I - ln Ix + 3I + C 1 1 1
1
(e)
2(x _
1) + 4 ln Ix - 11 - 4 ln Ix + 11 + C
(b)
6 (ln Ix + 2I - ln Ix + 4I) + C
1 Q8
(c) ln Ix + 1I + ln I2x + 11 + C 4
2 (a) ln Ix I - ln Ix - 3I - x _ +C
1 1 1 3
(d)
4 ln Ix + l I - 3 ln Ix I + 12 ln Ix - 3I+ C (b) Hlnlx-11-lnlx+ll-x =l)+c
10
(e) - ln 2 2 1
3 (c) 9 (ln Ix - l I - ln Ix + 21) - (x + 2) + C
3
7
(f) 4 ln 2 - - ln 3
2
Answers 277

Q9 Exercise 4F
t-formula Substitutions
(a)
2 rx
3 ijx + 3 ln I ijx + 11 +
1
C
Fl
(b) ln xft + 11 +C 2t 1 - t2 2t
I

xft-1 (a) 1 + t2 (b) (c) 1 - t2


1+t 2

(c) ~ H: ~ ~ I + 2 tan - 1 ( ijx)) + C F2


d0
2
= - -2 dt
l+t
QlO
(a) Expand the right-hand side. Ql

(b) 2J3
(a) Htan ml- m ~tan
2
+c
Qll
(a) Expand the right-hand side. {b) ~ tan- 1
G (m +
tan C

(b)

Q12
~
2 (c) tan m +C

(a) Expand the right-hand side. {d) ~ tan- 1 ( ~ tan (m + C


(b) See full worked solutions.

Q13
(a) A (x) = 2 cos x + 3 sin x, B (x) = Q2
2 cos x - 3 sin x
(a) ln I tan ( ! ) - 21 + C
(b) ln 5 2 tan(!)+ 1

Q14 (b) -1 ln 13 + tan ( !0 ) I

+C
3 3 - tan ( 2 )
(a) Expand the right-hand side.
(b) A = 4, B = -4 (c) ~ ln 12 + tan (
4 2 - tan ( 2 )
p I + C
(d) 21r
(d) ~ ln I tan ( ! ) - 31 + C
Pl 4 3 tan(!) - 1
(a) Multiply both sides by x - a 1 and isolate c1 .
(b) Take the limit as x ➔ a 1 and recall the
(e) ~ ln 15 + tan (
5 5 - tan ( 2 )
p I + C

definition of the first derivative.


(f) ~ ln tan ( ! ) - 41 + C
P2 15
I

4tan rn) - 1
(a) Multiply both sides by x + k and isolate c1 .
Then let x = -k and simplify. Q3

(b) Let x = 1 and multiply both sides by n!


(a) - ln Itan ( ! ) - 1 I + C

(b) ln I tan ( ! ) I + C
tan(!)+ 1
278 Answers

1 ln 11 - v'2 - tan (~) I Exercise 4G


2
(c) Integration by Parts
J2 1 + J2 - tan ( !)
Q4 Fl
(a) 11+tan(~)
ln 1 - tan ( 0 ) I + C1
uv - j u'vdx
2
F2
(b)
(c)
ln I sec0 + tan0 j + C2
See full worked solutions.
uvl~ -1' u'v dx

F3
(a) 1
Pl
1 (b) integrated
- +c
1 + tan0
P2 Ql

(a) - 1 tan _ 1 can0)


-- + C
(a) (x - l)ex +C
J5 J5
2 _1 ( 2 tan 0 + l ) C
(b)
2
x
2
( lnx - 1) +
2 C
(b) v'3 tan v'3 +
(c) -2xjx
- ( lnx- 2) +C
3 3
(c) 1
-tan _1 can20)
-- +C
J2 J2 (d) sin x - x cos x +C
(d) ✓tan0 +C (e) x tan x + In I cos x I + C
1
P3 (f)
4 ((2x
2
- 1) sin- 1 (x) + x~) + C
(a) Expand the right-hand side. 1
(g) ((x 2 + 1) tan- 1 (x) - x) +C
2
(b) tan -1 can20)
- - +C 1 + lnx
2 (h) - +c
X

1 .
8 (sm 2x - +C
P4 (i) 2x cos 2x)

1
ln I ✓a +b
2 2
- a +btan (~) I +C
Q2
✓a + b2
✓ a 2 + b2
2
+a - b tan ( !) (a) x(Inx - 1) +C
P5
(b) xsin- 1 (x)+~+C
1r 1
(c) xtan- 1 (x) + ln jx 2 + lj + C
va2-=-I 2
(d) xln(x 2 + 1) + 2tan- 1 (x) - 2x + C
P6
(e) 2(-Jx - l)evx + C
na X

2 (sin (ln x) - +C
(a) (f) cos (ln x))
(a 2 -1) v'a2-=-I
(b) See full worked solutions.
Q3
ex
(a)
2
(sinx - cosx) +C
(b) 2x sin x + (2 - x 2
) cos x +C
2
(c) x ( (ln x) - 2 ln x + 2) + C
Answers 279

Q4 Exercise 4H
6x 3 - 4 Reduction Formulae
(a) (1 + x 3 ) ✓1 + x 3 + C
45
x4 - 2 Fl
-- ✓ 1+x 4 +C
(b)
6 (a) index (b) repeatedly (c) In
2
x 2 sin(x 2) + cos(x ) C
(c) 2 +
F2
x2 - 1 x2 C
(d) --e + parts, parts
2
1 . F3
(e) - (sm( e3 x) - e3 x cos( e3 x)) + C
3 (a) 1 - cos 2 x (b) 1 - sin 2 x (c) sec 2 x - 1
3x 2 + 2 (d) 1 + tan 2 x (e) xn - 1 (f) 1 - xn
(f) - (1 - x 2 ) ✓ 1 - x 2 + C
15

Ql
Q5
1 (a) See full worked solutions.
2 (sec x tan x + ln \ sec x + tan x \) + C
(b) (x 2 -2x+2)ex+ c
Q6
7r - 2 e2 + 1 Q2
(a) (b) --
2 4 (a) See full worked solutions.
1r - 2 ln 2 1 + e-~
(c)
4
(d)
2 (b) ~ x 2 ( ln x) 2 - ~2 x 2 ln x + ~4 x 2 + C
2
7r 7r2 ln 2 1
(e) ----- (f) -
4 32 2 6 Q3
1 1 (a) See full worked solutions.
(g) 1 (h)
2 e 7r2
(b) - - 2
(i) 2ln2 4

Pl Q4
2 l (X) + C
- ✓4 x- x See full worked solutions.
(a) - sin -
2
Q5
1 1 1
~+ sin- (x) + C See full worked solutions.
(b)
2x 2
Q6
(c) X +C
1 + ln x (a) See full worked solutions.

(d) x ln ( x + ✓ x 2 - a2) - ✓ x 2 - a 2 + C (b) 16


35

(e) 2\IT+xsin- 1 (x) + 4 ✓f"=x + C Q7


(a) See full worked solutions .
(f) 2\IT+x tan-
1
( fa) - 2 ln Ifa+ vT+xl + C 243
(b) 1540
P2
See full worked solutions. Q8
(a) See full worked solutions.
32
(b) 315
280 Answers

Q9 P3
(a) See full worked solutions. (a) See full worked solutions.
14yl2 - 16 (b) In= 1f if n is odd, and In= 0 if n is even.
(b)
15

QlO P4
(a) See full worked solutions. (a) See full worked solutions.
8 (b) See full worked solutions.
(b)
105 1 1
(c) n +1 x
Qll
(a) See full worked solutions.
P5
x2 + 2
(b) - -- ~ +C See full worked solutions .
3

Q12
(a ) See full worked solutions. Exercise 41
X 3x 3 Further Substitutions
(b) ---- + --
2
- + -tan- 1
(x)
4(x 2 +1)2 8(x +1) 8
Fl
Q13
They are equal.
(a) See full worked solutions.
(b) 3 ln 2 - 2 F2
(a) u = -x (b) u = ax

Q14 (c) u=a-x (d) u = a+x


(a ) See full worked solutions.
(b) 28 Ql
15 See full worked solutions.

Q15 Q2
See full worked solutions. 1f 1f 1!"2
(a) (b) (c)
4 4 4
Q16
1!"2 1f 1f
(a) See full worked solutions. (d) (e) (f) 1- -
4 4 4
(b) Use the reduction formula recursively.
(c) The integrand of h+ 1 is lower than the Q3
integrand of h since O ::; sin x ::; 1 in the See full worked solutions.
domain of integration.
Q4
(d ) See full worked solutions. 1
1f (a) 3 (b) n (c) 2
(e) - 4
2
Q5
Pl
See full worked solutions.
See full worked solutions.
Q6
P2
See full worked solutions.
(a) See full worked solutions.
(b) 23 Q7
15 See full worked solutions.
Answers 281

Q8 Chapter Review
(a) See full worked solutions. Rl
7r
(b) 12 sin 3 x
(a) sin x - - - + C
3
Q9
See full worked solutions. (b) (x 2 + 1) tan- 1 (x) - x + C

QlO 2 5 2 3

(a) See full worked solutions.


(c)
5(x+1)2- 3(x+1)2+C
(b) a ln (sec a)
(d) ~ ln 11 + x22 1+ C
4 1- x
Qll 1 + e1
(a) See full worked solutions. (e)
2
(b) See full worked solutions.
2
(f)
cot (
X) - 1 +C
(c) ~- 1 2
2

Pl
(g) ~tan- 1 (x; 3
) +C

1r ln 2 1r ln 2
(a) (b) - -
8 2 (h) x ln lx 2 - 11 + ln I l + x I - 2x + C
1- xl
7r
(c) -
4 (i) ln Ix - 1 I + 1
tan - l ( i) + C
P2
1r0
R2
4sin0 3x sin 2x sin 4x C
(a) 8--4-+32+
P3
(a) See full worked solutions.
2
(b) - - 1
l+e
(c) 2 cos y'x + 2 y'x sin y'x + C
P4
(a) See full worked solutions. (d) ln I1 + sin 2 x I + C
2
(b) - -1
l+ e

P5
(a) Use the substitution u = n1r - x. (f) 2 ln [x 2 + 2x + 51 - ; tan- 1 ( x ~ 1) + C
(b) Consider cos 2 x = cos 2 (x + k1r) for k E Z.
ln 3
(g) 4

(h) -2-Jx - 21n 11 - vxl + C


282 Answers

R3 sinx - x cosx
(d)
xsinx + cosx

(e) 2n ln 2

(f) 0

R6
7r 2
(a) - (b) 15
6

(c) 2 ln 2 - 1 (d) n 2
(e) 6ijx - 3rx + 2ft - 61n 11 + rx1 4 3

1 1 8 3n
(f) ln 11 - 2e-xl + ln 11 + e-xl + C (e) 15 (f) 16
6 3
+ sin x + C 1 - Jl ✓3 7r
(g) 2Jl + sin x + ln -~=== I

(g) 2 +3 (h) n - 2
1
1 + Jl + sinx

(h) 4sin- 1 ( ';") - Jx(4 - x) +C (i) ln(l + ✓2) - ~ (j) 7r


3 ✓3

7r
(k) 3✓3 (1) 3n

R4
(a) 2 tan- 1 (ft)+ C
(m) ~ ln m (n)
1 7r
- ln 2 - -
2 8
4 11 7r

(b)
nln2 (o) - - - ln2
3 9 (p) 3✓3
8
1 1
(c) - (q) ln 2 (r) - ln 6
4 5
4
(d) x+--+C 1
9(2e + 1)
3
1- eX (s)

(e) ( 1 + ln x) ln (1 + ln x) - ln x + C
1
(u)
D
(n - 2)
1
4
(f) tan - ( x + +C
R7
1 2n
(g) b (a) x3 +1 (b) 3 ✓3

(h) VSlll2x+C R8
cosx
(a) See full worked solutions.
(i) x ln (x + Jx2=1) - Jx2=l + C 3 2
(b) x ( ln x) - 3x ( ln x) + 6x ln x - 6x + C
R5

(a) Is R9
(a) See full worked solutions.
7r 1 4 8
(b) J2 (b) cos 4 xsinx + cos 2 xsinx + sinx
5 15 15
1
(c)
2 (x - ln (cos x + sin x))
Answers 283

RIO F3
(a) See full worked solutions. (a) Turn v into dx then solve the differential
32 dt
(b) - - equation.
315
(b) Turn a into dv then solve the differential
Rll dt
equation to obtain v = f (t) , then integrate
(a) See full worked solutions.
again with respect to t .
1
(b) -
336 (c) Turn a into v dv and solve the differential
dx
equation and obtain v = f (x), then follow
R12 the same steps as above.
(a) See full worked solutions.
16
(b) Ql
105 1 1
(a) ln t (b) - (c)
t 2t 2
R13
(a) See full worked solutions.
Q2
(b) 1
v2= ln1x;11 2

R14
Q3
(a) See full worked solutions.
Jx+4
(b) 0 (a) V = 4Jx + 4 (b) t= - 1
2

R15 Q4
(a) See full worked solutions. 1
21 ms- 2
(b) 7r
(a) (b) t =
3 1n 7 seconds
4
Q5
2
(a) a= 8x 3 (b) x=--
5. Mechanics 4t + 1

Exercise 5A Q6
3 1 2
Velocity-Displacement Equations v=-- +4x)
(a)
x+2
(b) t=
6(x
Fl Q7
(a)
dv
dt (b) d~Gv 2
)
1
dx dv (a) v 2 = -(16 - · x 4 )
(c) (d) v - 4
dt dx
(b) 2ms- 1 when x = 0
F2 (c) Initially, the particle is at rest at x = 2.

d~ Gv 2
) = :V
dv
Gv x
2
:: )
Since acceleration is then negative, the
particle moves to the left. The particle then
=v- oscillates back and forth between x = ± - 2.
d~x dv Speed is maximised at the origin.
=-X- Acceleration is maximised at the end-points
lt dx X = ±2.
dt
=a
284 Answers

Q8 Q15

(a) -5__
dx 2
(~v =
2
) 2x - 6 (a) a = -5__
dx
(~v
2
2
)

(b) Right , since


(c) When x= 3, v < 0
x > 0 when x
2
= 4 initially.
(b) x = ln (tan (t✓2 + ~D)
(d) Pl
v2
(a) When x = R , a= -g.

(b) a = -5__
dx 2
(~v 2
)

(c) Substitute u 2 = 2gR.


-2
(d) When the particle is projected with speed at
(e) The particle is initially at x = 4 and moves
least y'2gR, then the particle will never
to the right indefinitely.
return to Earth.

Q9
P2
2
v 2: 0 so either lxl :S ~ or lxl 2: 3. But initially the x = -5__ (~v 2 ) and solve the differential equation.
dx 2
1 1
2.
particle is at x = 0 so it must satisfy -
2 :S x :S
QlO Exercise 58
(a) (1
Use -d -v 2 ) .
dx 2
Simple Harmonic Motion

1
(b) x = ln(l + t)
2
(c) x ➔ oo and v ➔ 0
F2
(d) Initially the particle is at the origin and A affects how far back and forth the particle moves.
. 1
moving to the right with speed m s- 1 . The n affects the period of movement.
2 x 0 affects the centre of motion
particle then moves to the right indefinitely. a affects the initial position.

Qll F3
(a) v = ✓ 18 - 2e-x (a) v 2 = n 2 (A 2 - x2)
(b) The particle moves to the right indefinitely (b) v2 = n 2 (A 2 - (x - x 0 ) 2 )
with speed approaching 3 ✓2° m s- 1

F4
Q12 21r
See full worked solutions. T=-
n
Q13 F5
(a) To the right. (a) X = 0 (b) X = ±A
(b) a= x - 2 (c) x = -A (d) x = 0
(c) x =2-2e- t
(d) The particle will approach x = 2 with Ql
decreasing speed indefinitely. (a) Negative sine. (b) Positive sine.
(c) Positive cosine. (d) Negative cosine.
Q14
See full worked solutions.
Answers 285

Q2

(a) A= 3 (b) T = 1r Q9
(a) (b)
51r x( t )
(c) xo = 1 (d) -2 :S x:S4 x (t ) 6 6

(e) v=6 (f) a= 12

3yl3
(g) x = l +2- (h) v = 3
-6
(i) a= -6vl3
Q3 (c)
x( t)
4
t = ~ 1r
5
(a) (b) t = 1r l01r 12
3' 9 9 ' 9

(c) t = ~ 77r (d) t = 51r ll1r


6 ' 18 18 ' 18

Q4
-12
(a) V =0 (b) v = 0
QlO
(c) V=-8 (d) v=4vl3
12 +2
(a) xo = -- = 7
2
Q5
(a) x = - 9x (b) x= - 4(x + l) T = 8 son=~
4
(c) x = -4x (d) i=-9(x-l)
12 - 2
A=--=5
2
Q6 (b) 3:29am
(a) Moving to the right towards O.
(c) 8:31am
(b) At the origin and moving left.
(c) Moving left towards O.
Qll

Q7
1 1
(a) a= !!:_
dx
(~v
2
2
)

(a) A=
2, T = 1r, x 0 = 2 (b) 5
1 1 2
(b) A = - , T = 1r, x 0 = - (c) A=~ T = 1r
2 2 3' 3

(c) A= l, T = 1r, x 0 = 4 5
(d) x = - cos 3t
3
3 1r 1
(d) A=-, T = -, x 0 = -
2 2 2 Q12
(a) a= -4n 2 x (b) v = 61r, a= O
Q8
(c) v = 61r, a= 121r 2 (d) 6, v = -61r
(a) A= 5, T = 1r, x 0 = 0

(b) A=5, T=i , xo=O Q13


3
21r (a) A~ ' ; (b) 3./I
(c) A= 13, T = - , x 0 = 7
3

(d) A= )2, T = i, xo = 1
286 Answers

Q14 Exercise SC
(a) a= -4(x - 2) Projectile Motion
(b) A=3 , T=n , xo=2
(c) The particle travels 9 metres and is at x = 2. Fl
(a) a(t) = -gj
Q15 V ( t) = V cos 0 i + ( - gt + V sin 0) j

(a) The centre of motion is x = 4 and when r (t) = V t cos 0 i + ( - ~ gt 2 + V t sin 0) j


X = 0, V = 0

(b) v 2 2 0 so 8x - x 2 2 0 and hence O :S x :S 8 X


(b) Substitute t = - V into
cos 0
Q16 1 2
y = - gt + Vt sin 0.
A= 2v13, n = 1 2
Q17
Minimum distance is 5 - 3v'2 metres, which occurs
77f 2
whent= . (- x-) + Vsin0 (- x-)
8 = -~g
2 v cos 0 v cos 0
Q18
2n gx2
A=6T= - 2
' 3 = - 2v2 sec 0 + xtan0
Q19
A=~ gx2
= xtan0 - V (1+tan 2 0)
2 2
7f

Q20
(a) a 3 + b3
F2

Q21 T = Vsin0 V 2 sin 2 0


(a)
g
(b) H=
See full worked solutions. 2g

Q22 V 2 sin 20 v2
(c) R= (d) Rmax = -
(a) Consider x = 2 sin t, which has maximum g g
speed 2m s- 1 . According to the claim, when
the particle is at x = 1 it should have speed Ql
1 m s- 1 . But actually, the particle has speed (a) Let y = 0, solve for t, then substitute into x.
v13ms- 1 .
(b) Substitute 0 = 15°, x = 40 into the previous
(b) k = v'2 result.
(c) Usual Cartesian equation derivation, but
Pl this time using V 2 = 80g.
See full worked solutions.
(d) 0=45°,72°
P2
k
(a) Show that this satisfies i = - - x
m Q2
(b) M(t) = ~kA 2 Substitute (x 1 , h) and (x 2 , h) into the Cartesian

P3
equation. Eliminate the -
2 2
i
sec 2 0 parts by
manipulating and dividing the two equations to get
Let Xp = A cos(nt + a) and let XQ = B cos(nt + /3).
h - X1 tan0 x21
h - x2 tan0 x22
Answers 287

Re-arrange to make tan 0 the subject. Alternatively, dR a n


substitute y = h into the Cartesian equation and (d) Let d = 0 and solve to get 0 = + ,
0 2 4
obtain a quadratic in terms of x, which has roots x 1
which is precisely halfway between the
and x2. Find X1 + x2 and X1X2 using the
vertical and the angle of the plane.
sum/product of roots and divide them.
a, n
(e) Find R and T for 0 = + .
Q3 2 4
Equate the y-equations of the trajectory of the
1 Q8
monkey y = h - gt 2 and the dart
2 Substitute P(p, q) into the Cartesian equation and
l h
y = --gt 2 + Vtsin0 to get t = - - . - . Substitute
Vfiln 0
manipulate.
2
this into the x-equation of the dart x = V t cos 0 to Q9
d
get x = h cot 0. But cot 0 = h and so x = d at that V 2 sin 2 0
(a) H = - - - and the max of sin 2 0 is 1.
same point in time. 2g
(b) Standard Cartesian equation and substitute
Q4 v2
gd2 in h = - .
(a) h= - V sec 2 0 + dtan0. This is a 2g
2 2
dh (c) Part (b) is a quadratic in terms of tan 0.
quadratic in terms of tan 0. Let d = 0 and The discriminant is positive.
0
solve for tan 0. (d) The product of roots is greater than 1, which
(b) Substitute the previous result into the cannot occur if tan 01 < 1 and tan 02 < 1.
Cartesian equation.
(c) Use the fact that h > 0. QlO
(a) Substitute (d,O) into the Cartesian equation
gx2 2
Q5 y = - 2v2 sec 0 + X tan 0 + h
(a) First square both sides of iJ to get (b) The previous result is a quadratic in terms
of d. When 0 = a, the derivative with
(y) 2 = g2t 2 - 2Vgtsin0 + V 2 sin 2 0 respect to 0 is zero .

then expand and substitute in (c) Substitute the previous result and d = D
1 back into part (a) and re-arrange.
h = - gt 2 + V t sin 0
2 (d ) Use the double angle formula for cot 2a.
(b) S 2 = (x) 2 + (y) 2. Use the previous result
Qll
and x= V cos 0.
V 2 sin 2 0
(a) Show that H = - - - by usual means,
2g
Q6
(a) x = ut cos a, and x = wt cos f3. When t = T , H
then substitute y = into the Cartesian
x = d and then equate. 2
Do a similar thing for y to get equation and re-arrange to express it as a
(b)
h = T( u sin a, - w sin /3). Divide with
quadratic in terms of x.
d = uT cos a and use the fact that (b) The horizontal distance is
h .
d= tan 0, and the prev10us result.

Q7 then use the sum and product of roots to


(a) Usual Cartesian equation proof. find X1 + x2 and X1X2.
(b) Solve the equation of the trajectory
simultaneously with the equation of the Q12

(c)
ramp, which is y = x tan a.
Use right-angled triangles and note that
First show that V 2 2: .g:
2 sm cos 0
since it clears the

d fence , then substitute into the Cartesian equation.


cos a= Ii"

.1
288 Answers

Q13 dx
(e) For (b) , use v = dt then re-arrange and
V 2 sin 20 solve. For (c) , integrate directly with respect
Show that R( 0) = - - - and calculate R - a and
g tot.
R + b, then subtract the results.
(f) X = 10
Q14
(a) Equate the x components and use the fact Q2
that O < cos 0 < 1
dv
(b) Equate y to find the time to collide. (a) a = -4v 2 then use a= v dx
Substitute this value into the time equation
for the particle being projected horizontally. dx
(b) v = dt then re-arrange and solve for x.

Q15
(c) x = 4 ln 3
. V sin0 3 5
(d) t=
(a) The time of flight is T = - - . When
' g 8, v= 8
t = T , x = d and y = h. Obtain two
expressions and combine them. Q3
(b) Square one of the previous results , use the (a) k =~ (b) k = 2 ln 2 (c) k= 2
fact that cos 2 0 = 1 - sin 2 0, and substitute 8
the other result in to eliminate 0.
Q4
Q16 dv
(a) Use a = v dx
Similar set of steps to Question 7.
dv
Pl (b) Use a= dt
See full worked solutions.
(c) Combine the previous two results.
P2
See full worked solutions. u
(d) x= 2k
1
Exercise 5D (e) t= k ln 2
Resisted Horizontal Motion u
(f) Substitute x = k into v = u - kx , or
2
1
Fl substitute t = k ln 2 into v = ue-kt.

a = X= ~~ = v :~ = 1 Gv 2
)
Q5
20
F2 (a) X= -
k
(b) k= ~
(a) force , velocity 2

(b) gravity Q6
(c) t dv
(a) Use a = v dx

Ql dv
(b) Use a= dt
(a) 5a = -20v
(c) Combine the previous two results or obtain
dv it directly from either one.
(b) a= v -
dx
(d) As t ➔ oo, x ➔ oo so no it does not have a
dv limiting position.
(c) a= -
dt
(d) Equate v and re-arrange.
Answers 289

Pl 1
(e) x = - (2 ln 2 - 3)
Find the limiting position ~ and form the 5

relationship v =f (x). Substitute x = r% x ku into


Q2
and show that v = (100 - r)% x u
dv
P2 (a) a=-
dv dt
(a) a = - (v + v 3 ) and use a = v dx (b) Let v = 0.
dv
(b) a= dt dv
(c) a=v-
(c) x = tan - l ( u) dx
(d) As u ➔ oo, the limiting position approaches (d) Let v = 0.
7r

2
Q3
(e) v =0
(a) a = g - kv then let a = 0.

P3 dv
(b) a=v-
No. A counter-example is a = -v 2 which leads to dx
x = ln(l + ut) where u is the initial speed. This
particle moves to the right indefinitely. (c) Substitute in w = f into the previous result.
P4
dv
No. In fact, the heavier particle travels further. (d) a = dt then repeat as above.

Q4
Exercise SE
Resisted Vertical Motion dv
(a) a= v -
dx

Fl (b) Let v = 0 in the above result.


(a) negative , negative dv
(c) a= -
(b) positive, negative dt
(d) Let v = 0 in the above result.
F2
(e) Take the limit of T as u ➔ oo.
(a) weight
(b) weight, positive, negative
Q5
(a) a= g - kv 2 and let a= 0
F3
(a) terminal (b) Form the acceleration equation for upwards
(b) terminal, t, x motion

(c) 0 2
a = -g - kv = -k ( f+ v
2
) = -k( w
2
+ v2)

Ql dv
then use a = v dx .
(a) 4a = 40 - 20v
dv (c) Let v = w to find the displacement. Then
(b) a= - find the difference between that
dt
displacement and the maximum height .
(c) One is to let a = 0, and the other is to let
t ➔ oo. Either method yields VT= 2
1
(d) - ln 2
5
290 Answers

Q6 (c) Solve the first-order differential equation for


dv y, then substitute back into (b) and solve
(a) a=v- again for x(t). Alternatively just
dx
(b) Re-arrange the previous result and make v 2 differentiate the given result and show that
the subject. it satisfies the differential equation.
(d) The particle approaches x = 0 from the
(c) Either let a = 0 or take the limit as x ➔ oo right, but never crosses it. So the particle
does not even make it to the origin from the
to get w = ft starting point at x = A.

x(t)
(d) Divide the numerator and denominator of A

the term in the bracket by k then substitute

in w 2 = fl_k.

dv t
(e) a = dt and use partial fractions.
(f) Re-arrange the previous result. Pl
(a) See full worked solutions.
Q7 2
(b) As u ➔ oo, V ➔ -vr
7f
(a) a = g - kv and let a = 0
dv P2
(b) a=-
dt See full worked solutions.
(c) Produce a new acceleration equation
P3
dv
a = - g - kv and use a = dt . Find the time for the upwards and downwards
journey separately, then add them.
(d) Substitute the result from (c) into (b) .
P4
See full worked solutions.
Q8
(a) ma= mg - mkv 2
(b) V = fl_ Exercise SF
k
(c) Derive x in terms of v, and substitute Resisted Projectile Motion
X = D, V = W.

(d) Produce a new acceleration equation Fl


dv . . (a) x=O (b) x = -kx
a = -g - kv 2 and use a = v-. Fmd x m i) = -g i) = -ky - g
dx
terms of v and substitute x = H, v = 0.
(e) 0 bserve that H = D since the particle falls F2
the same amount that it travels up. (a) t(t)=-gi_, (b) t(t) = -kt(t) - g i_,
(f) Make W the subject from the previous
result. F3
(g) The impact speed is lesser. parabolic, limiting, infinity

F4
Q9 (a) No air resistance.
(a) Start with our standard acceleration for (b) With air resistance.
simple harmonic motion x = -n 2 x. Then
add the resistive acceleration which is -2nv.
Ql
(b) Calculate the left-hand side and show it is See full worked solutions.
equivalent to (a) .
Answers 291

Q2 Exercise SG
(a) Show that Inclined Planes and Pulleys

x=V -cos
-- ( - e -kt)
0l
Fl
k
(a) N cos 0 + T sin 0 = mg
and then take the limit of x as t ➔ oo N sin 0 - T cos 0 = 0
(b) Show that (b) N = mg cos 0
T = mgsin0
y
g
. = (k . 0)
+ Vsm g
e -kt - k
F2
(a) parallel, perpendicular
and then let iJ = 0
(b) imbalance, net
(c) Substitute the time value from (b) into

y = -1 ( -g + V sm
. 0) g
( l - e -kt) - -t Ql
k k k (a) 0 = N - mgcos30°
ma = mg sin 30°
1
dD (b) ma = mg sin 30° = 20 x 10 x = 100
(d) Let d0 = 0. 2
Newtons.
(c) ma= 100 and m = 20, so a= 5 ms- 2 .
Q3
dx (d) 20ms- 1
(a) x= - (e) 40m
dt
1
(b) x = k In ( 1 + ukt sin 0)
Q2
(c) Make t the subject from the x-equation, and (a) Resolve parallel to the plane.
substitute into the y-equation.
(b) Integrate the previous result.
(d) If Bob's model was correct, then it should
(c) Integrate the previous result.
show a linear trajectory since gravity is not
present. However, the trajectory is clearly
not linear. Q3
(a) Resolve parallel to the plane.
Q4
See full worked solutions. (b) a=(s-f)ms- 2

Pl
(a) We want the magnitude of drag to be Q4
proportional to the square of the speed of (a) See full worked solutions.
the particle, which klili satisfies. We also (b) Set a ~ 0 for the particle to not slide down
want drag to point against the direction of the ramp.
the particle at any time t , which is why i
has been left in vector form. Q5
(b) Note that Iii = J ±2 + y2 . (a) The ramp and pulley are frictionless, so the
pulley only re-directs the tension force
without affecting their magnitude.
P2
(b) The string is inextensible so if we pull the
See full worked solutions.
string by some amount , the other side of the
string gets pulled by the exact same amount .
(c) Resolve parallel to the plane.
T
(d) x=--50vl3
10
292 Answers

100 Q16
T = -(l + v'3) Newtons
v'3 See full worked solutions.
5
x= -(2 - v'3) m s- 2 Pl
3
(a) E =Ek+ Ep and recognise that h = y.

(b) Differentiate both sides of y = 3 x 3 wit h


2 •

See full worked solutions. 2


respect to time, and use the chain rule.
See full worked solutions.
(c) Substitute (b) into (a) and also find the
Set a= 0 in part (a) . actual value of Eby recognising that E is
constant and so we can just use the initial
gravitational potential energy Ep = mgh
3
ma= mg-T where h = .
2
ma = -mg+T
The string is inextensible and taut so both
particles accelerate by the same amount. It
is similar to when a person drags a mass
tied to a rope at a speed of 1 metre per
second. The mass moves at the same speed Chapter Review
as the person as long as the string is taut. Rl

(d) T = -40 Newtons, a= -28 ms - 2


(a) x.. = dx
d (2
1 v2 )
3 3
(b) t = 2 ln 2 seconds
Q8
See full worked solutions. R2
Q9
See full worked solutions.
(a) a= !!_
dx 2
(~v 2
)

(b) To the right .


QlO
(c) No, because v 2 2: 0 so x 2: 3 or x:::; -1 , so it
(a) N + T sin 0 - mg = 0
will never be at x = 2.
Tcos0 - µN = ma
(d) x = 5, a= 3
(b) See full worked solutions.

R3
Qll
(a) N - mg cos 0 = 0
Fpush - µN - mgsin0 = ma
(a) X = d~ Gv 2
)

dx
(b) See full worked solutions. (b) x= -
dt
and solve for x .

Q12 R4
(a) N + Tsina - mgcos0 = 0
Tcosa - mgsin0 - µN = ma
(a) x= d~ Gv 2
)

(b) See full worked solutions. dx


(b) x= -
dt
then solve for x.

Q13 R5
See full worked solutions. (a) x= -x (b) X = -41r 2 x
Q14 (c) x= -9(x - l) (d) x= -4x
See full worked solutions.
(e) x= -4(x - 2) (f) x= -4x
Q15 (g) x= - 9(x - 2) (h) x= -l8x
See full worked solutions.
(i) x= -32(x + 1)
Answers 293

R6 RIO
7r
(a) X = 4 (b) X =- See full worked solutions.
2
RU
R7 R= 40
(a) x = - (x - 3) 3

(b) x 0 = 3, A = 3, T = 21r R12


See full worked solutions.

R8 R13
21r See full worked solutions.
(a) t= - (b) 6\i'3ms - 1 (c) 4 metres
9
R14
R9 See full worked solutions.

(a) A = 2 T = 16 n = ~ R15
' ' 8 (a) See full worked solutions.
(b) x = 10 - 2 cos (-it) (b) See full worked solutions.
(c) 7:40am till 6:20pm (c) There is enough friction so that the object
on the table will not slide.

You might also like